You are on page 1of 95

Index

Business Mathematics
Ratio, Proportion, Indices and Logarithms .............................................................................................................................. 2
Equation .................................................................................................................................................................................................. 8
Linear Inequality .............................................................................................................................................................................. 14
Time Value of Money ...................................................................................................................................................................... 15
Permutation and Combination ................................................................................................................................................... 22
Sequence and Series ........................................................................................................................................................................ 28
Sets, Relations and Functions...................................................................................................................................................... 32
Differentiation and Integration .................................................................................................................................................. 36

Business Statistics
Statistical Description of Data..................................................................................................................................................... 44
Measure of Central Tendency ..................................................................................................................................................... 47
Measure of Dispersion.................................................................................................................................................................... 53
Probability ........................................................................................................................................................................................... 59
Theoretical Distribution ................................................................................................................................................................ 65
Correlation and Regression ......................................................................................................................................................... 69
Index Number .................................................................................................................................................................................... 76

Priority Questions of Logical Reasoning


Number Series, Coding, Decoding ............................................................................................................................................. 81
Direction Sense Test........................................................................................................................................................................ 84
Seating Arrangements .................................................................................................................................................................... 89
Blood Relations ................................................................................................................................................................................. 93

www.escholars.in
1
Business Mathematics
Ratio, Proportion, Indices and Logarithms
✓ Ratio:
A Ratio is the Simplest form of two or numbers or we can say that it is a comparison of the sizes of
the two or more quantities of the same kind by division.
✓ Point of Remember:
➢ If a and b are Two quantities of the same kind, then the fraction a/b is called the ratio of a to
b and it is written as a:b. The quantities a and b are called terms of the ratio a is called the first
term or antecedent and b is called the second term or consequent.
For Example – 3:2
3 is called the antecedent. 2 is called the consequent.
➢ Both the terms of ratio can be multiplied and divided by the same number (non-zero).
➢ Terms of the ratio compared must be in the same unit.
𝑎 𝑎2
➢ Duplicate Ratio of 𝑏
is 𝑏2
𝑎 𝑎3
➢ Triplicate Ratio of 𝑏
is 𝑏3
𝑎 𝑎
➢ Sub-duplicate Ratio of 𝑏
is √𝑏
3
𝑎 √𝑎
➢ Sub-triplicate Ratio of 𝑏
is 3
√𝑏
𝑎 𝑏
➢ Inverse Ratio of 𝑏
is 𝑎
𝑏 𝑎
or Inverse Ratio of 𝑎 is 𝑏
➢ Compounded Ratio → It means Multiplication of the ratios.
For, Example Compounded Ratio of a: b and c: d
𝑎 𝑐
= × = 𝑎𝑐 ∶ 𝑏𝑑
𝑏 𝑑
➢ Continued ratio→ It is the relation (or comparison) between the magnitudes of three or more
quantities of the same kind. The continued ratio of three similar quantities a, b, c is written as
a: b: c.
➢ A ratio a : b is said to be of greater inequality if a>b and of less inequality if a<b.

✓ Proportion
➢ Proportion: It means equality of two Ratios.
➢ If a : b = c : d. It can also be written as a : b :: c : d
𝑎 𝑐
×
𝑏 𝑑
➢ In a : b :: c : d
a & d are called Extremes
and b & c are called Means
𝑎 𝑐
= ⇒ 𝑎𝑑 = 𝑏𝑐
𝑏 𝑑
Product of extremes = Product of means

a b c d

First Second Third Fourth


Proportion Proportion Proportion Proportion

www.escholars.in
2
➢ If there are three quantities a, b, c of the same kind are said to be in continuous proportion
𝑎 𝑏
𝑎: 𝑏 ∷ 𝑏: 𝑐 i.e. 𝑏 = 𝑐 i.e. 𝑏2 = 𝑎𝑐
➢ Componendo:
𝑎 𝑐 𝑎+𝑏 𝑐+𝑑
𝑏
= 𝑑 ⇒ by using component 𝑏 = 𝑑
➢ Dividendo:
𝑎 𝑐 𝑎−𝑏 𝑐−𝑑
𝑏
= 𝑑 = by using dividendo 𝑏 = 𝑑
➢ Componendo and Dividendo:
𝑎+𝑏 𝑐+𝑑
=
𝑎−𝑏 𝑐−𝑑
➢ Addendo:
𝑎 𝑐 𝑒
= = =𝑘
𝑏 𝑑 𝑓
𝑎+𝑐+𝑒
Then, =𝑘
𝑏+𝑑+𝑓
➢ Subtrahendo:
𝑎 𝑐 𝑒
If 𝑏 = 𝑑 = 𝑓 = 𝑘
𝑎−𝑐−𝑒
Then, =𝑘
𝑏−𝑑−𝑓
➢ Alternendo:
If a: b = c: d then a:c = b: d
𝑎 𝑐 𝑎 𝑏
𝑏
= 𝑑 then 𝑐 = 𝑑
➢ Invertendo:
𝑎 𝑐 𝑏 𝑑
If = 𝑡ℎ𝑒𝑛 =
𝑏 𝑑 𝑎 𝑐
✓ Indices
Point of Remember:
➢ (𝑎 + 𝑏)2 = 𝑎2 + 𝑏2 + 2𝑎𝑏
➢ (𝑎 − 𝑏)2 = 𝑎2 + 𝑏2 − 2𝑎𝑏
➢ (𝑎 + 𝑏)3 = 𝑎3 + 𝑏3 + 3𝑎𝑏(𝑎 + 𝑏)
➢ (𝑎 − 𝑏)3 = 𝑎3 − 𝑏3 − 3𝑎𝑏(𝑎 − 𝑏)
➢ 𝑎3 − 𝑏3 = (𝑎 − 𝑏)(𝑎2 + 𝑏2 + 𝑎𝑏)
➢ 𝑎3 + 𝑏3 = (𝑎 + 𝑏)(𝑎2 + 𝑏2 − 𝑎𝑏)
➢ 𝑎2 − 𝑏2 = (𝑎 + 𝑏)(𝑎 − 𝑏)
➢ 𝑎𝑚 × 𝑎𝑛 = 𝑎𝑚+𝑛
𝑎𝑚
➢ = 𝑎𝑚−𝑛
𝑎𝑛
0
➢ 𝑎 =1
1
➢ 𝑎𝑚 = 𝑘 ⟹ 𝑎 = 𝑘 ⁄𝑚
➢ (𝑎𝑚 )𝑛 = 𝑎𝑚𝑛
1
➢ 𝑎−𝑚 = 𝑚
𝑎
1
➢ 𝑎−𝑚
= 𝑎𝑚
➢ If 𝑎 𝑥 = 𝑎 𝑦 𝑡ℎ𝑒𝑛 𝑥=𝑦
➢ 𝑥 𝑎 = 𝑦 𝑎 𝑡ℎ𝑒𝑛 𝑥=𝑦
1
𝑚
➢ √𝑎 = (𝑎)𝑚
✓ Logarithm
Point of Remember:
➢ 𝑙𝑜𝑔𝑎 + 𝑙𝑜𝑔𝑏 = 𝑙𝑜𝑔𝑎𝑏
𝐴
➢ log (𝐵) = log 𝐴 − log 𝐵
➢ 𝑙𝑜𝑔𝑎 𝑏𝑛 = 𝑛 𝑙𝑜𝑔𝑎 𝑏
1
➢ 𝑙𝑜𝑔𝑎𝑚 𝑏 = 𝑚 𝑙𝑜𝑔𝑎 𝑏

www.escholars.in
3
𝑛
➢ log𝑎𝑚 𝑏𝑛 = 𝑚 𝑙𝑜𝑔𝑎 𝑏
➢ 𝑎log 𝑏 = 𝑏log 𝑎
➢ log𝑎 𝑎 = 1
log 𝑏
➢ log𝑎 𝑏 = log𝑒 𝑎
𝑒
➢ log 1 = 0
➢ log 10 = 1
When 𝑎 𝑥 = 𝑏 𝑡ℎ𝑒𝑛 𝑙𝑜𝑔𝑎𝑏 = 𝑥
23 = 8
➢ 𝑙𝑜𝑔28 = 3
➢ 𝑎𝑙𝑜𝑔𝑛 = 𝑛
➢ log𝑏 𝑎 × log 𝑐 𝑏 ⟹ log𝑐 𝑎
Apply base change formula
log 𝑎 log 𝑏 log 𝑎
𝑙𝑜𝑔 𝑏
× 𝑙𝑜𝑔 𝑐 = 𝑙𝑜𝑔 𝑐 = 𝑙𝑜𝑔𝑐 𝑎

Q. No. Priority Questions


1. Fourth proportional to x, 2x, (x+1) is:
Ans. Let the fourth proportional to x, 2x, (x+1) be t, then,
𝑥 𝑥+1
=
2𝑥 𝑡
1 𝑥+1
2
= 𝑡
t = 2x + 2
∴ Fourth proportional to x, 2x, (x+1) is (2x+2)
i.e. x : 2x :: (x+1) (2x+2)
2. a)
2𝑛 +2𝑛–1
2𝑛+1 –2𝑛
b) If 2x × 3y × 5z = 360 Then what is the value of x, y, z,
Ans. a) 2n + 2n – 1/2n–1 – 2n
2n + 2n*2 – 1/2n*2 – 1 – 2n
2n (1 + 2 – 1)/2n(2 – 1 – 1)
= 3/2
b) 2x × 3y × 5z = 360. ……………………. (1)
The factors of 360 are:
23 × 32 × 5.
∴ 23 × 32 × 51 = 360 ………………….... (2)
On comparing (1) and (2), we get;
x = 3, y = 2 and z = 1
3. The students of two classes are in the ratio 5 : 7, if 10 students left from each class, the
remaining students are in the ratio of 4 : 6 then the number of students in each class is:
Ans. Let the ratio be 5x : 7x
If 10 student left, Ratio became 4 : 6
5𝑥–10 4
=
7𝑥–10 6
30x – 60 = 28x – 40
2x = 20
x = 10
∴ No. of students in each class is 5x and 7x
i.e. 50, 70
4. The ratio Compounded of 4 : 5 and sub–duplicate of a:9 is 8:15. Then value of “a” is:
Ans. Sub duplicate ratio of a : 9 = √𝑎: √9, Compound Ratio (C.R.) = 8:15

www.escholars.in
4
Compound Ratio of 4: 5 and sub duplicate ratio of a: 9 is given by
4 √𝑎
C.R = ×
5 √9
8 4 √𝑎
15
=5×
√9
8×5×√9
√𝑎 = 15×4
8×5×3
√𝑎 = 15×4
√𝑎 = 2
2
On squaring (√𝑎) = 22
a=4
5. a) The value of log4 9. Log3 2 is:
b) The value of (logy x . logz y . logx z)3 is
Ans. a) Log49 . log32
𝑙𝑜𝑔9 𝑙𝑜𝑔2
= .
𝑙𝑜𝑔4 𝑙𝑜𝑔3
𝑙𝑜𝑔32 𝑙𝑜𝑔2
= 𝑙𝑜𝑔22 . 𝑙𝑜𝑔3
2𝑙𝑜𝑔3 𝑙𝑜𝑔2
= .
2𝑙𝑜𝑔2 𝑙𝑜𝑔3
=1
b) (logyx . logzy . logxz)3
𝑙𝑜𝑔𝑥 𝑙𝑜𝑔𝑦 𝑙𝑜𝑔𝑧 3
= (𝑙𝑜𝑔𝑦 . 𝑙𝑜𝑔𝑧 . 𝑙𝑜𝑔𝑧)
= (1)3
=1
6. For three months, the salary of a person are in the ratio 2 : 4 : 5. If the difference between the
product of salaries of the first two months and last two months is ₹4,80,00,000; then the
salary of the person for the second month will be:
Ans. Ratio of the salary of a person in three months = 2 : 4 : 5
Let, Salary of Ist month = 2x
Salary of IInd month = 4x
Salary of IIIrd month = 5x
Given
(Salary of Product of last two months) – (Salary of Product Ist two months)
= 4, 80,00,000
(4x–5x) – (2x.4x) = 4,80,00,000
20x2 – 8x2 = 4,80,00,000
12x2 = 4,80,00,000
x2 = 40,00,000
x = 2,000
Salary of the person for second month = 4x = 4 × 2,000 = 8,000
7. If px = q, qy = r and r2 = p6, then the value of xyz will be:
Ans. If px = q, qy = r and rz = p6
q = px, qy = r and rz = p6
(qy)z = p6
[(px)y]z = p6
Pxyz = p6 = xyz = 6
8. a) The value of log5 3 × log3 4 × log2 5.
b) If log4 (x2 + x) – log4 (x+1) = 2, then the value of X is:
Ans. a) log5 3 × log3 4 × log2 5

www.escholars.in
5
𝑙𝑜𝑔3 𝑙𝑜𝑔4 𝑙𝑜𝑔5
= × ×
𝑙𝑜𝑔5 𝑙𝑜𝑔3 𝑙𝑜𝑔2
log 4
= log 2
log 22
= log 2
2 log 2
= =2
log 2
b) If log4 (x2 + x) – log4 (x+1) = 2
(𝑥 2 +𝑥)
⇒ 𝑙𝑜𝑔4 { (𝑥+1) } = 2
𝑥(𝑥+1)
⇒ 𝑙𝑜𝑔4 { (𝑥+1) } = 2
⇒ 𝑙𝑜𝑔4 𝑥 = 2
x = 42
x = 16
9. If p : q is the sub–duplicate ratio of p – x2 : q – x2, then x2 is:
Ans. Sub duplicate ratio of (p–x2) : (q–x2) = √𝑝– 𝑥 2 : √𝑞– 𝑥 2
p:q = √𝑝– 𝑥 2 : √𝑞– 𝑥 2
𝑃 √𝑝–𝑥 2
=
𝑄 √𝑞–𝑥 2
an squaring both side
𝑝2 𝑝–𝑥2
=
𝑞2 𝑞–𝑥 2
p2 (q–x2) = q2(p–x2)
p2q – p2x2 = q2p – q2x2
p q – q2p = p2x2 – q2x2
2

pq (p–q) = (p2–q2)x2
pq (p–q) = (p+q) (p–q)x2
𝑝𝑞(𝑝–𝑞)
x2 = (𝑝+𝑞) (𝑃–𝑞)
𝑝𝑞
x2 = (𝑝+𝑞)
10. If P = 𝑥 1/3 + 𝑥 –1/3 then P3 = 3P =
Ans. If P = 𝑥 1/3 + 𝑛–1/𝑝 then P3 – 3P
Given P = 𝑥 1/3 + 𝑥 –1/𝑝 …………………… (1)
Cube on both side
3
𝑃3 = (𝑥 1/3 + 𝑥 –1/𝑝 )
3 3
𝑃3 = (𝑥 1/3 ) + (𝑥 –1/𝑝 ) + 3𝑥 1/3 . 𝑥 –1/3 (𝑥1/3 + 𝑥 –1/3)
= 𝑥 + 𝑥 –1 + 3 × 1 × 𝑃
1
𝑃3 = 𝑥 + 𝑥 + 3𝑃
1
𝑃3 – 3𝑃 = 𝑥 + 𝑥
11. A bag contains 105 coins containing some 50 paise, and 25 paise coins. The ratio of the
number of these coins is 4:3. The total value (in ₹) in the bag is
Ans. 4
No. of 50 paise coins = × 105 = 60
7
3
No. of 25 paise coins = 7 × 105 = 45
Value of 1 50–paisa coin = ₹0.50
Therefore, value of 60 50–paisa coins = 60 × ₹0.50 = ₹30
Value of 1 25–paisa coin = ₹0.25
Therefore, value of 45 25–paisa coins = 45 × ₹0.25 = ₹11.25
Therefore, total value = ₹30 + ₹11.25 = ₹41.25
www.escholars.in
6
12. a) log √3 = 6 base a, then ‘a’ will be:
b) 𝑙𝑜𝑔√2 64 is equal to:
Ans. a) Here 𝑙𝑜𝑔√3 a = 6 (∵ 𝑙𝑜𝑔𝑎 𝑏 = 𝑛 ⇒ 𝑏 = 𝑎𝑛 )
6
⇒ 𝑎 = (√3)
𝑠3
⇒ 𝑎 = (31/2 )
a = 33
a = 27
log 64 log 26 6𝑙𝑜𝑔2
b) 𝑙𝑜𝑔√2 64 = = 1 = 1 = 6 × 2 = 12
log √2 𝑙𝑜𝑔2
log(2)2 2

13. X, Y, Z together starts a business. If X invests 3 times as much as Y invests and Y invests two
third of what Z invests, then the ratio of capitals of X, Y, Z is:
Ans. 2
Given x = 3y and y = 3 𝑧
𝑥 3 𝑦 2
= 𝑎𝑛𝑑 =
𝑦 1 𝑧 3
x : y = 3 : 1 and y : z = 2 : 3
=3×2:1×2
=6:2
x:y:z=6:2:3
14. If log 2 = 0.3010 and log 3 = 0.4771, then the value of log 24 is:
Ans. If log 2 = 0.3010 and log 3 = 0.4771
then log 24 = log (2×2×2×3)
= log 2 + log 2 + log 2 + log 3
= 3 log 2 + log 3
= 3 × 0.3010 + 0.4771
= 0.9030 + 0.4771
= 1.3801
15. 3𝑥–2 2
is the duplicate ratio of 3 then find the value of x:
5𝑥+6
Ans. ∵
3𝑥–2
is the duplicate ratio of i.e.
2 3𝑥–2
=
22
5𝑥+6 3 5𝑥+6 32
3𝑥–2 4
⇒ =
5𝑥+6 9
27x – 18 = 20x + 24
27x – 20x = 24 + 18
7x = 42
x=6

www.escholars.in
7
Equation
✓ Equation is defined to be a mathematical statement of equality.
✓ A Simple equation is one unknown 𝑥 is in the form of 𝑎𝑥 + 𝑏 = 0
Where a, b are known constants.
A Simple equation has only one root.
✓ Elimination method: In this method, two given linear equations are reduced to a linear equation in
one unknown by eliminating one of the unknown and then solving for the other unknown.
✓ Cross Multiplication method: let two equations be: -
a1 𝑥 + b1 𝑦 + c1 = 0
a2 𝑥 + b2 𝑦 + c2 = 0
𝑏1 𝑐2 −𝑏2 𝑐1 𝑐1 𝑎2 − 𝑐2 𝑎1
𝑥= 𝑦=
𝑎1 𝑏2 −𝑎2 𝑏1 𝑎1 𝑏2 −𝑎2 𝑏1
✓ An equation of the form 𝑎𝑥 2 + 𝑏𝑥 + 𝑐 = 0 where 𝑥 is a variable, and a, b, c are constants with a ≠ 0
is called a quadratic equation or equation of the second degree.
✓ When b=0, the equation is called a pure quadratic equation; when b ≠0, the equation is called an
affected quadratic: -
✓ The roots of a quadratic equation: -
−b±√b2 −4ac
✓ 𝑥=
2a
✓ The Sum and Product of the roots of the quadratic equation
b Coefficient of 𝑥
Sum of roots = − = −
a Coefficient of 𝑥2
c constant term
Product of roots = =
a coefficient of 𝑥 2
✓ To construct a quadratic equation for the equation 𝑎𝑥 2 + 𝑏𝑥 + 𝑐 = 0 we have
𝒙𝟐−(Sum of roots)𝑥 + Product of roots=0.
✓ Conditions for solvability of pair of liner equations
S. Pairs of lines Condition Graphical Algebric
No. representation interpretation
1. a1 𝑥 + b1 𝑦 + c1 = 0 𝒂 𝟏 𝒃𝟏 Intersecting Unique Solution

a2 𝑥 + b2 𝑦 + c2 = 0 𝒂𝟐 𝒃𝟐 lines (Consistent)

2. a1 𝑥 + b1 𝑦 + c1 = 0 𝒂𝟏 𝒃𝟏 𝒄𝟏 Coincident Infinitely many


a2 𝑥 + b2 𝑦 + c2 = 0 = = lines Solution
𝒂𝟐 𝒃𝟐 𝒄𝟐
(Consistent)
3. a1 𝑥 + b1 𝑦 + c1 = 0 𝒂𝟏 𝒃𝟏 𝒄𝟏 Parallel lines
a2 𝑥 + b2 𝑦 + c2 = 0 = ≠ No Solution
𝒂𝟐 𝒃𝟐 𝒄𝟐
(Inconsistent)

✓ Nature of the roots


−b ± √b 2 − 4ac
𝒙=
2a
➢ If 𝒃𝟐 − 𝟒𝒂𝒄 = 𝟎 the roots are real and equal.
➢ If 𝒃𝟐 − 𝟒𝒂𝒄 > 𝟎 then the roots are real and unequal (or distinct)
➢ If 𝒃𝟐 − 𝟒𝒂𝒄 < 𝟎 then the roots are Imaginary
➢ If 𝒃𝟐 − 𝟒𝒂𝒄 is Perfect Square (≠0); the roots are real, rational and unequal (distinct).
➢ If 𝒃𝟐 − 𝟒𝒂𝒄 > 𝟎 but not a perfect square. The roots are real Irrational and Unequal.
➢ Since 𝑏2 − 4𝑎𝑐 discriminates the roots 𝑏2 − 4𝑎𝑐 is called discriminant in the equation.
www.escholars.in
8
➢ 𝒂𝒙𝟐 + 𝒃𝒙 + 𝒄 = 𝟎 as it actually discriminates between the roots.

Special points
✓ Irrational roots occur in pairs means if one root is 𝑎 − √𝑏 another root will be 𝑎 + √𝑏
✓ If one root is the reciprocal of another root, then the product of roots will be 1.
✓ If two roots of the equation are equal but with opposite signs, then the sum of roots will be zero.

Q. Priority Questions
No.
1. One root of the equation:
x2 – 2 (5 + m)x + 3(7 + m) = 0 is
reciprocal of the other.
Find the value of M.
Ans. If one root of the equation is reciprocal of the other then the product of the roots is 1.
Now, x2 – 2(5 + m) x + 3 (7 + m) = 0
x2 – (10 + 2m) x + (21 + 3m) = 0
Since the roots of the equation is reciprocal of each other.
Therefore, product of the roots = 1
𝑐
=1
𝑎
21+3𝑚
1
=1
21 + 3m = 1
3m = -20
m = 20/3 = -6.6 or -7
Therefore, the value of m is 7.
2. If the length of a rectangle is 5 cm more than the breadth and if the perimeter of the rectangle
is 40 cm, then the length & breadth of the rectangle will be:
Ans. Let the breadth of the rectangle be x cm. Therefore, length = (x+5) cm.
Now, Perimeter = 2(l + b)
40 = 2[(x + 5) + x]
20 = x + 5 +x
20 = 2x + 5
2x = 20 – 5
2x = 15
15
X= 2
X = 7.5
So, breadth = x = 7.5 cm and length = x + 5 = 7.5 + 5 = 12.5 cm.
3. If roots of equation x2 + x + r = 0 are ‘ ∝ ’ and ‘𝛽’ and ∝3 + 3 = –6. Find the value ‘r’?
Ans. Given : ‘’ and ‘’ are roots of x2 + x + r = 0
and 3 + 3 = –6
Quadratic Equation: x2 + x + r = 0
here a = 1, b = 1 and c = r
−𝑏
∴ Sum of Roots :  +  = 𝑎 = –1
𝑐
and Product of Roots:  –  = 𝑎 = r
also
∵ 3 + 3 = ( + )3 – 3 (a + b)
on putting the values,
– 6 = (-1)3 – 3r (-1)
⇒ r = -5/3
4. The quadratic equation x2 – 2kx + 16 = 0 will have equal roots when the value of ‘k’ is__________.

www.escholars.in
9
Ans. Given, Quadratic Equation
x2 – 2kx + 16 = 0
Comparing from ax2 + bx + c = 0
We get
a = 1, b = -2k, c = 16
If Roots of Quadratic Equation are equal
So, D = b2 – 4ac = 0
⇒ (-2k)2 – 4 × 1 × 16 = 0
⇒ 4k2 – 64 = 0
⇒ 4k2 = 64
64
K2 = 4 16
K = +4
5. If b2 – 4ac is a perfect square but not equal to zero than the roots are:
Ans. If b2 – 4ac ± 0 and have perfect square i.e. D > 0 and have perfect square
So, Roots of Q.E are real unequal and Rational
6. The roots of the equation y3 + y2 – y – 1 = 0 are:
Ans. Given Equation
y3 + y2 – y – 1 = 0
y2 (y + 1) – 1(y + 1) = 0
(y + 1) (y2 – 1) = 0
(y + 1) (y + 1) (y – 1) = 0
then y = -1, -1, 1
7. The roots of the cubic equation x3 – 7x + 6 = 0 are:
Ans. x3 – 7x + 6 = 0
x3 – x2 + x2 – 7x + 6 = 0
x2 (x – 1) + x2 – x – 6x + 6 = 0
x2 (x – 1) + x (x – 1) – 6 (x – 1) = 0
(x – 1) (x2 + x – 6) = 0
(x – 1) (x2 + 3x – 2x – 6) = 0
(x – 1) [x (x + 3) – 2(x + 3)] = 0
(x – 1) (x + 3) (x – 2) = 0
If x – 1 = 0 If x + 3 = 0 If x – 2 = 0
𝑥=1 𝑥 = −3 𝑥=2
8. A cottage industry produces a certain number of pottery articles in a day. It was observed on a
particular day that the cost of each article (in ₹) was 2 more than thrice the number of articles
produced on that day. If the total cost of production on that day was ₹800, the number of
articles produced was
Ans. Let No. of articles = N
cost of 1 articles (x) = (3N + 2)
Total cost of production on that day = 800
N × = 800
N (3N + 2) = 800
3N2 + 2N – 800 = 0
3N2 + 50N – 48N – 800 = 0
N(3N + 50) – 16 (3N + 50) = 0
(3N + 50) (N – 16) = 0
If 3N + 50 = 0 and N – 16 = 0
50
N = – (Impossible) N = 16
3
9. 1
When two roots of quadratic equation are , 𝑎 then what will be the quadratic equation:
Ans. 1
If Roots of Q.E are , &  = ,  =
1
𝑎 

www.escholars.in
10
Sum of Roots (S) =  + 
1
=+
2 +1
=( )

Product of Roots (P) =  . 
1
=. =1

Q, E is given by
x2 – 5x + p = 0
2 +1
x2 - ( 
)x+1=0
x2 – (2 + 1) x + a = 0
10. If one root is half of the other of a quadratic equation and the difference in roots is a, then the
equation is
Ans. Let one root be , and another root be .
1 
Since one root is half of the other root, we have  = 2 ×  = 2
Since the difference of the roots is a, we have:

– =a
2
2−
⇒ 2
=a
 2𝑎
Therefore,  = 2 = 2 = a
Sum of roots = 2a + a = 3a
Product of roots = 2a × a = 2a2
When the roots are known, the equation is given by:
x2 – (Sum of Roots) x + Product of Roots = 0
Therefore, the equation is:
x2 – 3ax + 2a2 = 0
11. A plumber can be paid either ₹600 and ₹50 per hour or ₹170 per hour. If the job takes ‘n’ hour,
for what value of ‘n’ the method earns better wages for the plumber?
Ans. Let Plumber takes ‘n’ hour to complete the Jobs
Given, 600 + 50 × n = 170 × n
600 + 50n = 170n
600 = 170n – 50n
600 = 120n
600
n=
120
n=5
12. What will be the value of k, if the roots the equation (k-4)x2 – 2k x + (k+5) = 0 are equal?
Ans. Given, Q.E
(k-4)x2 – 2kx + (k + 5) = 0
Computing from ax2 + bx + c = 0
We get a = (k – 4), b = -2k, c = (k + 5)
If roots of Q.E are equal
Then D = 0
b2 – 4ac = 0
(-2k)2 – 4(k-4) (k+5) = 0
4k2 – 4(k2 + 5k – 4k – 20) = 0
4k2 – 4k2 – 4k + 80 = 0
4k = 80
80
K = = 20
4

www.escholars.in
11
13. If the cost of 3 bags and 4 pens is ₹257 whereas the cost of 4 bags and 3 pens is ₹324, then the
cost of one bag is:
Ans. Let the cost of 1 bag = ₹x
The cost of 1 pen = ₹y
According to 1st condition and 2nd condition are
3x + 4y = 257 (1)
4x + 3y = 324 (2)
Multiply by 3 in equation (1)
9x + 12y = 771 (3)
Multiply by (4) in equation (2) (4)
16x + 12y = 1296
eq. (4) – eq. (3)
16x + 12y = 1296
9x + 12y = 771
— — —
7x = 525
525
x= = 75
7
The cost of one bag = 75.
14. The solution of the following system of linear equations 2x – 5y + 4 = 0 and 2x + y – 8 = 0 will
be:
Ans. Given Equation 2x – 5y + 4 = 0 ⇒ 2x – 5y = -4 ________ (1)
and 2x + y – 8 = 0 ⇒ 2x + y = 8 _________ (2)
eq. (2) – eq. (1)
2x + y = 8
2x – 5y = -4
- + +
6y = 12
12
y= =2
6
Putting y = 2 in eq. (2)
2x + 2 = 8
2x = 8–2
2x = 6
6
x=2=3
x = 3, y = 2
15. The sum of square of any real positive quantity and its reciprocal is never less than:
Ans. “Real Positive Quantity” ranges from 0 to infinity.
Let the number be 0.1.
1
Now, 0.12 + = 10.01
0.1
Let the number be 0.5.
1
Now, 0.52 + 0.5 = 2.25
Let the number be 1.
1
Now, 12 + 1 = 1 + 1 = 2
Let the number be 2.
1 1
Now, 22 + 2 = 4 + 2 = 4.5
Let the number be 3.
1 1
Now, 33 + 3 = 9 + 3 = 9.33
Clearly, the sum will never be less than 2.

www.escholars.in
12
www.escholars.in
13
Linear Inequality
✓ Two real numbers or algebraic expressions related by the symbols <, >, ≤ 𝑜𝑟 ≥ form an inequality.
✓ Equal number added to (subtracted from) both side of the inequality without affecting the sign of
inequality.
✓ Both sides of an inequality can be multiplied or divided by the same positive number (non-zero).
But when both sides multiplied or divided by the negative number, then the sign of inequality is
reversed.

www.escholars.in
14
Time Value of Money
𝑃×𝑅×𝑇
✓ Simple Interest = 100
P = Principal, R = Rate of Interest, T = Time
✓ Amount = Principal + Interest
If the Simple Interest, Then
𝑃×𝑅×𝑇
𝑨 = 𝑃𝑟𝑖𝑛𝑐𝑖𝑝𝑎𝑙 + 𝑜𝑟 𝑨 = 𝑃 + 𝑃 𝑖 𝑇
100
𝑨 = 𝑃(1 + 𝑖𝑇)
𝑅
A= Amount, R= Rate of interest, 𝒊 = 100
100
✓ If ‘r’ is the Simple rate of Interest, then Amount becomes double of itself in 𝑟
years.
200
✓ If ‘r’ is the Simple rate of Interest, then Amount becomes triple of itself in 𝑟 years.
✓ When the Interest is compounded. The present value P of the amount, A, due at the end of n period
at the rate of I per interest period may be obtained by:
Formula to find the amount:
𝑨 = 𝑃(1 + 𝑖)𝑛
A = Amount
P = Principal
𝑅𝑎𝑡𝑒 𝑜𝑓 𝐼𝑛𝑡𝑒𝑟𝑒𝑠𝑡
𝒊= 100
𝒏 = 𝑇𝑜𝑡𝑎𝑙 𝑛𝑜. 𝑜𝑓 𝑐𝑜𝑛𝑣𝑒𝑟𝑠𝑖𝑜𝑛𝑠.
✓ When Compound Interest is to be find:
𝑪. 𝑰. = 𝑃 [(1 + 𝑖)𝑛 − 1]
C.I. = Compound Interest =, P = Principal, n = Total number of conversions
𝑅𝑎𝑡𝑒 𝑜𝑓 𝐼𝑛𝑡𝑒𝑟𝑒𝑠𝑡
𝒊=
100
72
✓ If ‘r’ is the Compound rate of Interest, then Amount becomes double itself in years.
𝑟
114
✓ If ‘r’ is the Compound rate of Interest, then it will become triple of itself in 𝑟 years.
✓ If the difference between S.I. and C.I. is given for 2 years, in such a case P will be
𝑑×(100)2
𝑷 = 𝑟2
d = difference between Simple Interest and Compound Interest.
r = rate of interest
✓ If the difference between Simple Interest and Compound Interest is given for 3 years, in such a case,
‘P’ will be
𝑑×(100)3
𝑷 = 𝑟 2 (𝑟+300)
✓ If a sum of money deposited in a bank becomes 𝐴1 in 𝑡1, years, & A2 in 𝑡2 years, then amount deposit
initially. Simple
𝐴 𝑡 −𝑡 𝐴
= 21 2 1 Interest
𝑡1 −𝑡2
𝑚−1
✓ If the sum of money becomes ‘n’ times in ‘t’ years. Then it will become ‘M’ times in ( 𝑛−1 ) × 𝑡 years
from simple interest.
✓ Effective Rate of interest: To compute Rate of Interest Compounded Annually;
𝑬 = (1 + 𝑖)𝑛 − 1
𝑅𝑎𝑡𝑒 𝑜𝑓 𝐼𝑛𝑡𝑒𝑟𝑒𝑠𝑡
𝒊= , 𝒏 = 𝑡𝑜𝑡𝑎𝑙 𝑛𝑢𝑚𝑏𝑒𝑟 𝑜𝑓 𝑐𝑜𝑛𝑣𝑒𝑟𝑠𝑖𝑜𝑛𝑠
100
𝑬 = 𝐸𝑓𝑓𝑒𝑐𝑡𝑖𝑣𝑒 𝑅𝑎𝑡𝑒 𝑜𝑓 𝐼𝑛𝑡𝑒𝑟𝑒𝑠𝑡

www.escholars.in
15
✓ Annuity: Regular Payment at Regular Interval

Ordinary Annuity/ Regular Annuity Annuity Due / Annuity


annuity Immediate

First Payment is made at the First Payment at the


end of the period. beginning of period.

For Example – Rent Paid, For Example – Recurring


Repayment of loan. deposit, Insurance Premium.
Future Value Present Value
Ordinary Annuity/ (1 + 𝑖)𝑛 − 1 1 − (1 + 𝑖)−𝑛
Regular Annuity 𝑅[ ] 𝑅[ ]
𝑖 𝑖
Annuity Due / Annuity (1+𝑖)𝑛 −1
𝑅[ ](1+i) 1 − (1 + 𝑖)−(𝑛−1)
Immediate 𝑖 𝑅[ ]+𝑅
𝑖

𝑅𝑎𝑡𝑒 𝑜𝑓 𝐼𝑛𝑡𝑒𝑟𝑒𝑠𝑡
R = Annuity, 𝑖 = 100
𝒏 = 𝑡𝑜𝑡𝑎𝑙 𝑛𝑢𝑚𝑏𝑒𝑟 𝑜𝑓 𝑐𝑜𝑛𝑣𝑒𝑟𝑠𝑖𝑜𝑛𝑠
✓ Sinking Fund: It is a fund created for a specified purpose by way of periodic payment over a time
period at a special Rate of Interest. Interest is compounded at the end of every period. The amount
of sinking fund deposit is computed from
(1 + 𝑖)𝑛 − 1
𝑨 = 𝑹[ ]
𝑖
Amount of Sinking Fund = Future value.
✓ Investment Decision
Benefits from Asset > Cost of Asset → Buy the Asset
Benefit from Assets < Cost of Asset → Take It on lease

✓ Benefits form Asset: Benefits Form Asset

Reduction in Cost of Increase in Revenue


Production (Sales)
Step I → Present Value of Benefits from Asset
1−(1+𝑖)−𝑛
𝑷. 𝑽 = 𝑅 [ 𝑖
]
𝑅𝑎𝑡𝑒 𝑜𝑓 𝐼𝑛𝑡𝑒𝑟𝑒𝑠𝑡
R = Annuity, 𝑖 = 100
𝒏 = 𝑡𝑜𝑡𝑎𝑙 𝑛𝑢𝑚𝑏𝑒𝑟 𝑜𝑓 𝑐𝑜𝑛𝑣𝑒𝑟𝑠𝑖𝑜𝑛𝑠
Step II→ Cost of Asset → Given in the question
Step III → Step I > Step II (Buy it)
Step I < Step II (Ignore the Proposal)
Step I = Step II (Indifferent)
✓ Bond: A bond is a debt security in which the Issuer owes the holder debt and is obliged to Repay the
Principal and interest. Bonds are generally issued for a fixed term, generally longer than one year.
✓ Valuation of Bond:
Step 1 → Find Interest Receivable
= Face Value × Coupon Rate
Step 2→ Find Present value of Interest receivable

www.escholars.in
16
1−(1+𝑖)−𝑛
= 𝑅[ 𝑖
]
𝑅𝑒𝑞𝑢𝑖𝑟𝑒𝑑 𝑅𝑎𝑡𝑒 𝑜𝑓 𝑅𝑒𝑡𝑢𝑟𝑛
R = Interest Receivables, 𝒊 = 100
Step 3 → Find Present value of Redemption Value
𝐴
𝑷. 𝑽. = (1+𝑖)𝑛
P.V. = Present Value
A = Future Value
Step 4 → Step 2 + Step 3
In step 4 value of Bond is find
✓ Perpetuity: It is an annuity in which the Periodic payments or Receipts begins on a fixed date and
continue Indefinitely or perpetually.
𝑹
✓ Present Value = 𝒊
R → Annuity Per Month/Half-yearly/Yearly
𝑖 → Interest Per Month/Half-yearly/Yearly
✓ Growing Perpetuity: A Stream of cash flows that grow at a constant rate forever is known as
growing perpetuity.
𝑅
Present Value Annuity =
𝑖−𝑔
R → Annuity Per Month/Half-yearly/Yearly
𝑔 → Growing rate Per Month/Half-yearly/Yearly
i and g should be in the same quantities.
𝟏
✓ P.V Factor = 𝒏 (𝟏+𝒊)
✓ Net present Value:
➢ Net present Value: Present Value of Cash Inflow – Present Value of Cash Outflow.
➢ Steps to Calculating Net Present Value are;
▪ Determine the Net Cash Inflow in each year of the Investment.
▪ Select rate of return or discounted rate, or weighted average cost of capital.
▪ Find the discount factor for each year based on the desired rate of return selected.
▪ Determine the present values of net Cash flows by multiplying the cash flows by respective
discount factors of the Respective period called Present Value of Cash flows.
▪ Total the Amount of all P.V’s. of Cash flows.
▪ Decision Rule:
❖ N.P.V. > 0 [Accept the proposal]
❖ N.P.V. < 0 [Reject the proposal]
✓ Nominal Rate of Interest ⇒ Nominal Rate of Interest = Real Interest Rate + Inflation.
➢ CAGR: Compound Annuity Growth Rate is an important factor is business valuation, and
particularly used in growth Industries to compare the growth rates of two investments. CAGR
is often used to describe the growth over a period of time of some elements of the business
for Example (Revenue, units delivered, Registered users etc.)
1
𝑉(t ) t −t0
➢ 𝑪𝑨𝑮𝑹 = [[ 𝑉(t𝑛) ] 𝑛 − 1] × 100
0

➢ V (𝐭𝒏 ) = Value at the End period.


➢ V (𝐭𝟎 ) = Value at the Beginning period.
✓ Present Value of Deferred Annuity:
➢ First find the present value of an ordinary consisting of payment of ₹R each made at the end
of each of m payment intervals, together with payments of ₹R each made at the end of each of
n additional payment intervals. The present value of this annuity is
𝟏−(𝟏+𝒊)−𝒎−𝒏
𝑷𝟏 = 𝑹 [ 𝒊
]
Next, find the present value of ordinary annuity consisting of payments of ₹R each made at the end
of each of m payment intervals. The present value of this annuity is:
www.escholars.in
17
𝟏−(𝟏+𝒊)−𝒎
𝑷𝟐 = 𝑹 [ 𝒊
]
✓ Hence the present value of the deferred annuity is:
1−(1+𝑖)−𝑛 1−(1+𝑖)−𝑚
𝑷 = 𝑷𝟏 − 𝑷𝟐 = 𝑅 [ 𝑖
]−𝑅[ 𝑖
]

Q. Priority Questions
No.
1. How much interest will be earned on ₹3000 at 7% interest for 3 years, also calculate the total
amount.
Ans. S. I = Simple Interest
𝑃×𝑅×𝑇 P = Principal
S. I = 100 [ ]
R = Rate of interest
T = Time
3000×7×3
=
100
= ₹630
Amount (A) = P + S.I
= ₹(3000+630)
= ₹3630
2. ₹1000 is invested at annual rate of interest of 5%. What is the amount after two years if
compounding is done
a) Annually
b) Semi–annually
c) Quarterly
d) Monthly
Ans. a) Compounding is done a Annually.
P = ₹1000
𝑛 i = 5⁄
(1
𝐴 = 𝑃 + 𝑖) [ 100 = 0.05]
n = 2 years
= 1000 (1+0.05) 2

= ₹1102.5
b) Compounding is done semi Annually
P = ₹1000
𝑛 5
𝐴 = 𝑃(1 + 𝑖) [i = 100×2 = 0.025]
n =2×2=4
= 1000 (1+0.025)4
= ₹1103.81
c) Compounding is done quarterly.
P = ₹1000
𝑛 5
𝐴 = 𝑃(1 + 𝑖) [i = 100×4 = 0.0125]
n =2×4=8
A = 1000 (1+0.0125)8
= ₹1104.49
d) Compounding is done Monthly.
P = ₹1000
𝑛 5
𝐴 = 𝑃(1 + 𝑖) [i = 100×12 = 0.004]
n = 2 × 12 = 24
= 1000 (1+0.004)24

www.escholars.in
18
= ₹1100.55
3. ₹4000 is invested in Term Deposit Scheme that fetches interest 5% per annum compounded
quarterly. What will be the interest after 1 year. What is effective rate of interest.
Ans. C.I = 𝑃[(1 + 𝑖)𝑛 – 1]
P = ₹4000
5 4 5
= 4000 [(1 + 400) – 1] [i = 4×100 = 0.0125]
n=1×4=4
= ₹203.78
For effective rate of interest
E = [(1 + 𝑖)𝑛 – 1] × 100
5 4
= [(1 + 100×4) – 1] × 100
= E = 5.09%
4. Which is a better investment 2% per year compounded monthly or 2.2% per year simple
interest.
Ans. i =
2
= 0.0017
E (effective rate of interest) = [(1 + 𝑖)𝑛 – 1] × 100 [ 100×12 ]
n = 12
= [(1 + 0.0017)12 – 1] × 100
E = 2.06%
Effective rate of interest is less than 2.2% Hence, Simple interest 2.2% per year is better
investment.
5. Suppose Mr. Sachin invested ₹5000 in a two year investment that pays him 10% per annum.
Calculate the future value of investment.
Ans. F.V = PV (1 + 𝑖)𝑛
10 2
F.V = 5000 (1 + )
100
F.V = ₹6050
6. ₹300 is invested at the end of each month in an account paying interest 5% per year
compounded monthly. What is the future value of this annuity after 11th payment.
Ans. Note * The question is based on regular Annuity

R(Annuity) = ₹300
(1+𝑖)𝑛 –1 5
FV = R[ 𝑖
] [i = = 0.004 ]
12×100
n = 11
(1+0.004)11 –1
FV = 300 [ 0.004 ]
FV = ₹3366.80 (approx)
7. Sachin invests ₹12,000 every year starting from today for next 8 years. Suppose interest rate
is 10% per annum compounded annually. Calculate future value of annuity.
Ans. Note * Given question is based on Annuity due.

R = ₹12,000
(1+𝑖)𝑛 –1 10
FV = 𝑅 [ ] [1 + 𝑖]
[i = = 0.1]
𝑖 100
n=8
(1+0.1)8 –1
FV = 12,000 [ 0.1 ] [1 + 0.1] = ₹150953.73
8. ₹3000 is paid every year for ten years to pay off a loan. What is the loan amount if interest rate
be 12% per annum compounded annually.
Ans. Note * Given question is based on Regular annuity.

www.escholars.in
19
R = 3000
1–(1+𝑖)–𝑛 12
P. V = 𝑅 [ ] [i = 100 = 0.12]
𝑖
n = 10
1–(1+0.12)–10
= 3000 [ 0.12
]
P.V. = ₹16950.67 (approx)
Hence, the loan amount is ₹16950.67
9. Suppose your parents decide to gift you ₹5000 every year starting from today for the next 10
years. You deposit this amount in a bank as and when you receive and get 5% per annum
interest rate compounded annually. What is the present value of this annuity.
Ans. Note * Given question is based on Annuity due.
𝑅 = ₹5000
1–(1+𝑖)–(𝑛–1) 5
PV = 𝑅 [ ]+𝑅 [𝑖 = = 0.05]
𝑖 100
𝑛 = 10
1–(1+0.05)–(10–1)
= 5000 [ 0.05
] + 5000
PV = ₹40539.108
10. How much amount is required to be invested every year so as to accumulate ₹2,00,000 at the
end of 12 years if interest is compounded annually at 8%?
Ans. Note * Given question is based on sinking fund.
FV = ₹2,00,000
(1+𝑖)𝑛 –1 80
FV = 𝑅 [ ] [i= = 0.08 ]
𝑖 100
n = 12
(1+0.08)12 –1
2,00,000 = 𝑅 [ 0.08 ]
∴ R = ₹10539
11. XYZ Ltd. wants to lease out an asset costing ₹4,00,000 for a four year period. It has fixed a rental
of ₹1,45,000 per annum payable annually starting from the end of the first year. Suppose rate
of interest is 14% per annum compounded annually on which money can be invested by the
company. Is this agreement favourable to the company.
Ans. Note * Given question is based on Leasing, we have to find present value of annuity in this
question.
–𝑛
(1–(1+𝑖))
PV = 𝑅 [ 𝑖
]
14 –4
1–(1+ )
PV = 1,45,000 [ 14 100 ]
⁄100
R = ₹1,45,000
1–(1+0.14)–4 14
PV = 1,45,000 [ 0.14
] [i = = 0.14]
100
n=4
PV = ₹422488
Which is greater than the initial cost of the asset and consequently leasing is favourable to the
lesson.
12. Raja wants to retire and receive ₹4000 a month. He wants to pass this monthly payment to
future generations after his death. He can earn an interest of 6% compounded annually. How
much will he need to set aside to achieve his perpetuity goal? Assuming that the discount rate
is 8% per annum, how much would you pay to receive ₹ 50 growing at 6 % annually forever?
Ans. Note* Given question is based on perpetuity

𝑅 R = 4000
P.V = 𝑖
[i = 6
= 0.005]
12×100

www.escholars.in
20
4000
=
0.005
P.V = ₹8,00,000
𝑅 50
P.V = = = ₹ 2500
𝑖−𝑔 0.08−0.06
( i = interest rate or discount rate, g = growth rate in interest
13. The CAGR of a initial value of a investment of ₹15,000 and find value of ₹25,000 in 3 years is.
Ans.
Note * Given question is based on compound annual growth rate.

Initial value (Vto) = 15000


Final value (Vtn) = 25,000
1
𝑉 𝑡 –𝑡𝑜
CAGR (0,3) = [(𝑉𝑡𝑛 ) 𝑛 – 1] × 100
𝑡𝑜

1⁄
25000 3
= [( ) – 1] × 100
15000

= [1.1856–1] × 100
= 0.1856 × 100 = 18.56
14. If the cost of capital be 12% per annual, then the net present value (in nearest ₹) from the given
cash flow is given as:
Years 0 1 2 3
Operating profit (in thousands ₹) (100) 60 40 50
Ans. Net Present Values = Present Value of Inflows – Present value of Outflows
60,000 40,000 50,000
Present Value of Inflows = + (1.12)2 + (1.12)3 = 1,21,048
(1.12)
Present Value of Outflows = 1,00,000
Therefore, Net Present Value = 1,21,048 – 1,00,000 = ₹21,048
Since the nearest option is (d), We’ll mark (d).
15. An investor intends purchasing a three year ₹1,000 par value bond having nominal interest
rate of 10%. At what price the bond may be purchased now if it matures at par and the investor
requires a rate of return of 14%?
Ans. Present value of the bond
100 100 100 1,000
= (1+0.14)1 + (1+0.14)2 + (1+0.14)3 + (1+0.14)3
= 100 × 0.87719 + 100 × 0.769467 + 100 × 0.674972 + 1,000 × 0.674972
= 87.719 + 76.947 + 67.497 + 674.972
= 907.125
Thus the purchase value of the bond is ₹907.125

www.escholars.in
21
Permutation and Combination
✓ Combinations: The number of ways in which smaller or equal number of things are arranged or
selected from a collection of things where the order of selection or Arrangement is not Important,
are called combination.
Number of combinations of n different things taken r at a time.
Denoted by nCr or C (𝑛, 𝑟)
𝒏 = Total Items
𝒓 = Items to be Selected
nC =
𝑛!
r
(𝑛−𝑟)!𝑟!
✓ Permutation: The ways of arranging or Selecting Smaller or equal number of persons or collection
of objects with due regard being paid to the order of arrangement or selection, are called
permutation.
Number of Permutation when r objects are chosen out of n different objects. Denoted by nPr or (𝑛, 𝑟)
𝒏 = 𝑇𝑜𝑡𝑎𝑙 𝐼𝑡𝑒𝑚𝑠
𝒓 = 𝐼𝑡𝑒𝑚𝑠 𝑡𝑜 𝑏𝑒 𝑆𝑒𝑙𝑒𝑐𝑡𝑒𝑑 𝑎𝑛𝑑 𝑎𝑟𝑟𝑎𝑛𝑔𝑒𝑑
nPr=
𝑛!
(𝑛−𝑟)!
✓ Factorial:
The factorial n, written as 𝑛! Or ⌊𝑛, represents the Product of all Integers from 1 to n both inclusive.
To make the notations meaningful. When n = 0, the desire 0! The define 0! or ⌊0= 1
𝑇ℎ𝑢𝑠, 𝑛! = 𝑛 (𝑛 − 1)(𝑛 − 2) … … … … .3.2.1

✓ Circular Permutation: The number of Circular permutations of n different things chosen at a time
is ((𝑛 − 1)!)
➢ The number of ways of arrangement n persons along a round table so that no person has the
1
same two neighbors is (𝑛 − 1)!
2
➢ In Forming a necklace or a garland there is no distinction between a clockwise and anti-
clockwise direction, because we can simply turn it over. So that Clockwise become
1
anticlockwise and vice-versa. So required number of arrangements is (𝑛 − 1)!
2
✓ Restricted Permutation:
Number of Permutations of n distinct objects taken r at a time. When a particular object is not taken
in any arrangement is 𝑛 − 1𝑃𝑟
n-1Pr
(𝑛−1)!
= (𝑛−1−𝑟)!
Number of Permutations of r objects out of n distinct objects when a Particular object is always
included in any arrangement is r.
(n-1)P(r – 1).

(n-1)P(r – 1) =
(𝑛−1)! (𝑛−1)!
𝑛−1−𝑟+1
= (𝑛−𝑟)!
✓ Properties of nCr
➢ nCr = nCn-r
➢ n+1Cr = nCr + nCr-1
𝑛! 𝑛!
➢ nC0 = (𝑛−0)! = = 1
0! 𝑛!
nCn =
𝑛! 𝑛!
➢ (𝑛−𝑛)!𝑛!
= = 1 [∴ 0! = 1]
𝑛! 0!
✓ Permutation when some of the things are alike, taken all at a time
𝑛!
𝑷=
𝑛1 ! 𝑛2 ! 𝑛3 !
✓ The number of permutations of n things taken ‘r’ at time. When each thing may be repeated r items
in any arrangement is 𝑛𝑟 .
✓ Combination of n different things taking some or all n things at a time.
www.escholars.in
22
(2𝑛 − 1)
In symbol =∑𝑛𝑟=1 𝑛𝑐𝑟 ⟹ 2𝑛 − 1
✓ Shortcuts
➢ Arrangement of n items when Particular 2 items never Come together.
= (𝑛 − 1)! (𝑛 − 2)
➢ Number of Straight lines from n points of Which ‘m’ are Collinear.
= nC2 − mC2 + 1
Number of Triangle from n points of which ‘m’ are Collinear.
= nC3 − mC3
➢ Number of diagonals to be formed from n Points
𝑛(𝑛−3)
= 2
➢ If a family of m || lines are Intersected by family of n || lines then number of parallelograms
𝑚.𝑛 (𝑚−1) (𝑛−1)
will be =
4

Q. Priority Questions
No.
1. Find
i) 6!
8!
ii)
5!
1 𝑥
iii) =
91 10!
Ans. i) 6! or ⌊6 = 6 × 5 × 4 × 3 × 2 × 1 = 720
8! ⌊8 8×7×6×5!
ii) or ⌊5 = = 336
5! 5!
1 𝑥 1 𝑥
iii) 9! = 10! = 9! = 10×9! ⇒ 𝑥 = 10
(Note Factorial (!) can also be denoted as L)
2. Evaluate;
i) 6P4
ii) 7P3 – 6P3
Ans. i) 6P4 = 6! = 6! = 6×5×4×3×2! = 360 [npr = 𝑛! ]
(6−4)! 2! 2! (𝑛−𝑟)!
7! 6! 7×6×5×4! 6×5×4×3!
ii) 7P3 – 6P3 = (7−3)! – (6−3)! = 4!
- 3!
= 210 – 120 = 90

[Note* np1 = npn– n!, np0= 1]


3. How many different words can be formed with the letters of the word LIBERTY. In how many
ways can a family consist of three children having different birthdays in a leap year.
Ans. i) Number of different words that can be formed from the letters of ‘LIBERTY’ = 7p7 = 7!
= 7×6×5×4×3×2×1 = 5040
Note* 0! = 1

ii) Number of ways can a family consist of three children have different birthday in a leap year =
366 × 365 × 364
Note* Leap year = 366 days; Since children have different birthday
Number of ways, the birthday of list child can be any day out of 366 days
Number of ways, the birthday of 2nd child can be (366-1) = 365 days
Number of ways the birthday of 3rd child can be (365-1) = 364 days

www.escholars.in
23
4. The four digit numbers that can be formed out of the seven digits 1,2,3,5,7,8,9 such that no digit
is repeated in any number and are greater than 3000 are

www.escholars.in
24
Ans. Note* Question is based on permutation with restrictions

1, 2, 3, 5, 7, 8, 9

5 6 5 4
The 4 digit numbers should be greater than 3000 without repetition of digits.
1st Place can be filled in 5 ways
2nd Place can be filled in 6 ways
3rd Place can be filled in 5 ways
4th Place can be filled in 4 ways
Total number of ways = 5×6×5×4 = 600 ways
5. There are 5 books on English, 4 books on Tamil and 3 books on Hindi. In how many ways can
these books be placed on a shelf if the books on the same subjects are to be together.
Ans. Note * Question is based on some particular items to come together.

Number of English book = 5


Number of Tamil book = 4
Number of Hindi book = 3

E1E2E3E4E5 T1T2T3T4 H1H2H3


1 2 3
Number of ways 5! × 4! × 3! × 3! = 1,03680.
(Books can be arranged among itself)
6. The number of words which can be formed by the letters of the word ‘ALLAHABAD’.
Ans. Note * Question is based on similar items occurs multiple times.

ALLAHABAD
𝑛!
Number of ways =
𝑝!𝑞!
Total numbers of letters (n) = 9
Numbers of Letter ‘A’ (P) = 4
Numbers of Letter ‘L’ (q) = 2
9! 9×8×7×6×5×4!
Number of words = 4!2! = 4!2!
= 7560
7. Three girls and five boys are to be seated in a row so that no two girls sit together. Total number
of ways of this arrangement are.
Ans. Note * Question is based on some particular items to never come together.

The boys can be arranged in 5! Ways.


_______ B1 _________ B2 _________ B3 __________ B4 ___________ B5
The three girls can be arranged in empty spaces shown above. There are 6 empty spaces and 3
girls.
So, 3 spaces are to be selected out of these 6, and then the three girls are to be arranged in as
many ways as possible i.e.
6c3
6×5×4 𝑛!
× 3! = 1×2×3 × 3 × 2 × 1 = 120 ways (ncr = 𝑛!(𝑛–𝑛)!)
So, total number of ways = 5! × 120 = 14,400.

www.escholars.in
25
8. The number of parallelogram, formed from a set of Six parallel lines intersecting another set of
four parallel lines is
Ans. Note: Question is based on Geometry (parallelogram) Always use combination in this case.
Number of Parallelogram are formed from a set of m parallel lines intersecting another set of n
parallel lines.
= mc2 × nc2
Here m = 6 n = 4
6c × 4c
2 2
6×5 4×3
2×1
× 2×1 = 15×6
=90
9. The number of ways 5 boys and 5 girls can be seated at a round table, so that no two boys are
adjacent is
Ans. Note * For circular arrangements, use formula (n–1)! since one place to be fixed
B1

G1 G2

B5 B2

G3
G5

B4 B3
G4
First, let the girls sit, so there are five girls to be arranged at a round table.
Number of ways = (5–1)! = 4! = 24
Now the remaining 5 places can be filled by boys in 5! ways = 120
∴ Total number of ways = 24 × 120 = 2,880
10. How many ways a team of 11 players can be made out of 15 players if one particular player is
not to be selected in the team.
Ans. Note * Question is based on things to never always occur
If one particular player is not selected
14×13×12
Number of ways = 14c11 = = 364
3×2×1
11. The total number of ways in which six ‘+’ and four ‘–‘signs can be arranged in a line such that no
two ‘–‘ sign occur together is.
Ans. Note * Question is based on flipping of identical objects

++++++
Six ‘+’ sign can be selected in 6c6 ways.
Four ‘–‘sign can be selected in 7c4 ways.
∴ Total ways 6c6 × 7c4 = 1 × 35 = 35
12. The number of ways in which 8 different beads be strong on a necklace is

www.escholars.in
26
Ans. (Note * For question based on necklace use
(𝑛–1)!
, Since, necklace will be as it is in case of
2
clockwise and anticlockwise)
(8–1)! 7! 7×6×5×4×3×2×1
2
= 2= 2
= 2520
13. The sum of all 4 digit number containing the digits 2, 4, 6, 8, without repetition is.
Ans. Note * Question based on sum of the numbers formed from digits. Use → (sum of digits) (n–
1)! × 1111.

If digits are identical divide by such number of factorial.


Sum of digits (n–1)! × 1111 [n = place]
(2+4+6+8) (4–1)! × 1111
20 × 3 × 2 × 1111 = 133320
14. What will be the rank or order of the word ‘ZENITH’ in the dictionary.
Ans. Note * Question based on ranking of a word in dictionary.
i) Give the alphabet numerical order as per ascending order
ii) Check for the smaller number in rightward direction from the digits given to the alphabet
iii) From rightward to leftward start giving 0! , 1! Etc. in increasing order
iv) Now multiply the steps followed in step ii) and step iii) and add them
v) Finally add 1 to the total to get the rank of the word in the dictionary.
Z E N I T H
6 1 4 3 5 2

5 0 2 1 1 0

5! 4! 3! 2! 1! 0!
600 + 0 + 12 + 2 + 1 + 0 = 615
+ 1
616
15. Find the number of ways of selecting 4 letters from the word ‘EXAMINATION’.
Ans. There are 11 letters in the word of which A, I, N are repeated twice.
Thus we have 11 letters of 8 different kinds (A, A), (I, I), (N, N), E, X, M, T, O.
The group of four selected letters may take any of the following forms:
i) Two alike and other two alike
ii) Two alike and other two different
iii) All four different
In case i), the number of ways = 3C2 = 3.
In case ii), the number of ways = 3C1 × 7C2 = 3 × 21 = 63.
8×7×6×5
In case iii), the number of ways = 8C4 = 1 × 2 × 3 × 4 = 70
Hence, the required number of ways = 3 + 63 + 70 = 136 ways

www.escholars.in
27
Sequence and Series
✓ An expression of the form 𝑎1 + 𝑎2 + 𝑎3 + _ _ _ _ _ _ 𝑎𝑛 . Which is the sum of the elements of the
sequence (𝑎𝑛 ) is called a series. If the series contains a finite number of elements, it is called finite
series, otherwise, called an Infinite series.
✓ Arithmetic Progression (A.P.): A sequence 𝑎1 , 𝑎2 , 𝑎3 , _ _ _ _ _ _ 𝑎𝑛 is called an Arithmetic Progression
(A.P.). When 𝑎3 − 𝑎2 = 𝑎2 − 𝑎1 = 𝑎𝑛 − 𝑎𝑛−1 . That Means A.P. is a sequence in which each term is
obtained by adding a constant to the preceding term. This constant ‘d’ is called the common
difference of the A.P. If 3 numbers a, b, c is in A.P.
Then,
b – a = c – b or a + c = 2b. b is called the arithmetic mean between a and c.
a = First Term, d = Common difference
✓ Formula to find the nth term = 𝒂𝒏 = 𝑎 + (𝑛 − 1)𝑑
𝑛(𝑛+1)
✓ Sum of first n natural number =
2
𝑛(𝑛+1)(2𝑛+1)
✓ Sum of square of first n natural number is = 6
𝑛(𝑛+1) 2
✓ Sum of cubes of first, n natural number = [ ]
2
✓ Sum of the first n odd numbers = 𝑆 = 𝑛 2
𝑛 𝑛
✓ Formula to find the sum of n natural numbers = (𝑎 + 𝑙) or [2𝑎 + [𝑛 − 1]𝑑]
2 2
✓ Sum of the first n odd numbers = 𝑆 = 𝑛2
✓ Shortcut
➢ If Pth of an A.P. is given as 𝑞. 𝑞th term of an A.P. is given as P. Then 𝑟th term of A.P. will be:
𝒂𝒑 = 𝑞 , 𝑎𝑞 = 𝑝
𝒂𝒓 = 𝑝 + 𝑞 − 𝑟
➢ If Pth term of an A.P. is given as q. 𝑞th term of an A.P. is given as P. then (𝑝+𝑞)th term will be:-
𝒂𝒑 = 𝑞, 𝑎𝑞 = 𝑝, 𝑇ℎ𝑒 𝑎𝑝+𝑞 = 0
➢ If sum of P terms of an A.P. is given as 𝑞. The sum of 𝑞 terms of an A.P. is P. then sum of (𝑝+𝑞)th
terms will be: -
𝑺𝑷 = 𝑞, 𝑆𝑞 = 𝑝
𝑺𝒑+𝒒 = −(𝑝 + 𝑞)
1 1
➢ If pth term of an A.P. is given as 𝑞 . qth term of an A.P. is 𝑝 . the sum of (𝑝+𝑞)th terms will be:
1 1 1
𝒂𝒑 = , 𝒂𝒒 = , = 𝑺𝒑𝒒 = (𝑝𝑞 + 1)
𝑞 𝑝 2
➢ If Ratio of sum of n terms of two series is given in the question, then Ratio of 𝑛𝑡ℎ term of two
series will be → Replace ‘n’ from ‘2n-1’
➢ If Ratio of nth term of two A.P. Series is given then ratio of sum of ‘n’ terms of A.P. Series will
n+1
be → Replace n from [ 2 ] .
✓ Geometric Progression:
If in a sequence of terms each term is constant multiple of the preceding term, then the sequence is
called a Geometric Progression (G.P.). The constant multiplier is called common Ratio.
𝑎𝑛𝑦 𝑡𝑒𝑟𝑚 𝑡𝑛
⇒𝒓= =
𝑃𝑟𝑒𝑐𝑒𝑒𝑑𝑖𝑛𝑔 𝑡𝑒𝑟𝑚 𝑡𝑛−1
𝑎2 𝑎3
𝐹𝑜𝑟 𝑬𝒙𝒂𝒎𝒑𝒍𝒆 ⟹ 𝒓 = 𝑎 = 𝑎
1 2
𝒓 = 𝐶𝑜𝑚𝑚𝑜𝑛 𝑅𝑎𝑡𝑖𝑜
✓ Formula to find the nth term of G.P. = 𝒂𝒓𝒏−𝟏
a = First term, r = Common Ratio
n = Number of terms.
✓ Formula to find the sum of n terms of a G.P.
𝑎(1−𝑟 𝑛 )
𝑺𝒏 = 1−𝑟 𝑊ℎ𝑒𝑛 𝑟 < 1
www.escholars.in
28
𝑎(𝑟 𝑛 −1)
𝑺𝒏 = 𝑟−1 𝑊ℎ𝑒𝑛 𝑟 > 1
✓ Formula to find the sum of G.P of an Infinite series.
𝑎
𝑺∞ = 1−𝑟 , 𝑟 < 1
(𝑎+𝑏)
✓ A.M. of a and b is = 2
.
𝑏 𝑐
✓ If a, b, c is in G.P. We get 𝑎 = 𝑏 ⇒ b2 = ac, b is called the Geometric mean between a and c.

Q. Priority Questions
No.
1. Find the sum to infinity of the following series:
1 – 1 + 1 – 1 – 1 + ………… ∞
Ans. a=1
r = –1
𝒂 1 1
Sum of the G.P. up to infinity = = =
𝟏–𝒓 1–(–1) 1+1
1
Sum =
2
2. Insert two, Arithmetic means between 68 and 260.
Ans. Let A1 A2, be the two Arithmetic Means between 68 and 260
∵ 68, A1, A2, 260 are in A.P
Here.
First Term (a) = 68
Last Term (I) = 260
Total no. of terms = 4
we know,
l (last term) = a + (n–1) d
260 = 68 + (4–1) d
∵ d = 64
∵ A1 = a + d = 68 + 64 = 132
and A2 = a + 2d = 68 + 2 × 64 = 196
3. If each month ₹100 increases in any sum then find out the total sum after 10 months, if the sum
of first month is ₹2,000.
Ans. First term (a) = 2,000
Common diff (d) = 100
(n) = 10
Sn = ?
𝒏
Sn = [2a+(n–1)d]
𝟐
10
= 2
[2 × 2,000 + (10– 1)100]
= 5[4,000+900]
= 5 × 4,900
= ₹24,500
4. If arithmetic mean between roots of a quadratic equation is 8 and the geometric mean between
them is 5, the equation is ____________.
Ans. Say, 𝛼, 𝛽 are the roots of quadratic equation them as per given conditions, we have
𝜶+𝜷
A.M. ⇒ 𝟐
=8

www.escholars.in
29
⇒ 𝛼 + 𝛽 = 16 (i.e. Sum of roots)
And given
G.M. ⇒ √𝜶𝜷 = 5
⇒ 𝛼. 𝛽 = 25 (i.e. Product of roots)
∴ required quadratic equation is:
x2 – (Sum of roots) x + (Product of roots) = 0
⇒ x2 – 16x + 25 = 0
5. If x, y, z are the terms in G.P. then the terms x2 + y2, xy + yz, y2 + z2 are in:
Ans. If x, y, z are in G.P
Then, let x = 1, y = 2, z = 4 (In G.P, common ratio should be same)
x2 + y2, xy + yz, y2 + z2
= 12 + 22, 1 × 2 + 2 × 4, 22 + 42
6. If a, b, c are in Arithmetic Progression (A.P.), then the value of a–b+c is:
Ans. If a, b, c are in A.P.
then a = 1, b = 2, c = 3 ( In A.P, common difference should be same)
a–b+c=1–2+3=4–2=2
7. The income of a person is ₹5,00,000 in the firm in the first year and he receives an increase of
₹15,000 per year for next 10 years. The total amount he receives in 10 years is:
Ans. Given a = 5,00,000, d = 15,000, n = 10
Total Amount after n years
𝒏
Sn = 𝟐 [𝟐𝒂 + (𝒏– 𝟏)𝒅]
10
S10 = [2 × 5,00,000 + (10– 1) × 15,000]
2
= 5[10,00,000 + 9 × 15,000]
= 5 [10,00,000 + 1,35,000]
= 5[11,35,000]
= 56,75,000
8. The sum of n terms of the series 1 + (1+3) + (1+3+5) + …………… is
Ans. Given Series
1 + (1+3) + (1+3+5) + …… n term
= 1 + 4 + 9 + ……… n term
= 12 + 22 + 32 + ……… + n2
= ∑ 𝑛2
𝒏(𝒏+𝟏)(𝟐𝒏+𝟏)
= 𝟔
9. The value of K, for which the terms 7K + 3, 4K – 5, 2K + 10 are in A.P., is
Ans. If 7K + 3, 4K – 5, 2K + 10 are in A.P.
Then,
(4K–5) – (7K+3) = (2K+10) – (4K–5)
4K – 5 – 7K – 3 = 2K + 10 – 4K + 5
– 3K – 8 = –2k + 15
– 8 – 15 = – 2K + 3K
– 23 = K
10. If AM and HM for two numbers are 5 and 3.2, respectively. GM will be:
Ans. We know that
(GM)2 = AM × HM (A.M represents A.P and G.M represents G.P
Here (GM)2 = 5 × 3.2
(GM)2 = 16
(GM) = 4
11. The sum of series 7 + 14 + 21 + …… to 17th term is:
Ans. Clearly, this is an AP with a = 7; d = 14–7 = 7; n = 17
www.escholars.in
30
𝒏
Sn = 𝟐 {𝟐𝒂 + (𝒏– 𝟏)𝒅}
17
S17 = 2 {(2 × 7) + (17– 1)7} = 1.071
12. The sum of first n terms an AP is 3n2 + 5n. The series is:
Ans. S1 = t1 = a = 3(1)2 + 5(1) = 3 + 5 = 8
S2 = 3 (2)2 + 5 (2) = 22
t2 = 22 – 8 = 14
d = t2 – t1 = 14 – 8 = 6
Therefore, the series is 8, 14, 20, 26, ………….
13. If the sum of five terms of AP is 75. Find the third term of the series
Ans. We know,
𝑛
Sn = [2𝑎 + (𝑛– 1)𝑑]
2
n=5 S5 = 75
5
S5 = 2 [2𝑎 + (5– 1)𝑑]
5
75 = 2 [2𝑎 + 4𝑑]
5×2
75 = [𝑎 + 2𝑑]
2
15 = a + 2d ______ Eq (1)
T3 = a + (3–1) d
T3 = a + 2d
_______ From Eq (1)
T3 = 15
14. In a G.P. If the fourth term is ‘3’ then the product of first seven terms is
Ans. In G.P. T4 = ar4–1 = 3
ar3 = 3
Product of 1st seven terms
= (a. ar . ar2 . ar3 . ar4 . ar5 . ar6)
= a7 . r21
= (ar3)7
= (3)7
15. Divide 69 into 3 parts which are in A.P. and are such that the product of first two parts is 460:
Ans. Let Three Nos. are in AP is
(a – d), a, (a + d)
their sum = 69
a – d + a + a + d = 69
3a = 69
a = 23
The product of first two parts = 460
(a–d) a = 460
(23 – d) 23 = 460
23 – d = 20 ⇒ d = 3
Nos. are
a –d, a, a + d
23 – 3, 23, 23 + 3
20, 23, 26

www.escholars.in
31
Sets, Relations and Functions
✓ Sets: collection of well-defined distinct objects.
✓ Singleton Set: A Set containing one element is called singleton set.
✓ Equal Set: Two Set A & B are said to be equal, written as A=B. If every element of A is in B and every
element of B is in A.
✓ Non-empty Set: A set has at least one element is called non-empty set. Thus, the set {0} is non-empty
set. It has one element 0.
✓ Equivalent set: Two finite sets A & B are said to be equivalent If n (A) = n(B)
All equal sets are equivalent sets. but all equivalent sets are not equal.
✓ Null Set/ Void Set/ Empty Set: Set having zero element
✓ Sub-set: If all the elements of set are present in another set, then such set is sub-set of another Set.
✓ Power Set: The collections of all possible subsets of a given set A is called the power set of A, to
denoted by P(A).
✓ Every set is a subset of itself
✓ 𝜙 is the subset of all sets.
✓ No. of subsets in set = 2𝑛 , 𝑛 = 𝑛𝑜. 𝑜𝑓 𝑒𝑙𝑒𝑚𝑒𝑛𝑡𝑠
✓ Proper subset: All subsets excluding same or identical one is called proper sub-set.
Proper subset = 2𝑛 − 1
✓ If A and B are two sets then
n(A∪B) = n (A) + n(B) − n(A∩B)
✓ If A and B are disjoint sets, then n(A∪B) = n(A) + n(B), as n (A∩B) = 0
✓ For three Sets P, Q and R
n(P∪Q∪R) = n(P) + n(Q) + n(R) − n(P∩Q) − n(P∩R) − n(Q∩R) + n(P∩Q∩R)
When P, Q and R are disjoint sets
n(P∪Q∪R) =n(P) + n(Q) + n(R)
✓ Cartesian Product of sets: If A and B are two non-empty sets then the set of all ordered pair (a, b)
Such that a belongs to A and b belongs to B, is called the cartesian product of A and B, to be donated
by A × B.
Thus A × B = {(a, b): a ∈ A and b ∈ B}
If A = 𝜙 or B = 𝜙 ⟹ we define A × B = 𝜙
✓ Relations and function: Any Subset of the product set X.Y is said to define a relation from X to Y in
which no two different ordered pairs have the same first element is called a function.
Let A and B be two non-empty sets. Then, a rule or correspondence f which associates to each
element X of A, a unique element, denoted by f (𝑥) of B is called a function or mapping from A to B
and we write F: A→B
✓ The element 𝑓(𝑥) of B is called the Image of 𝑥, while 𝑥 is called the Pre-image of f (𝑥)
Let f: A→B, then A is called the domain of F, while B is called Co-domain of ‘f’.
✓ One- one Function: Let f: A→B. If different elements in A have different images in B, then f is said to
be one-one or an Injective or mapping.
✓ Onto or surjective function: Let f: A→B If every element in B has atleast one pre-image in A, then f
is said to be an onto function.
✓ Bijection function: A one-one and onto function is said to be bijective. Bijective function is also
known as one-to-one correspondence.
✓ Identity function: Let A be a non-empty Set. Then, the function I defined by
I : A→A: I (𝑥) for all X ∈ A is called an Identity function on A.
✓ Inverse function: - Let f be a one-one onto function from A to B. let y be an arbitrary element of B.
Thus, f being onto, there exists an element 𝑥 in A Such that f (𝑥) = y.
✓ Different types of relations:
Let S= {a, b, c, d_ _ _ _} be any set then the relation R is subset of the product S×S,
If R contains all orders pain of the form (a, a) in S×S, then R is called Reflexive, In, a reflexive relation
‘a’ is related to itself.
➢ If (a, b)∈R ⇒ (b, a) ∈R for every a, b∈s then R is called Symmetric.
www.escholars.in
32
➢ If (a, b) ∈R and (b, c) ∈ R ⇒ (a, c) ⇒R for every a, b, c ∈ S then R is called Transitive.
✓ A relation which is reflexive, Symmetric and transitive is called equivalence relation.
✓ Domain & Range of a relation: If R is a relation from A to B then set of all first Co-ordinates of
elements of R is called domain of R, while the set of all second co-ordinates of elements of R is called
the Range of R.

Q. No. Priority Questions


1. Given the function f(x) = (2x + 3), then the value of f(2x) – 2f(x) + 3 will be:
Ans. f(x) = 2x + 3
f(2x) – 2f(x) + 3
= [2(2x) + 3] – [2(2x + 3)] + 3
= 4x + 3 – 4x – 6 + 3
= 4x – 4x + 6 – 6.
= 0.
2. In the set N of all natural numbers the relation R defined by a R b “if and only if, a divide be”,
then the relation R is:
Ans. For a function to be a partial order Relation, it should be
1) Reflexive
2) Antisymmetric and
3) Transitive
a divides b satisfies the above 3 relations as follows:
1) a/a Reflexive
2) a/b and b/a a = b Antisymmetric
3) a/b, b/c a/c Transitive
a/b is not a symmetric function and hence, not an equivalence relation.
3. There are 40 students, 30 of them passed in English, 25 of them passed in Math’s and 15 of
them passed in both. Assuming that every Student has passed at least in one subject. How
many students passed in English only but not in Math’s.
Ans. Given
total No. of Students n(E  N) = 40
No. of students passed in Eng. n(E) = 30
No. of students passed in Math’s n(M) = 25
No. of students passed in both n(EM) = 15
Therefore, required to Find: n(only E) = ?

E M
15 15 10

n(only E) = n(E) – n(E  M)


= 30 – 15 = 15
𝑥 𝑥
4. If f(x) = and g(x) = Find fog?
√1+𝑥 2 √1−𝑥 2
𝑥 𝑥
Ans. Given: f(x) = and g(x) =
√1+𝑥 2 √1−𝑥 2
fog(x) = f{g(x)}
𝑥
=f| |
√1−𝑥 2
𝑥
√1−𝑥2
= =x
𝑥2
√1+( )
√1+𝑥2

5. If A = (1,2,3,4,5), B = (2,4) and

www.escholars.in
33
C = (1,3,5) then (A-C) × B is
Ans. (A-C) = {1,2,3,4,5} – {1,3,5} = {2, 4}
(A-C) × B = {2, 4} × {2, 4} = {(2, 2), (2, 4), (4, 2), (4, 4)
6. The number of proper sub set of the set {3, 4, 5, 6, 7} is
Ans. Given set A = {3, 4, 5, 6, 7}
Cardinal No n(A) = 5
Number of proper subset = 2n – 1
= 25 – 1
= 32 – 1 = 31
7. The number of elements in range of constant function is
Ans. The range set of a constant function is a singleton set. Therefore, the number of elements in
the range set of a consistent function is one.
8. If f(x) = 2x + 2 and g(x) = x2, then the value of fog (4) is:
Ans. f(x) = 2x + 2 and g(x) = x2
fog(x) = f{g(x)}
= f{x2}
= 2x2 + 2
then
fog(4) = 2(4)2 + 2
= 2 × 16 + 2
= 32 + 2 = 34
9. If f(x) =
𝑥 2 −25
then f(5) is
𝑥−5
Ans. If f(x) =
𝑥 2 −25
𝑥−5
(5)2 −25 0 = does not exist
f(5) = =0
5−5 = not defined
10. If R is the set of all real numbers, then the function f: R – R defined by f(x) = 2x
Ans. f(x) = 2x
f(𝑥1 ) = 2𝑥1 and f(x2) = 2𝑥2
Now, f(x1) = f(x2)
2𝑥1 = 2𝑥2 ⇒ x1 = x2
so, f(x) = 2x is one-one
and
f(x) = 2x
y = 2x
log y = log 2x
log y = x log 2
x = log2y [log is not valid value if y is negative]
So, range of function ≠ B so it is into function.
11. The number of subsets of the set {3, 4, 5) is:
Ans. Here, A = {3,4,5}
n(A) = 3
No of Subset = 2n
= 23
=8
12. Two finite sets respectively have x and y number of elements. The total number of subsets of
the first is 56 more than the total number of subsets of the second. The value of x and y
respectively.
Ans. Let A and B are two set
Given n(A) = x and n(B) = y
No. of subset of A = 2x and No. of subset of B = 2Y
According the question

www.escholars.in
34
2X = 2Y + 56 ………….1
Option (a) is satisfied eq. (1) so
x = 6, y = 3
13. Let U be the universal set, A and B are the subsets of U. If n(U) = 650,
n(A) = 310
n(A  B) = 95 and n(B) = 190, then n(A ̅B ̅) is equal to (A
̅ and B
̅ are the complement of A
and B respectively):
Ans. n(A  B) = n(A U B)’ = n(U) – n(A  B)
n(A  B) = n(A) + n(B) – n(A  B)
n(A  B) = 310 + 190 – 95 = 405
n(A  B) = n(U) – n(A  B) = 650 – 405 = 245
14. Let A = {1,2,3} and consider the relation R = {(1,1), (2,2), (3,3), (1,2), (2,3), (1,3)}.
Then R is:
Ans. Let A = {1,2,3}
R = {(1,1), (2,2), (3,3), (1,2), (2,3), (1,3)}
then R is reflexive but not symmetric.
15. Given A = {2,3}, B = {4,5}, C = {5,6} then A × (B  C) is:
Ans. A = {2,3}, B = {4,5}, C = {5,6}
B  C = {5}
A × (B  C) = {2, 3} × {5}
= {(2,5), (3,5)}

www.escholars.in
35
Differentiation and Integration
✓ Basic formulas of differentiation:
𝐝
➢ (𝒙𝐧 ) = n𝑥 𝑛−1
𝐝𝒙
𝒅
➢ (𝒄𝒐𝒏𝒔𝒕𝒂𝒏𝒕) = 0
𝒅𝒙
𝒅
➢ (𝒆𝒙 ) = 𝑒 𝑥
𝒅𝒙
𝒅
➢ (𝒂𝒙 ) = 𝑎 𝑥 𝑙𝑜𝑔𝑒 𝑎
𝒅𝒙
𝒅
➢ (𝒆𝒂 𝒙 ) = 𝑎. 𝑒 𝑎𝑥
𝒅𝒙
𝒅 1
➢ 𝒅𝒙
(𝐥𝐨𝐠 𝒙) = 𝑥
𝒅 𝑑 𝑑
➢ 𝒅𝒙
(𝒖𝒗) =
𝑑𝑥
(𝑢). 𝑣 +
𝑑𝑥
(𝑣). 𝑢 (Product Rule)
𝑑 𝑑
𝒅 𝒖 𝑣. (𝑢)−𝑢 . 𝑣
➢ ( )= 𝑑𝑥 𝑑𝑥
(Quotient Rule)
𝒅𝒙 𝒗 𝑣2

✓ Application Related from cost function:


➢ TC = TVC + TFC
𝑇𝐶 𝑇𝑉𝐶 𝑇𝐹𝐶
➢ AC = 𝑂𝑢𝑡𝑝𝑢𝑡 , 𝑨𝑽𝑪 = 𝑜𝑢𝑡𝑝𝑢𝑡 , 𝑨𝑭𝑪 = 𝑜𝑢𝑡𝑝𝑢𝑡
𝑑
➢ MC = 𝑑𝑥 (𝑇𝐶)
𝑑
➢ MC = 𝑑𝑥 (𝑇𝑉𝐶)

✓ Basic Formulas of Integration:


𝑥 𝑛+1
➢ ∫ 𝒙𝒏 𝒅𝒙 = + 𝐶 , 𝑛 ≠ −1
𝑛+1

➢ ∫ 𝒅𝒙 = 𝑥 + 𝑐
➢ ∫ 𝒆𝒙 𝒅𝒙 = 𝑒 𝑥 + 𝐶
𝑒𝑎 𝑥
➢ ∫ 𝒆𝒂𝒙 𝒅𝒙 = +𝐶
𝑎
𝒅𝒙
➢ ∫ 𝒙
= 𝑙𝑜𝑔𝑥 + 𝐶
𝑎𝑥
➢ ∫ 𝒂𝒙 𝒅𝒙 = +𝐶
𝑙𝑜𝑔 𝑒𝑎

✓ Some Important formulas of Integration:


𝒅𝒙 1 𝑥−𝑎
➢ ∫ 𝒙𝟐 −𝒂𝟐 = 2𝑎
log 𝑥+𝑎 + 𝐶
𝒅𝒙 1 𝑎+𝑥
➢ ∫ 𝟐 𝟐= log +𝐶
𝒂 −𝒙 2𝑎 𝑎−𝑥
𝒅𝒙
➢ ∫ = log |𝑥 + √𝑥 2 + 𝑎2 | + 𝐶
√𝒙𝟐 +𝒂𝟐
𝒅𝒙
➢ ∫ = log |𝑥 + √𝑥 2 − 𝑎2 | + 𝐶
√ 𝒙𝟐 −𝒂𝟐

➢ ∫ 𝒆𝒙 [𝒇(𝒙) + 𝒇′(𝒙)] 𝒅𝒙 = 𝑒 𝑥 𝑓(𝑥) + 𝐶

www.escholars.in
36
𝑥 𝑎2
➢ ∫ √𝒙𝟐 + 𝒂𝟐 𝒅𝒙 = 2 √𝑥 2 + 𝑎2 + 2
log |𝑥 + √𝑥 2 + 𝑎2 | + 𝐶
𝑥 𝑎2
➢ ∫ √𝒙𝟐 − 𝒂𝟐 𝒅𝒙 = 2 √𝑥 2 − 𝑎2 − 2
log |𝑥 + √𝑥 2 − 𝑎2 | + 𝐶
𝒇′ (𝒙)
➢ ∫ 𝒇(𝒙)
𝒅𝒙 = log 𝑓(𝑥) + 𝐶
𝑑
✓ Integration by parts ∫ 𝒖𝒗 𝒅𝒙 = 𝑢 ∫ 𝑣 𝑑𝑥 − ∫[ (𝑢) ∫ 𝑣𝑑𝑥] 𝑑𝑥
𝑑𝑥

✓ Important Properties of definite Integrals


𝒃 𝑏
➢ ∫𝒂 𝒇(𝒙)𝒅𝒙 = ∫𝑎 𝑓(𝑡)𝑑𝑡
𝒃 𝑎
➢ ∫𝒂 𝒇(𝒙)𝒅𝒙 = − ∫𝑏 𝑓(𝑥)𝑑𝑥
𝒃 𝑐 𝑏
➢ ∫𝒂 𝒇(𝒙) 𝒅𝒙 = ∫𝑎 𝑓(𝑥)𝑑𝑥 + ∫𝑐 𝑓(𝑥)𝑑𝑥, 𝑎 < 𝐶 < 𝑏
𝒂 𝑎
➢ ∫𝟎 𝒇(𝒙) 𝒅𝒙 = ∫0 𝑓(𝑎 − 𝑥)𝑑𝑥
𝑛𝑎 𝑎
𝑊ℎ𝑒𝑛 𝒇(𝒙) = 𝑓 (𝑎 + 𝑥) = ∫ 𝑓(𝑥)𝑑𝑥 = 𝑛 ∫ 𝑓(𝑥)𝑑𝑥
0 0
𝒂 𝑎
➢ ∫−𝑎 𝒇(𝒙)𝒅𝒙 = 2 ∫0 𝑓(𝑥)𝑑𝑥 If 𝑓(−𝑥) = 𝑓(𝑥)
𝒇(−𝒙) = −𝑓(𝑥) = 0

Q. Priority Questions
No.
1. Differentiate each of the following functions with respect to 𝑥.
a) 5𝑥 3 + 5𝑥 2 + 3𝑥– 2
b) 𝑥 𝑏 + 𝑎 𝑥 + 𝑏 𝑏
3𝑥
c)
5𝑥 3 –7
d) 𝑒 𝑥 𝑙𝑜𝑔𝑥
Ans. a) 5𝑥 3 + 5𝑥 2 + 3𝑥– 2
𝑑
𝑥 𝑛 = 𝑛𝑥 𝑛–1
3
𝑦 = 5𝑥 + 5𝑥 + 3𝑥– 2 2
[𝑑𝑥
𝑑 ]
𝑘=0
𝑑𝑥
𝑑𝑦 𝑑
= 𝑑𝑥 (5𝑥 3 + 5𝑥 2 + 3𝑥– 2)
𝑑𝑥
= 5 × 3𝑥 2 + 5 × 2𝑥 + 3 × 1– 0
= 15𝑥 2 + 10𝑥 + 3
b) 𝑦 = 𝑥 𝑏 + 𝑎 𝑥 + 𝑏𝑏
𝑑𝑦 𝑑 𝑑
𝑑𝑥
= (𝑥 𝑏 + 𝑎𝑛 + 𝑏𝑏 )
𝑑𝑥
[𝑑𝑥 𝑎 𝑥 = 𝑎 𝑥 𝑙𝑜𝑔𝑎]
3–1
= 𝑏𝑥 + 𝑎𝑥 𝑙𝑜𝑔𝑎 + 0
3𝑥
c) 𝑦 = 5𝑥 3 –7
𝑑 𝓊
𝑑𝑥 𝒱
( )=
𝑑𝑦 𝑑 3𝑥
𝑑𝑥
= 𝑑𝑥 (5𝑥 3 –7) [𝒱 𝑑 𝒰 –𝒰 𝑑 𝒱]
𝑑𝑥 𝑑𝑥
𝒱2
𝑑 𝑑
(5𝑥 3 –7) (3𝑥)–(3𝑥) (5𝑥 3 –7)
𝑑𝑥 𝑑𝑥
= (5𝑥 3 –7)2

www.escholars.in
37
(5𝑥 3 –7)×3–3𝑥(15𝑥 2 )
= (5𝑥 3 –7)2
15𝑥 3 –21–45𝑥 3
= (5𝑥 3 –7)2
–30𝑥 3 –21
= (5𝑥 3 –7)2
d) 𝑦 = 𝑒 log 𝑥 𝑥
𝑑 𝑑 𝑑
(𝒰𝒱) = 𝒱 𝒰 + 𝒰 𝒱
𝑑𝑥 𝑑𝑥 𝑑𝑥
𝑑𝑦 𝑑 𝑑
= 𝑑𝑥 (𝑒 𝑥 log 𝑥) 𝑑𝑥
𝑒𝑛 = 𝑒𝑛
𝑑𝑥
𝑑 1
[ 𝑑𝑥 𝑙𝑜𝑔𝑥 = 𝑥 ]
𝑑 𝑥 𝑑
log 𝑥 𝑑𝑥 𝑒 + 𝑒 𝑥 𝑑𝑥 log 𝑥
1
log 𝑥 𝑒 𝑥 + 𝑒𝑥 ×
𝑥
1
𝑒 𝑥 (log 𝑥 + 𝑥)

2. 𝑑𝑦
Find 𝑑𝑥 for 𝑦 = 2 𝑧 2 + 1, 𝑧 = 4𝑥– 2
Ans. Note * Question is based on function of a function

𝑦 = 2𝑧 2 + 1 𝑎𝑛𝑑 𝑧 = 4𝑥 – 2
𝑑𝑦 𝑑𝑧
𝑑𝑧
= 4𝑧 𝑎𝑛𝑑 𝑑𝑥
=4
𝑑𝑦 𝑑𝑦 𝑑𝑧
∴ 𝑑𝑥
= 𝑑𝑧
∗ 𝑑𝑥 = 4𝑧 . 4 = 16𝑧 = 16 (4𝑥– 2) = 64𝑥 – 32
3. 𝑑𝑦
Find 𝑑𝑥 for 𝑥 2 + 𝑦 2 = 4
Ans. Note * Question is based on implicit function.

Differentiating both sides


𝑥2 + 𝑦2 = 4
𝑑𝑦
2𝑥 + 2𝑦 =0
𝑑𝑥
𝑑𝑦
2𝑦 = – 2𝑥
𝑑𝑥
𝑑𝑦 –2𝑥
𝑑𝑥
= 2𝑦
𝑑𝑦 –𝑥
𝑑𝑥
= 𝑦
4. If 𝑥 = log t, 𝑦 = et, then 𝑑𝑥
𝑑𝑦

Ans. Note * Question is based on parametric function/


parametric equation.

𝑥 = log t 𝑦 = et
𝑑𝑥 𝑑 𝑑𝑦 𝑑
𝑑𝑡
= 𝑑𝑡 (log 𝑡) 𝑑𝑡
= 𝑑𝑡 (𝑒 𝑡 )
𝑑𝑥 1 𝑑𝑦
𝑑𝑡
= 𝑡 𝑑𝑡
= 𝑒𝑡
𝑑𝑦
𝑑𝑦 𝑒𝑡
∴ = 𝑑𝑡
𝑑𝑥 = 1 = 𝑡𝑒 𝑡
𝑑𝑥
𝑑𝑡 𝑡

5. Differentiate 𝑥 𝑥 𝑤. 𝑟. 𝑡. 𝑥
Ans. Note * Question is based on logarithm Differentiation.
www.escholars.in
38
𝑦 = 𝑥𝑥
Taking logarithm on both sides.
log 𝑦 = log 𝑥 𝑥
log 𝑦 = 𝑥 log 𝑥 [log 𝑎𝑏 = 𝑏 log 𝑎]
Differentiating with respect to 𝑥
1 𝑑𝑦 𝑑 𝑑
𝑦 𝑑𝑥
= log 𝑥 𝑑𝑥 𝑥 + 𝑥 𝑑𝑥 log 𝑥
1 𝑑𝑦 1
𝑦 𝑑𝑥
= log 𝑥 × 1 + 𝑥 × 𝑥
1 𝑑𝑦
𝑦 𝑑𝑥
= log 𝑥 + 1
𝑑𝑦
𝑑𝑥
= 𝑦 (log 𝑥 + 1)
𝑑𝑦
⇒ 𝑑𝑥 = 𝑥 𝑥 (log 𝑥 + 1)
6. If 𝑦 = 𝑎𝑒 𝑛𝑥 + 𝑏𝑒 –𝑛𝑥 then
𝑑2 𝑦
is equal to
𝑑𝑥 2
Ans. Note * Question is based on second order derivative
𝑦 = 𝑎𝑒 𝑛𝑥 + 𝑏𝑒 –𝑛𝑥
Differentiate with respect to 𝑥
𝑑𝑦
= 𝑎𝑒 𝑛𝑥 . 𝑛 + 𝑏𝑒 –𝑛𝑥 (– 𝑛)
𝑑𝑥
𝑑𝑦
𝑑𝑥
= 𝑛𝑎𝑒 𝑛𝑥 – 𝑛𝑏𝑒 –𝑛𝑥
Differentiate with respect to 𝑥
𝑑2 𝑦
𝑑𝑥 2
= 𝑛𝑎 . 𝑒 𝑛𝑥 . 𝑛𝑏 . 𝑒 –𝑛𝑥 (– 𝑛)
= 𝑛2 (𝑎𝑒 𝑛𝑥 + 𝑏𝑒 –𝑛𝑥 )
𝑑2 𝑦
∴ = 𝑥 2𝑦
𝑑𝑥 2
7. Find the gradient of the curve 𝑦 = 5𝑥 2 – 3𝑥 + 8 at the point (2,1)
Ans. Note * Question is based on geometric interpretation of the derivative.
𝑦 = 5𝑥 2 – 3𝑥 + 8
𝑑𝑦
= 10𝑥– 3
𝑑𝑥
𝑑𝑦
𝑥 = 2, 𝑦 = 1
𝑑𝑥
= 10 × 2 – 3 = 17
Thus the gradient (slope) of the curve at (2, 1) is 17
8. The cost function for the production of 𝑥 units of a commodity is given by.
𝑐(𝑥) = 2𝑥 3 – 15𝑥 2 + 36𝑥 + 15
Ans. Note * Question is based on cost function.
𝑐(𝑥) = 2𝑥 3 – 15𝑥 2 + 36𝑥 + 15
𝑐 ′(𝑥) = 6𝑥 2 – 30𝑥 + 36
𝑐 ′(𝑥) = 0
6𝑥 2 – 30𝑥 + 36 = 0
𝑥 2 – 5𝑥 + 6 = 0
𝑥 2 – 3𝑥– 2𝑥 + 6 = 0
𝑥(𝑥– 3) – 2 (𝑥– 3) = 0
(𝑥– 3) (𝑥– 2) = 0
∴ 𝑥 = 3, 2
𝑐 ′′ (𝑥) = 2𝑥 – 5
at 𝑥 = 2
= 2 × 2 – 5 = –1 (–ve) < 0
The function is maxima at 𝑥 = 2 at 𝑥 = 3
2 × 3 – 5 = 1 (+ ve) > 0
The function is minima at 𝑥 = 3.

www.escholars.in
39
9. For a firm the Total Revenue (TR) and Total Cost (TC) functions are given as TR = 20Q and TC =
Q2 = 4Q + 2O where Q = output. Find the profit maximizing output and maximum profit.
Ans. Note * Question is based on Profit function.
Total Revenue (TR) = Price (P) × Quantity (Q) = 4Q
Profit = TR – TC
= 4Q – Q3 + 15Q2 – 31Q – 100
= – 27Q – Q3 + 15Q2 – 100
𝑑(𝑃𝑟𝑜𝑓𝑖𝑡)
= – 27 – 3𝑄2 + 30𝑄.
𝑑𝑄
∴ 3Q2 – 30Q + 27 = 0 [First order condition for maximization]
or,
Q2 – 10Q + 9 = 0
Q2 – Q – 9Q + 9 = 0
Q(Q–1) –9 (Q–1) = 0
(Q–9) (Q–1) = 0
∴ 𝑄 = 9, 1
𝑑2 (𝑃𝑟𝑜𝑓𝑖𝑡) 𝑑 𝑑(𝑃𝑟𝑜𝑓𝑖𝑡) 𝑑
𝑑 𝑄2 𝑑𝑄
[ 𝑑𝑄
]= 𝑑𝑄
[– 27– 3𝑄2 + 30𝑄]
= – 6Q + 30.
At Q = 1 Profit = 24 > 0
At Q = 9 Profit = – 24 < 0.
So the profit is maximized at Q = 9
The maximum profit at Q = 9 is (–27) × 9 – 93 + 15 ×92 – 100
= 143
10. Evaluate the following integral
a) ∫ √𝑥 𝑑𝑥
b) ∫ 𝑒 –3𝑥 𝑑𝑥
c) 𝑥√𝑥𝑑𝑥
𝑥+1
d) ∫ ( 𝑥 2 ) 𝑑𝑥
Ans. 1⁄
1
𝑥 ⁄2+1 𝑥 𝑛+1
a) ∫ √𝑥𝑑𝑥 = ∫ 𝑥 2 𝑑𝑥 = 1⁄ +1 + 𝑐 [∫ 𝑥 𝑛 𝑑𝑥 = 𝑥+1
+ 𝑐]
2
2 3⁄
= 3
𝑥 2+𝑐

𝑒 –3𝑥
b) ∫ 𝑒 –3𝑥 𝑑𝑥 = +𝑐 [∫ 𝑒 3𝑥 𝑑𝑥 = 𝑒 3𝑥 + 𝑐]
–3
–1 –3𝑥
= 3
𝑒 +𝑐
1⁄ 1 3
c) ∫ 𝑥√𝑥𝑑𝑥 = ∫ 𝑥. 𝑥 = ∫ 𝑥 1+ ⁄2 𝑑𝑥 = ∫ 𝑥 ⁄2 𝑑𝑥
2 𝑑𝑥
3 5
𝑥 ⁄2+1 𝑥 ⁄2 5
= 3⁄ +1 + 𝑐 = 5 + 𝑐 = 2⁄5 𝑥 ⁄2 + 𝑐
2 ⁄ 2
𝑥+1 𝑥 1
d) ∫ 𝑥 2 𝑑𝑥 = ∫ (𝑥 2 + 𝑥 2 ) 𝑑𝑥 = ∫ 1⁄𝑥 𝑑𝑥 + ∫ 𝑥 –2 𝑑𝑥
𝑥 –2+1
= log 𝑥 + (–2+1)
+ 𝑐
–1
= log 𝑥 – 𝑥 +𝑐
1
= log – +𝑐
𝑥
11. ∫(3𝑥 + 2) 𝑑𝑥 7

Ans. Note * Question is based on substitution method (change of variable)


Let (3𝑥 + 2) = 𝑡 So 3𝑑𝑥 = 𝑑𝑡
𝑑𝑡
𝑑𝑥 = 3
𝑑𝑡
∴ ∫(3 + 2)7 𝑑𝑥 = ∫ 𝑡7 3

www.escholars.in
40
1 𝑡8 1
=3× 8
= 24
(3𝑥 + 2)8 + 𝑐
12. ∫ 𝑥 log 𝑥 𝑑𝑥
Ans. Note * Question is based on integral by parts Use – ILATE
Integrating by parts.
𝑑
log 𝑥 ∫ 𝑥 𝑑𝑥 – ∫ {𝑑𝑥 (log 𝑥) ∫ 𝑥 𝑑𝑥} 𝑑𝑥
𝑥2 1 𝑥2
= 2
log 𝑥– ∫ [𝑥 – 2
] 𝑑𝑥
𝑥2 1
= 2
log 𝑥 – 2 ∫ 𝑥 𝑑𝑥
𝑥2 𝑥2
2
log 𝑥 – 4
+𝑐
13. Find
𝑥 2 +𝑥+1
∫ (𝑥+2) (𝑥 2 +1) 𝑑𝑥

www.escholars.in
41
Ans. Note Question is based on integration by partial fraction method.
By partial fraction
𝑥 2 +𝑥+1 𝐴 𝐵𝑥+𝑐
(𝑥 2 +1) (𝑥+2)
= (𝑥+2)
+ 2
𝑥 +1
2 (𝑥 2
𝑥 +𝑥+1 =𝐴 + 1) + (𝐵𝑥 + 𝑐) (𝑥 + 2)
Putting 𝑥 = – 2
4 – 2 + 1 = A (5) + 0
⇒ 5𝐴 = 3
3
⇒𝐴= 5
Putting 𝑥 = 0
0 + 0 + 1 = A (0+1) + (0+C) (0+2)
⇒ 1 = A + 2C
3
⇒ 1 = + 2𝐶
5
2
⇒ 2𝐶 = 5
⇒ 𝐶 = 1⁄5
⇒ Putting 𝑥 = 1
⇒ 1 + 1 + 1 = 2A (B+C) (3)
⇒ 3 = 2A + 3 (B+C)
1
⇒ 3 = 2(3⁄5) + 3 (𝐵 + 5)
6
⇒ 3 – = 3 (𝐵 + 1⁄5)
5
2
𝐵=
5
Thus
2 1
𝑥 2 +𝑥+1 3 𝑥+
5 5
(𝑥+2) (𝑥2 +1)
= 5(𝑥+2)
+ 𝑥 2 +1
Now
𝑥 2 +𝑥+1 3 1 2𝑥 1 𝑑𝑥
∫ (𝑥+2)(𝑥 2 +1) 𝑑𝑥 = ∫ 5(𝑥+2) 𝑑𝑥 + 5 ∫ 𝑥 2 +1 𝑑𝑥 + 5 ∫ 𝑥 2 +1
3 1 1
= log|𝑥 + 2| + log|𝑥 2 + 1| + 𝑡𝑎𝑛–1𝑥 + 𝑐
5 5 5
14. 2
∫1 (𝑥 2 – 3𝑥 + 3)𝑑𝑥
Ans. Note * Question is based on definite integration. In definite integration the
constant (c) should not be added.
2
∫1 (𝑥 2 – 3𝑥 + 3)𝑑𝑥
2
𝑥 2+1 3𝑥 1+1
[ 2+1 – 1+1
+ 3𝑥]
1
2
𝑥3 3𝑥 2
[3 – 2
+ 3𝑥]
1
23 3×22 13 3×12
=[3 – 2
+ 3 × 2] – [ 3 – 2
+ 3 × 1]
8 12 6 1 3
[3 – 2 1
+ ] – [3 – 2 + 3]
16–36+36 2–9+18
[ 6
] –[ 6
]
16 11 5
– =
6 6 6
15. The speed of a train at a distance x (from the starting point) is given by 3x2 – 5x + 4. What is the
rate of change (of distance) at x = 1?
Ans. The speed of a train at a distance x is given by
v = 3x2 – 5x + 4
Different w.r.t. ‘x’
𝑑𝑣
( ) = 6 × 1– 5 = 6– 5 = 1
𝑑𝑥 𝑥=1
www.escholars.in
42
Rate of charge (of distance) at x = 1 is 1.

www.escholars.in
43
Business Statistics
Statistical Description of Data
✓ Statistics come from Latin word ‘status’, Italian word ‘Statista’, German word ‘Statistik’ and French
word ‘Statisque’.
✓ Statistics can be defined in a singular and plural sense. In a plural sense, it means the data collected
is qualitative and quantitative. While in the singular, it refers to the methods applied to these data.
✓ Statistics applied in economics, Business Management and Commerce and Industry.
✓ Attribute is a Qualitative characteristic. Variable refers to quantitative characteristics. Discrete, if
finite or infinite and continuous if it assumes a value in a given interval.
✓ Data can be primary or secondary. Primary data, if directly collected from the source, is collected at
the first source. If data has been obtained not from the primary source, then it is secondary data.
✓ Methods of collecting Primary method;
➢ Interview method
➢ Mailed questionnaire method.
➢ Observation method
➢ Questionnaires were filled and sent by enumerators.
✓ Interview Method: Personal interview, indirect interview & Telephone interview method.
✓ Sources of Secondary data: International sources, government sources, private and quasi-
government, unpublished source.
✓ The data collected should be scrutinise since the statistical analyses are made only on the basis of
data; it is necessary to check whether the data under consideration are accurate as well as
consistence.
✓ Mailed questionnaire method is the method of data collection that covers the widest area.
✓ Internal consistency of the collected data can be checked when a number of related series are given.
✓ Classification refers to the process of Arranging data. It makes data more relevant, precise and
condensed, make it comparable, and serve as the base for analysis.
✓ Data can be frequency data and non-frequency data. Time series is an example of non-frequency data.
✓ Presentation of data:
Textual Presentation: Presenting data in Paragraph, it is simple. However, Non-Statistical
Preferred.
Tabular Presentation: Presentation in Tables.
Table has 4 components caption, box head, stub and body; the caption is the upper part of the table
describing Column and Sub-columns. Box head is the entire upper part of the table; the stub is the
left part of the table providing the description of rows. The body is the main part of the table that
contains the numerical figures.
✓ Diagrammatic Presentation:
➢ Line diagram or Historiagram, Example: graph shows the relationship between two
variables, ogives
➢ Bar diagram (horizontal for qualitative data & vertical for quantitative data)
➢ Divided Bar charts or percentage bar diagrams for comparing and relating different
components of a variable.
➢ Pie chart circular diagrams are two dimensional.
✓ Frequency distribution is a tabular representation of statistical data; when made in respect of
discrete series, it is discrete distribution and when it relates to continuous data, it is called grouped
frequency distribution.
✓ Hidden trend, if any, in the data can be noticed in diagrammatic representation.
✓ The chart that uses the logarithm of the variable is known as the ratio chart.
✓ Multiple line chart is applied for two or more related time series when the variables are expressed in
the same unit.
✓ Multiple axis line chart is considered when there is more than one time series, and the units of the
variables are different.
✓ Mutually exclusive series is for continuous series. Mutually inclusive series are for discrete series.

www.escholars.in
44
✓ Relative frequency lies between 0 & 1. Frequency density corresponding to a class interval is a ratio
of class frequency/class length.
✓ Class limit means the upper and lower limit of class interval.
✓ Class boundary refers to the actual class limit of an interval. They are included in class intervals.
✓ Graphical Representation of frequency distribution:
➢ Historiagram helps in comparison of frequencies, calculations of mode – It is also an area
diagram; classes are overlapping. The width of all classes is equal.
➢ Frequency Polygon: Meant for single frequency distribution, all its classes, have equal width.
➢ Ogives or cumulative frequency graph for cumulative distribution – used for quartile, median
etc.
➢ Frequency curve is a smooth curve for which area is a limiting factor of frequency polygon and
historiagram
▪ Bell-shaped – Most used, Example where bell shape curve is used – Profits of a company.
▪ U Shapes
▪ J Shapes
▪ Mixed Curve

Q. Priority Questions
No.
1. Arrange the following dimension wise.
a) Pie–diagram
b) Bar diagram
c) Cubic diagram
Ans. Pie Diagram: Two-dimensional diagram, also called as ‘Area Diagram’. It is used when different
segments or components of value are also to be presented.
Bar Diagram: – One dimensional diagram only height is considered in this case. There is no
importance of width in this case.
Cubic Diagram: – Three-Dimensional diagram in which length, breadth and height is
considered.
2. With the help of histogram one can find __________.
Ans. With the help of histogram one can find mode.
Note: With the help of ogives (cumulative frequency) one can find median and Quartile.
3. Cost of sugar in a month, under the heads raw materials, labour, direct production and others
were 12, 20, 35 and 23 units respectively. The difference between their central angles for the
largest and smallest components of the cost of sugar is
Ans. Head Units Angular value
Raw Material 12 12
× 360 = 48
90
Labour 20 20
× 360 = 80
90
Direct production 35 35
× 360 = 140
90
Others 23 23
× 360 = 92
90

∴ Difference between their central angles for largest and smallest components of the cost of
sugar is = 140–48 = 92
4. A graph that uses vertical bars to represent data is called a ________
Ans. A graph that uses vertical bars to represent data is called as a bar diagram. It is one dimensional.
Width doesn’t matter.,

www.escholars.in
45
5. Frequency density of a class interval is the ratio of _________
Ans. Frequency density of a class interval is the ratio of frequency of that class interval to the
corresponding class length
6. Multiple axis line chart is considered when________
Ans. If there are more than one time series and unit of variables are different then multiple axis chart
is considered
7. A national institute arranged its student’s data in accordance with different states. This
arrangement of data is known as
Ans. Data arranged in accordance with states is geographical data (based on area)
8. Sweetness is an ________
Ans. Sweetness is an attribute (quality)
9. The best way of representing data is ____
Ans. The best way of representing data is tabular form
Note # Easiest method is textual form
# Accurate method is tabular form
# Attractive method is Diagrammatic representation
10. The most commonly used polygon curve is __________
Ans. Bell shaped curve is commonly used in polygon curve in order to find the maximum level of the
curve
11. If a data collected from a census report, what type of data it is?
Ans. If a data is collected from published source, it is called as secondary data, census report is the
data published by the government which is secondary form of data
12. The left part of table providing the description of a row is called
Ans. The left part of the table providing description of a row is called as Stub whereas Caption
represents column of a table.
13. The graphical representation of cumulative frequency distribution is called
Ans. A curve that represents the cumulative frequency distribution of a grouped data on a graph is
called Ogive, (Cumulative frequency on y axis and class interval on X axis)
14. Statistics cannot deal with __________-
Ans. Statistics cannot deal with Qualitative data, However Qualitative data can be converted into
Quantitative data.
15. The difference between upper limit and lower limit of a class interval is called as ____
Ans. The difference between upper limit and lower limit of a class interval is called as Class width or
Class length

www.escholars.in
46
Measure of Central Tendency
✓ Average are two types. It can be Mathematical, or it can be positional average.
✓ Mathematical Average can find by:
➢ Arithmetical mean
➢ Geometrical mean
➢ Harmonic mean
✓ Positional Average can find by:
➢ Median
➢ Mode
✓ Measure of central tendency for a set of observations measures the central location of the
observations.
✓ The best Measure of Central tendency usually is the Arithmetic Mean. It is rigidly defined, based on
all observations, easy to comprehend and simple to calculate. However, it has one drawback that is
very much affected by sampling fluctuations, and it is not used in the case of open-end classification.
✓ Median is also rigidly defined and easy to compute. But it is not based on all the observations, and it
is a positional average, so mathematical formulae cannot be applied.
✓ While computing the A.M from a grouped for frequency distribution, we assume that all the values
of a class are equal to the Mid-value of that class.
✓ For open-end classification, Median is the best measure of central tendency as it is not much affected
by sampling fluctuation.
✓ The presence of extreme observations does not affect the median as it is not affected by sampling
fluctuation.
✓ The arithmetic mean is appropriate if the values have the same units, whereas the geometric mean
is appropriate if the values have different units.
✓ The Harmonic mean is appropriate if the data values are ratios of two variables with different
measures, called rates. That’s why geometric mean and harmonic mean are considered for finding
the average rates.
✓ Relationship between mean, median and Mode for moderately as distribution:
➢ Mean – Mode = 3 (Mean - Median)
➢ Mode = 3 Median – 2 Mean (Imperial relationship)
✓ Weighted Averages are considered when all the observations are not of equal importance.
✓ Relationship between A.M., G.M card H.M:
➢ A.M ≥ G.M ≥ H.M
➢ But A.M = G.M = H.M (when the observations are same and when there are distinct positive
observations then relationship between A.M, G.M and H.M is A.M > G.M > H.M.
➢ When the observations are both positive and negative, then the geometric mean cannot be
used because the root can’t be negative.
➢ A.M, G.M and H.M Posses Mathematical Property.
✓ Mode and Median does not Possess Mathematical Property.
✓ Arithmetic Mean: Sum of all observation divided by no. of observations.
∑𝑥
➢ Individual Series = 𝑁
∑ 𝑓𝑥
➢ Discrete Series/ Frequency distribution = ∑𝑓
∑ 𝑓𝑑′
➢ Continuous Series/Grouped Frequency distribution =A + ∑𝑓
×𝐶
▪ A = Assumed mean
𝑋−𝐴
▪ d’ = 𝐶
▪ C = Class Interval
▪ X = Mid-point of class interval
✓ If all the observations are constant ‘K’, then the mean will also be K.
✓ If the deviations in the series are taken from the mean, then the sum of deviations from the mean
will be zero.
[deviation – A particular number is subtracted from all items of series].
✓ Mean affected by both changes in origin and change in scale.
www.escholars.in
47
✓ If Y = a + b𝑥 then 𝑌̅ = a + b 𝑋̅ . Here a is the change in origin and b is the change in scale.
✓ 𝑌̅ = Mean of y series, 𝑋̅ = Mean of series x
✓ Combined mean:
̅ 𝟏𝟐 = 𝑛1 𝑥̅1+𝑛2 𝑥̅2
𝑿 𝑛1 + 𝑛2
𝑛+1
✓ Mean, first n natural number = 2
✓ Mean, first n odd natural number = 𝑛
(𝑛+1)(2𝑛+1)
✓ Mean of square of first n natural number is = 6
(𝑛+1) 2
✓ Mean of cubes of first, n natural number = 𝑛 [ 2 ]
✓ Median:
➢ It is a positional average, divided a series into two parts, not affected by extremes.
𝑛+1 𝑡ℎ
➢ Median = ( ) in the case of an odd number, and in the case of an even number, it is a
2
simple average of two middle values.
𝑛
− 𝐶𝐹𝑝
➢ For grouped frequency = 𝐿1 + 2 𝑓
×𝑖
𝑳𝟏 = lower limit
𝒏 = Total frequency
𝑪𝑭𝒑 = Previous cumulative frequency from the median class
𝒇 = Frequency in median class
➢ For moderately skewed distribution, y = a + b𝑥.
➢ If absolute deviations are taken from the median, the sum of absolute deviations will be
minimum.
✓ Quartiles, deciles and Percentile:
➢ It is also positional average quartiles, deciles, and percentiles divide equation into 4, 10 and
100 parts respectively. There are 3, 9, 99 Quartile, decile and percentile.
➢ 𝑄2 = 𝐷5 = 𝑃50 = 𝑀𝑒𝑑𝑖𝑎𝑛
𝑁
𝑘( )−𝐶𝐹𝑝
➢ For quartiles = 𝐿1 + 4
×𝑖
𝑓
𝑁
𝑘( )−𝐶𝐹𝑝
➢ For deciles = 𝐿1 + 10
×𝑖
𝑓
𝑁
𝑘( )−𝐶𝐹𝑝
➢ For percentile = 𝐿1 + 100
𝑓
×𝑖
✓ Mode:
➢ It is the most popular measure of central tendency; there are cases when mode remains
undefined.
➢ Highest value in the series. It is not uniquely defined; it does not exist if all the observations
are equal. It represents a number which has repeatedly been most of the time.
𝑓 −𝑓
➢ Mode = 𝐿1 + 1 0 × 𝑖
2𝑓1 − 𝑓0 − 𝑓2
➢ When it is difficult to compute mode with the formula, it can be calculated using an equation.
➢ Mode = 3 Median – 2 Mean.
➢ For Moderately skewed distribution on y = a + 𝑏𝑥.
➢ Graphically it can be calculated by Histogram.
➢ Mode is affected by sampling fluctuations.
✓ Geometric Mean:
➢ It is the nth root of the product of the observation.
1
➢ For individual series G.M = (𝑥1 × 𝑥2 × 𝑥3 × __________𝑥𝑛 )𝑛
1
➢ For grouped frequency G.M = (𝑥1 𝑓1 × 𝑥2 𝑓2 × 𝑥3 𝑓3 × __________𝑥𝑛 𝑓𝑛 )𝑛
𝟏 1
➢ Where 𝒏
= ∑𝑓
➢ If all the observation of a series is K, then G.M is also K.

www.escholars.in
48
➢ Geometric mean of the product of two variables is the product of their G. M’s, i.e. 𝑍 = 𝑥𝑦 then
G.M of Z = (G.M of 𝑥) × (G.M of y).
➢ The ratio of two variables is the ratio of their G.M.
➢ It can be calculated if all the observations have a positive sign and none of them is 0.
➢ It is used for calculating the growth rate of the population.
➢ It is rigidly defined, difficult to comprehend for computing.
✓ Harmonic Mean:
➢ It is reciprocal of the A.M of the reciprocal of the observation.
𝑁
➢ H.M = 1 [for individual series]
∑( )
𝑥
𝑁
➢ H.M = 𝑓
∑( 𝑖 )
𝑥𝑖
➢ If all the observation is K, then the Harmonic is also K.
𝑛 +𝑛
➢ Combined Harmonic Mean = 𝑛11 𝑛22
+
ℎ1 ℎ2
➢ It is used for calculation of the average of prices, used for an average of speed.
✓ Weighted Average are as follows:
∑ 𝑊𝑥
➢ Weighted A. M = ∑ 𝑊
∑ 𝑊 log 𝑥
➢ Weighted G.M. = Antilog ∑𝑊
∑𝑊
➢ Weighted H.M = 𝑊𝑖
∑( )
𝑥𝑖

➢ Weighted Averages are used when all the observations are not of equal values.
➢ For two value A.M × H.M = 𝐺𝑀2

Q. Priority Questions
No.
1. Compute AM, GM and HM for the numbers 3,4,6,18. Given two positive numbers 𝑝 𝑎𝑛𝑑 𝑞, prove
that A.M × H.M = (G.M)2. Does the result hold for any set of observations?
x +x +x +⋯….xn
Ans. Arithmetic Mean (A.M) = 1 2 3
x
3+4+6+18 31
= 4
= 4
= 7.75
𝑛
Harmonic Mean (H.M) = 1 1 1 1
+ + ….+
𝑥1 𝑥2 𝑥3 𝑥4
4
=1 1 1 1 = 4.97(approx.)
+ + +
3 4 6 18
Geometric Mean (G.M.) = (𝑥1×𝑥2×….𝑥n)1/n
= (3×4×6×18)1/4
= (64)1/4 = 6
Note* short cut to calculate G.M.
Step I → Press square root 12 times
Step II → –1 =
Step III → Divide by power (given in denominator) =
Step IV → +1 =
Step V → Press × = 12 times

For two positive numbers 𝑝, 𝑎𝑛𝑑 𝑞,


𝑝+𝑞
A.M =
2
G.M = √𝑝𝑞
2𝑝𝑞
H.M =
𝑝+𝑞
𝑝+𝑞 2𝑝𝑞
AM × H.M = 2
× 𝑝+𝑞 = 𝑝𝑞
𝑝𝑞 = (G.M)2
www.escholars.in
49
Hence A.M × H.M = (G.M)2
The result holds for only wo positive observations and not for any set of observation.
2. Find the weighted A.M., weighted H.M of.
x 1 2 3 4
w 12 22 32 42
Ans. 𝑤𝑖 𝑥𝑖
Weighted A.M = 𝑤 𝑖
12 ×1+22 ×2 +32 ×3 +42 ×4 100
= = = 3.3
12 + 22 +32 + 42 30
𝑤𝑖
Weighted H.M = 𝑤
( 𝑥 𝑖 )
𝑖
12 +22 +32 +42 1+4+9+16
= 12 22 32 42
= 1 4 9 16
+ + + + + +
1 2 3 4 1 2 3 4
30 30
= 1+2+3+4 = 10 = 3
3. For a moderately skewed distribution of marks in statistics for a group of 100 students the
mean and median mark were found to be 27.8 and 26.2. What is the modal marks. In case of
symmetrical distribution if Mean = 5, find median and mode.
Ans. In case of moderately skewed distribution;
Mean – Mode = 3 (Mean – Median)
∴ Mode = 3 Median – 2 Mean
= 3 × 26.2 – 2 × 27.8
Mode (₹) = 78.6 – 55.6
Mode (₹) = 23
In case of symmetrically distribution
Mean = Median = Mode
∴ Mean = Median = Mode = 5
4. If 𝑦 = 1 + 0.75𝑥 and mode of 𝑥 is 7.5, what is the mode of 𝑦.
Ans. 𝑦 = 1 + 0.75𝑥
Mode of 𝑦 = 1 + 0.75 mode of 𝑥
Mode of 𝑦 = 1 + 0.75 × 7.5
= 6.625
Note: – Mean, mode and median affects due to change in origin as well as change in scale.
5. Find mean, mode and median of the following observations.
C.I 0–10 10–20 20–30 30–40
F 3 2 5 1
Ans. For mean
C.I 𝒇𝒊 𝒙𝒊 𝒇𝒊𝒙𝒊
0–10 3 5 15
10–20 2 15 30
20–30 5 25 125
30–40 1 35 35
∑ 𝑓 = 11 ∑ 𝑓𝑖𝑥𝑖 = 205

∑ 𝑓𝑖𝑥𝑖 205
Mean (𝑋) = ∑ 𝑓𝑖
= 11
= 18.64
For median
C.I 𝒇𝒊 cf
0–10 3 3
10–20 2 5
20–30 5 10
30–40 1 11
N = 11
www.escholars.in
50
𝑁⁄ 𝑡ℎ obsv = 11⁄ th obsv 5.5 th obsv.
2 2
𝑁⁄ –𝑐𝑓𝑝
Median (Md) = L+ 2 ×𝑖
𝑓
5.5–5
= 20 + 5
× 10
0.5
= 20 + 5 ×10
= 20 +1 = 21
For Mode
C.I 𝒇𝒊
0–10 3
10–20 2 – f0
20–30 5–f1
30–40 1 –f2
𝑓1 –𝑓0
Mode (₹) = L + ×𝑖
2𝑓1 –𝑓0 –𝑓2
5–2
= 20 + 2×5–2–1 × 10
3
= 20 + 7 × 10
= 24.29 (approx.)
6. Following are the wages of the labourers ₹72, ₹46, ₹80, ₹40, ₹110, ₹65, ₹70, ₹120, ₹55, Find
Q1, Q6, P82
Ans. Average the wages in ascending order
40, 46, 55, 65, 65, 70, 72, 80, 110, 120
(𝑛+1)𝑡ℎ
Q1 = value.
4
10+1 𝑡ℎ
= ( 4 ) value.
= 2.75th value.
= 2nd value + 0.75 × (3rd value – 2nd value)
= 46 + 0.75 × (55–46)
= ₹52.75
𝑛+1 𝑡ℎ
D6 = k ( ) value
10
10+1 𝑡ℎ
= 6( ) value
10
6.6 value.
th

= 6th value + 0.6 × (7th value – 6th value)


= 70 + 0.6 × (72–70)
D6 =71.20
𝑛+1 𝑡ℎ
P82 = k( 100 ) value
10+1 𝑡ℎ
= 82 ( 100 ) value
9.02th value
= 9th value + 0.2 (10th value – 9th value)
= 110 + 0.2 (120–110)
P82 = 112
7. The mean salary for a group of 12 males workers is ₹1560 per month and that for a group of
18 female workers is ₹2040. What is combined mean salary.
Ans. Given n1 = 12, n2 = 18, 𝑥1 = 1560 and 𝑥2 = 2040
𝑛 𝑥 +𝑛 𝑥
∴ Combined Mean (𝑥12 ) = 1 1 2 2 𝑛1 +𝑛2
12×1560+18×2040
= 12+18
18720+36720 55440
= 30
= 30
= ₹1848
www.escholars.in
51
8. The average salary of 50 men was ₹80 but it was found that salary of 2 of them were ₹46 and
₹28 which was wrongly taken as ₹64 and ₹82. The revised average salary is
Ans. Mean (𝑥) =
∑𝑥
𝑁
∑𝑥
80 =
50
∴ Wrong ∑ 𝑥 = 80 × 50 = 4000
Correct ∑ 𝑥 = 4000– 64– 82 + 46 + 28 = 3928.
∑𝑥 3928
∴ Revised average salary = 𝑁 = 50 = ₹78.56
9. Which of the following measures of central tendency cannot be calculated by graphical
method?
i) Mean
ii) Mode
iii) Median
iv) Quartile
Ans. Mean cannot be calculate by graphical method.
Median and Quartile can be calculated by ogive.
Mode can be calculated by histogram.
10. The rates of returns from three different shares are 50% 100% and 200% respectively. The
average rate of return will be.
Ans. Note * If given data are in the form of % then, use G.M. for average.
1
G.M = (𝑥1 . 𝑥2 . 𝑥3 ) ⁄3
1
= (50 × 100 × 200) ⁄3
1
= (10000000) ⁄3
= 100%
11. A person purchases 4 rupees worth of eggs from 8 different markets. If you are to find the
average number of eggs per rupee purchased from all the markets taken together. The suitable
average in this case is.
Ans. In case of rate, average speed (speed given) , use H.M. for suitable average.
12. One hundred participants expressed their opinion on recommending a new product to their
friends using the attributes: most unlikely, not sure, likely, most likely. The appropriate
measure of central tendency that can be used here is
Ans. One hundred participants expressed their opinion on recommending a new product to their
friends using the Attributes; most unlikely, not sure, likely, most likely.
The appropriate measure of central tendency that can be used here is Mode.
13. Along a road there are 5 buildings of apartments, marked as 1, 2, 3, 4, 5. Number of people
residing in each building is available. A bus stop is to be setup near one of the buildings so that
the total distance walked by the residents to the bus stop from their buildings must be kept
minimum. One must consider involving _____________ to find the position of the bus stop.
Ans. ‘Median’
The total distance walked by the residents to the bus stop from their building must be kept
minimum.
14. Which one of these affected by extreme values?
i) Mean
ii) Median
iii) Mode
iv) None
Sol. Median is not affected by extreme values
Mean is based on all values
Mode represents value having highest frequency.
15. The ordering of a particular design of a cloth showroom, a ________ size be more appropriate.
Sol. The ordering of the particular design of a cloth showroom, a mode size be more appropriate. It
represents observation with highest frequency.
www.escholars.in
52
Measure of Dispersion
✓ Measure of dispersion represents the Scatterness of the series.
✓ It is broadly classified into:
➢ Absolute Measures of dispersion
➢ Relative Measures of dispersion
✓ Absolute Measures of dispersion are:
a) Range; b) Mean deviation; c) Standard deviation; d) Quartile deviation.
✓ Relative Measures of dispersion are: -
a) Coefficient of Range; b) Coefficient of Mean deviation; c) Coefficient of variation; d) Coefficient of
Quartile deviation.
✓ Absolute measures of dispersion are unit passed, whereas the Relative measures of dispersion are
unit free.
✓ For comparing two or more distributions, relative measures of dispersion are considered, not
absolute measures of dispersion are considered.
✓ Absolute measures of dispersion are easy to compute. Whereas relative measures of dispersion are
difficult to compute and comprehend.
✓ Standard deviation is the most useful measure of dispersion.
✓ Quartile deviation is not affected by the presence of extreme observations.
✓ Mean deviation is based on absolute deviations only.
✓ Quartile deviation is the most appropriate measure of dispersion for open-end classification.
✓ Standard deviation is considered for finding a pooled measure of dispersion after combining several
groups.
✓ For any two numbers, the standard deviation is always half of the range.
✓ Range:
➢ Range = Largest – Smallest [𝐿 − 𝑆]
𝐿−𝑆
➢ Coefficient of Range = × 100. It is affected by the presence of extremes, just based on
𝐿+𝑆
two observations. Affected by change of scale, not origin.
➢ If equation 𝒚 = 𝑎 + 𝑏𝑥, 𝑡ℎ𝑒 𝑹𝒚 = |𝑏| × 𝑅𝑥
✓ Mean Deviation:
➢ It is defined as the arithmetic mean of the absolute deviations of the observations from Mean,
Median or Mode.
∑ |𝑋−𝑋̅|
➢ For ungrouped frequency, Mean deviation =
𝑁
∑ 𝑓|𝑋−𝑋̅|
➢ For grouped frequency, Mean deviation = ∑𝑓
𝑀𝑒𝑎𝑛 𝑑𝑒𝑣𝑖𝑎𝑡𝑖𝑜𝑛 𝑓𝑟𝑜𝑚 𝑀𝑒𝑎𝑛
➢ Coefficient of Mean deviation = 𝑜𝑟
𝑀𝑒𝑎𝑛
𝑀𝑒𝑎𝑛 𝑑𝑒𝑣𝑖𝑎𝑡𝑖𝑜𝑛 𝑓𝑟𝑜𝑚 𝑚𝑒𝑑𝑖𝑎𝑛
➢ = 𝑀𝑒𝑑𝑖𝑎𝑛
𝑀𝑒𝑎𝑛 𝑑𝑒𝑣𝑖𝑎𝑡𝑖𝑜𝑛 𝑓𝑟𝑜𝑚 𝑚𝑜𝑑𝑒
➢ = 𝑀𝑜𝑑𝑒
✓ Standard deviation:
➢ It is the best method to calculate dispersion, and it is rigidly defined.
➢ It is defined as the root mean square deviation when the deviations are taken from the A.M.
of the observations.
∑(𝑋−𝑋̅)2
➢ For ungrouped frequency distribution, 𝑺. 𝑫. = √ 𝑛
∑ 𝑓(𝑋−𝑋̅)2
➢ For grouped frequency distribution, 𝑺. 𝑫. = √ ∑𝑓
➢ Variance = (𝑆𝐷)2
𝑆.𝐷.
➢ Coefficient of variation represents the variation in a series = 𝐴𝑀. × 100 More C.V. means More
dispersion, and less coefficient of variation means more consistent.
➢ If all the variables of the series are K, then S.D. is zero.
➢ Standard deviation is affected by the change of scale only, but it is not affected by the change
of origin.
www.escholars.in
53
➢ 𝐼𝑓 𝑦 = 𝑎 + 𝑏𝑥 𝑡ℎ𝑒𝑛, 𝑆. 𝐷𝑦 = |𝑏| × 𝑆. 𝐷𝑥
𝑛1 𝜎1 2 +𝑛2 𝜎2 2 +𝑛1 𝑑1 2 +𝑛2 𝑑2 2
➢ Combined Standard Deviation = √
𝑛1 + 𝑛2
➢ Where,
𝒅𝟏 = 𝑋̅12 − 𝑋̅1
𝒅𝟐 = 𝑋̅12 − 𝑋̅2
𝒏𝟏 = 𝑁𝑜. 𝑜𝑓 𝑖𝑡𝑒𝑚𝑠 𝑖𝑛 𝑆𝑒𝑟𝑖𝑒𝑠 1.
𝒏𝟐 = 𝑁𝑜. 𝑜𝑓 𝑖𝑡𝑒𝑚𝑠 𝑖𝑛 𝑆𝑒𝑟𝑖𝑒𝑠 2.
𝝈𝟏 = 𝑆. 𝐷𝑜𝑓 𝑡ℎ𝑒 𝑆𝑒𝑟𝑖𝑒𝑠 1.
𝝈𝟐 = 𝑆. 𝐷𝑜𝑓 𝑡ℎ𝑒 𝑆𝑒𝑟𝑖𝑒𝑠 2.
|𝑎−𝑏|
➢ For any, the number S.D. is half of the range. Standard deviation of two numbers a & b 2
𝑛2 −1
➢ Standard deviation of the first ‘n’ natural number is √ 12
✓ Quartile Deviation:
𝑄 −𝑄
➢ Quartile deviation or Semi-Quartile deviation = 𝑄. 𝐷. = 3 2 1
𝑄3 −𝑄1
➢ Coefficient of Quartile Deviation = × 100
𝑄3 +𝑄1
➢ It is the best measure for open-end distribution, it is not affected by the change of origin, but
it is affected by the change scale.
➢ It is just based on the 50% of the observation
➢ Inter-Quartile range = 𝑄3 − 𝑄1 .
➢ 4SD = 5MD = 6QD, when the data is normal distributed

Q. Priority Questions
No.
1. Following are the wages of 10 workers in rupees 80, 94, 50, 73, 68, 63, 48, 68, 74, 78 Find range
and also its coefficient.
Ans. Range (The difference between largest and smallest observation) = L – S = ₹94 – ₹48 = ₹46
𝐿–𝑆
Coefficient of Range = 𝐿+𝑆 × 100
94–48
= × 100
94+48
46
= × 100 = 32.39
142
2. If x and y are related as 2𝑥 + 3𝑦 + 7 = 0 and mean deviation of 𝑥 is 4.20, what is the mean
deviation of 𝑦.
Ans. 2𝑥 + 3𝑦 + 7 = 0
3𝑦 = – 2𝑥 – 7
2 7
𝑦 = – 3 𝑥– 3
Hence, MD of 𝑦 = |b| × MD of 𝑥
2
M.D of 𝑦 = ⌊– ⌋ × M.D. of 𝑥
3
2
M.D of 𝑦 = + 3 × 4.20
= 2.80 (approx..)
Note * Range, Mean deviation, Quartile deviation, standard deviation, does not affect with change
in origin but affects with change in scale with modulus function.
3. The following observation is given.
3, 6, 7, 1, 5, find
i) Mean Deviation and coefficient of Mean Deviation.
ii) Quartile Deviation and Coefficient of Quartile Deviation.
iii) Standard Deviation and Coefficient of variation.
www.escholars.in
54
Ans. For Mean Deviation and Coefficient of Mean Deviation
X ⌊𝑿– 𝑿⌋
3 1.4
6 1.6
7 2.6
1 3.4
5 0.6
22 9.6

∑𝑥 22
𝑥= = = 4.4
𝑁 5
∑⌊𝑋–𝑋⌋ 9.6
M.D = = = 1.92
𝑁 5
𝑀.𝐷
Coefficient of M.D = × 100
𝑋
1.92
= 4.4
× 100 = 43.64
For Quartile Deviation and coefficient of Quartile Deviation
Arrange the observations in ascending order
x
1
3
5
6
7
𝑁+1 𝑡ℎ
First quartile (Q1) = ( ) obsv.
4
5+1 𝑡ℎ
=( ) obsv
4
= 1.5th obsv.
1st value + 0.5 × (2nd value – 1st value)
1+0.5×(3–1) = 2.
3(𝑁+1)
Q3 = 4
th obsv.
= 3 × 1.5th obsv. = 4.5th obsv.
4th value + 0.5 (5th value – 4th value)
6 + 0.7 (7–6)
= 6.7*
𝑄3 –𝑄1 6.7–2
Quartile Deviation (Q.D) = 2
= 2
= 2.35
𝑄3 –𝑄3 6.7–2
Coefficient of Q.D = × 100 = × 100
𝑄3 +𝑄1 6.7+2
4.7
= 8.7 × 100
= 54.02
For standard deviation and coefficient of variation
X X2
3 9
6 36
7 49

www.escholars.in
55
1 1
5 25
120

∑ 𝑥2 2
Variation (𝜎 2 ) = –𝑋
𝑁
120
= – (4.4)2
5
= 24–19.36
= 4.64
∑𝑥 22
𝑥= 𝑁
= 5
= 4.4
∑ 𝑥2 2
S.D (𝜎) = √ 𝑁
–𝑥
= √4.64
= 2.15
𝑆.𝐷 2.15
Coefficient of variation (C.V) = 𝐴.𝑀 × 100 = 4.4
× 100
= 48.86
4. For a normal distribution the relation between Quartile Deviation (Q. D) and (S. D) is.
Ans. For Normal Distribution
4 S. D = 5 M. D = 6 Q. D
6 QD = 4 SD
4
QD = S.D.
6
2
QD = 3 S. D.
5. If the standard deviation of first ‘n’ natural numbers is 2, then value of ‘n’ is ……..
Ans. 𝑛2 –1
S. D. of first ‘n’ natural numbers = √
12

𝑛2 –1
∴2= √
12
Squaring both sides,
𝑛2 –1
4=
12
n2 = 48 + 1
n = √49 = 7
6. The formula for range of middle 50% items of a series is
Ans. The formula for range of middle 50% items of a series is Q.D i.e. Q. D =
𝑄3 –𝑄1
2
7. Coefficient of variation is equal to _________
Ans. In statistics coefficient of variation also known as relative standard deviation is a standardized
measure of dispersion of frequency distribution.
It is expressed as a percentage and defined as the ratio of SD and Mean.
𝑆𝐷
∴ Coefficient of variation = 𝑀𝑒𝑎𝑛 × 100.
8. Which of the following measure of dispersion is based on absolute deviations.
i) Range
ii) S. D
iii) Mean Deviation
iv) Quartile Deviation

www.escholars.in
56
Ans. Only Mean Deviation (M. D) is based on Absolute Deviation and sum of absolute deviation from
median is zero.
9. Which of the following is absolute measure of dispersion.
i) Range
ii) Coefficient of Range
iii) Coefficient of Mean Deviation
iv) Coefficient of Quartile Deviation
Ans. Range is based on absolute measure of dispersion whereas others {(ii), (iii) & (iv)} are based on
relative measure of dispersion.
10. If the variance of random variable ‘𝑥’ is 17, then what is variance of 𝑦 = 2𝑥 + 5
Ans. Given, variance of 𝑥 (𝜎𝑥 2 ) = 17
S. D of 𝑥 (𝜎𝑥 ) = √17
S. D. of 𝑦 = ⌊𝑏⌋ S. D. of 𝑥
Given equation,
𝑦 = 2𝑥 + 5
S. D of 𝑦 = ⌊2⌋ × √17
= 2√17
2
∴ Variance of 𝑦 (𝜎𝑦 2 ) = (2√17)
= 4 × 17
= 68
11. If a = 5 and b = 3, find the standard deviation between a and b.
Ans. S. D for any two numbers is given by
⌊𝑎–𝑏⌋
S. D = 2
⌊5–3⌋ 2
= = =1
2 2
(Note * The absolute sign is taken, as S. D cannot be negative)
12. For any group of 50 boy students, the mean and SD of stats marks are 35 and 1 respectively. The
same figures for a group of 30 girl students are 45 and 2 respectively. What is the S. D of marks
if the two groups are pooled together. (Combined mean)
𝑛1 𝑥1 +𝑛2 𝑥2
𝑥12 = 𝑛1 +𝑛2
50×35+30×45
=
50+30
1750+1350
= 80
= 38.75
Ans. 𝑛1 𝑆12 +𝑛2 𝑆22 +𝑛1 𝑑12 +𝑛2 𝑑22
Combined S. D = √
𝑛1 +𝑛2
Given – 𝑛1 = 50, 𝑛2 = 30, 𝑆1 = 1, 𝑆2 = 2
𝑥1 = 35, 𝑥2 = 45
∴ 𝑑1 = 𝑥1 – 𝑥12 = 35– 38.75 = – 3.75
𝑑2 = 𝑥2 – 𝑥12 = 45 – 38.75 = 6.25
50×12 +30×22 +50×(–3.75)2 +30×(6.25)2
∴ Combined S. D = √ 50+30

50+120+703.125+1171.875
=√ 80

= √25.5625 = 5 (approx..)

www.escholars.in
57
13. Factory Mean Salary S. D (Salary)
A 10,000 30
B 12,000 40

Which factory is consistent in paying salaries.


Ans. 𝜎
C. VA = 𝑥 × 100 =
30
× 100 = 0.3
10,000
𝜎 40
C. VB = × 100 = × 100 = 0.33
𝑥 12,000
Hence, Factory A is consistent in paying salaries.
14. In measure of dispersion ____________ is based on all the observations and _________ is based on the
central fifty percent of the observations.
Ans. Mean deviation or standard deviation is based on all the observations whereas quartile deviation
is based on the central fifty percent of the observations.
15. If every observation is increased by 7 then what will be the affect on M.D, Q.D, S.D and Range.
Ans. By shifting the origin (add or subtract) S.D, M.D, Q.D and Range does not change So, if every
observation is increased by 7 then M. D, S. D, Q. D and Range will be changed.

www.escholars.in
58
Probability
✓ Probability means likelihood or something that is likely to happen; two types of probability are
subjective and objective.
✓ Experiment may be described as a performance that produces a certain result. The results or
outcomes of a random experiment are known as events. Sometimes events may be a combination of
outcomes. The events are of two types.
➢ Simple or elementary
➢ Composite or Compound
✓ Mutually Exclusive: A set of evenly A and B are known to be mutually exclusive. If nothing is
common between them or none of them can occur simultaneously. The occurrence of one event
implies the non-occurrence.
For Example 𝑃(𝐴 ∩ 𝐵) = 0
✓ Exhaustive event: The events 𝐴1 , 𝐴2 , 𝐴3 , …………… are known to be exhaustive. If one of the events
must occur and the set of events constitute the Universe.
✓ Equally likely: The events are those whose occurrence is equal.
𝐶ℎ𝑎𝑛𝑐𝑒𝑠 𝑜𝑓 ℎ𝑎𝑝𝑝𝑒𝑛𝑖𝑛𝑔
✓ Odds in Favour = 𝐶ℎ𝑎𝑛𝑐𝑒𝑠 𝑜𝑓 𝑛𝑜𝑡 ℎ𝑎𝑝𝑝𝑒𝑛𝑖𝑛𝑔
𝐶ℎ𝑎𝑛𝑐𝑒𝑠 𝑜𝑓 𝑛𝑜𝑡 ℎ𝑎𝑝𝑝𝑒𝑛𝑖𝑛𝑔
✓ Odds in Against =
𝐶ℎ𝑎𝑛𝑐𝑒𝑠 𝑜𝑓 ℎ𝑎𝑝𝑝𝑒𝑛𝑖𝑛𝑔
✓ The Probability of an event lies between 0 and 1, both Inclusive, i.e., 0 ≤ 𝑃(𝐴) ≤ 1
✓ When P (A) = 0, A is known to be an Impossible event, and when P (A) = 1, A is known to be a sure
event.
✓ Non-occurrence of event A is denoted by 𝐴𝑐 or 𝐴′ 𝑜𝑟 𝐴̅ It is known as a complimentary event of A.
Event A along with its complimentary A’ forms a set of mutually exclusive and exhaustive events.
i.e., 𝑃(𝐴) + 𝑃(𝐴′ ) = 1
𝑃 (𝐴′ ) = 1 − 𝑃(𝐴)
✓ Important Formulas:
➢ 𝑷 (𝑨 ∪ 𝑩) = 𝑃(𝐴) + 𝑃(𝐵) − P(A ∩ B)
➢ 𝑷 (𝑨 ∪ 𝑩 ∪ 𝑪) = 𝑃(𝐴) + 𝑃(𝐵) + 𝑃(𝐶) − 𝑃(𝐴 ∩ 𝐵) − 𝑃(𝐵 ∩ 𝐶) − 𝑃(𝐶 ∩ 𝐴) + 𝑃(𝐴 ∩ 𝐵 ∩ 𝐶)
➢ 𝑷(𝑨 − 𝑩) = 𝑃(𝐴) − 𝑃(𝐴 ∩ 𝐵)
➢ 𝑷(𝑨 ∩ 𝑩) = 0 For mutually exclusive events.
➢ 𝑷(𝑨 ∪ 𝑩 ∪ 𝑪) = 1 For mutually exhaustive events.
➢ 𝑷(𝑨 ∩ 𝑩) = 𝑃(𝐴) × 𝑃(𝐵), For independent events.
𝑩 𝑃(𝐴∩𝐵)
➢ 𝑷( ) = , For compound probability or conditional probability.
𝑨 𝑃(𝐵)
𝑨′ 𝑃(𝐴′ ∩𝐵) 𝑃(𝐵)−𝑃(𝐴∩𝐵)
➢ 𝑷( ) = =
𝑩 𝑃(𝐵) 𝑃(𝐵)
➢ When 𝑥 is a discrete random variable with probability mass function f (𝑥), then its expected
value is given by
𝑙𝑙 = ∑ 𝑥 𝑓(𝑥)
Variable = 𝐸(𝑥)2 − 𝑙𝑙 2
Where = 𝐸(𝑥)2 = ∑ 𝑥 2 𝑓(𝑥)
✓ Expectation of the product of a constant of a constant and a random variable is the product of the
constant and the expectation of the random variable.
𝑬(𝒌 𝒙) = 𝑘 𝐸(𝑥)
✓ Expectation of the product of two random variables is the product of the expectation of the two
random variables. Provided two variables are Independent.
𝑬 (𝒙𝒚) = 𝐸(𝑥) × 𝐸(𝑦)
Whenever 𝑥 and y are Independent

www.escholars.in
59
Q. Priority Questions
No.
1. A coin is tossed three times. What is the probability of getting:
i) Exactly 2 heads
ii) Atmost 2 heads
iii) Atleast 2 heads
Ans. HHH, HHT HTT TTT
S={ HHH THT }
THH TTH
3
i) P (Exactly two heads) = [HHT, HT, THH]
8
7
ii) P (At most two heads) i.e. 2 heads + 1 head + No head = 8 [HHT, HTH, THH, HTT, THT, TTH,
TTT]
4 1
iii) P (At least two heads) i.e. 2 heads + 3 heads = 8 = 2
[HHT, HTH, THH, HHH]
2. A dice is rolled twice. What is the probability of getting?
i) Difference of two points
ii) Sum greater than 10
iii) Doublet.
Ans. (1,1) (1,2) (1,3) (1,4) (1,5) (1,6)
(2,1) (2,2) (2,3) (2,4) (2,5) (2,6)
(3,1) (3,2) (3,3) (3,4) (3,5) (3,6)
S=
(4,1) (4,2) (4,3) (4,4) (4,5) (4,6)
(5,1) (5,2) (5,3) (5,4) (5,5) (5,6)
{(6,1) (6,2) (6,3) (6,4) (6,5) (6,6) }
8
i) P (Difference of two points = 36 [(1,3) (2,4) (3,1) (3,5) (4,2) (4,6), (5,3) (6,4)]
2
=
9
3
ii) P (Sum greater than 10) = [(5,6) (6,5) (6,6)]
36
1
=
12
6
iii) P (Doublet) = [(1,1)(2,2) (3,3) (4,4)(5,5) (6,6)]
36
1
=
6
3. What is the chance of picking
i) a club or an ace not of club from a pack of 52 cards.
ii) King and queen of spade.
iii) Red coloured cards.
Ans. Total number of elementary events (total outcome) = 52
16
i) P (A club or and ace not of club) = 52
4
=
13
Total no. of club = 13
[Total no. of ace excluding ace of club = 3]
Favourable outcome = 16
2
ii) P (King and queen of spade) = 52
1
= 26

www.escholars.in
60
King of spade =1
[ Queen of spade = 1]
Favourable outcome = 2
26
iii) P (Red coloured cards) = 52
1
=
2
No of Hearts (Red Coloured) = 13
[No of Diamond (Red Coloured cards) = 13]
Favourable outcome = 26

4. A committee of 8 members is to be formed from a group comprising 9 gentlemen and 6 ladies.


What is the probability that the committee would comprise.
i) 3 ladies
ii) At least 3 ladies.
Ans. Since, there are altogether 9 + 6 = 15 persons, a committee comprising 8 members can be formed
in,
15!
15𝐶8 = 8!7!
𝑛!
[𝑛𝐶𝑟 = ]
𝑟!(𝑛–𝑟)!
15×14×13×12×11×10×9×8!
= 8! 7!
15×14×13×12×11×10×9
Total ways = = 15×13×3×11 ways.
7×6×5×4×3×2
6𝑐3 ×9𝑐5
i) P (3 ladies) =
15×13×3×11
20×126
=
15×13×3×11
56
=
143
ii) Favourable outcome = 6𝑐3 × 9𝑐5 + 6𝑐4 × 9𝑐4 + 6𝑐5 × 9𝑐3 + 6𝑐6 + 9𝑐2
= 20 × 126 + 15 × 126 + 6 × 84 + 1 × 36
= 2520 + 1890 + 504 + 36
= 4950
P(At least 3 ladies)
4950 990
= 6435 = 1287
5. The three events A, B and C are mutually exclusive, exhaustive and equally likely. What is the
probability of the complementary event of A.
Ans. Mutually exclusive
𝑃(𝐴 ∪ 𝐵 ∪ 𝐶) = 𝑃(𝐴) + 𝑃(𝐵) + 𝑃(𝐶) ___ (i)
Exhaustive
𝑃(𝐴 ∪ 𝐵 ∪ 𝐶) = 1 ___ (ii)
Equally Likely
𝑃(𝐴) = 𝑃(𝐵) = 𝑃(𝐶) = 𝐾 (𝑙𝑒𝑡) ___ (iii)
From (i), (ii) & (iii)
K+K+K=1
3K = 1
1
K=
3
1
∴ 𝑃(𝐴) = 𝑃(𝐵) = 𝑃(𝐶) = 3
1 2
𝑃(𝐴′) = 1– =
3 3

www.escholars.in
61
6. A number is selected from the first 30 natural numbers. What is the probability that it will be
divisible by 4 or 7.
Ans. Total outcome (s) = 30
n(A) (divisible by 4) = {4, 8, 12, 16, 20, 24, 28}
n(B) (divisible by 7) = {7, 14, 21, 28}
𝑛(𝐴) 7
P(A) = 𝑛(𝑆) = 30
𝑛(𝐵) 4 2
P(B) = = =
𝑛(𝑆) 30 15
1
𝑃(𝐴 ∩ 𝐵) = [𝑛(𝐴 ∩ 𝐵) = 1]
30
𝑃(𝐴 ∪ 𝐵) (divisible by 4 or 7) = 𝑃(𝐴) + 𝑃 (𝐵)– 𝑃(𝐴 ∩∩ 𝐵)
7 4 1
= + –
30 30 30
10 1
𝑃(𝐴 ∪ 𝐵) = 30
= 3
7. The probability that an Accountant’s job applicant has a B. Com Degree is 0.75, that he is a CA is
0.20 and that he is both B. Com and CA is 0.15 out of 400 applicants, how many would be B.Com
or CA.
Ans. P(B) = 0.75, P(C) = 0.20 and 𝑃(𝐵 ∩ 𝐶) = 0.15
∴ 𝑃(𝐵 ∪ 𝐶) = 𝑃(𝐵) + 𝑃(𝐶)– 𝑃(𝐵 ∩ 𝐶)
= 0.75 + 0.20 – 0.15
= 0.80
∴ Expected frequency = 𝑁 × 𝑃(𝐵 ∪ 𝐶)
= 400 × 0.80
= 320
8. In a group of 18 males and 13 females, 10 males and 6 females are service holders. What is the
probability that a person selected at random from the group is a service holder given that the
selected person is a male.
Ans. Note * Question is based on conditional probability.

18
P(M) = 31
[M → Male, S → Service holder]
10
𝑃(𝑆 ∩ 𝑀) =
31
𝑃(𝑆∩𝑀) 10
∴ 𝑃(𝑆⁄𝑀) = 𝑃(𝑀) = 18 = 0.55
9. The odds in favour of an event is 3:5 and the odd against another event is 5:9. Find the
probability that only one of the two events occurs.
Ans. Let the two events be A and B
3 3
P(A) = 3+5 = 8
9 9
P(B) = =
5+9 14
Since A and B are independent
𝑃(𝐴 ∩ 𝐵) = 𝑃(𝐴) × 𝑃(𝐵)
3 9 27
= × =
8 14 112
Now, Probability that either only A occurs or only B occurs.
P(A–B) + P(B–A)
= [𝑃(𝐴)– 𝑃(𝐴 ∩ 𝐵)] + [𝑃(𝐵)– 𝑃(𝐴 ∩ 𝐵)]
= P(A) + P(B) – 2 𝑃(𝐴 ∩ 𝐵)
www.escholars.in
62
3 9 27
= + –2 ×
8 14 112
3 9 54
= + –
8 14 112
3 9 27
= + –
8 14 56
21+36–27
= 56
30 15
= =
56 28
10. Mr. Prabhat is selected for three posts. For the first post, there are 5 candidates, for the second,
three are 7 candidates and for the third, there are 12 candidates. What is the probability that
Mr. Prabhat would be selected.
Ans. 1
𝑃(𝐴) = , 𝑃(𝐵) = , 𝑃(𝐶) =
1 1
5 7 12
Selected for at least one post.
= 𝑃(𝐴 ∪ 𝐵 ∪ 𝐶)
= 1– 𝑃[(𝐴 ∪ 𝐵 ∪ 𝐶)′]
= 1– 𝑃(𝐴′ ∩ 𝐵′ ∩ 𝐶′)
[De Morgan’s Law]
= 1– 𝑃(𝐴′) × 𝑃(𝐵′) × 𝑃(𝐶′)
[As, A, B and C are independent, so are their compliments]
1 1 1
= 1– (1– 5) × (1– 7) × (1– 12)
4 6 11
= 1– × ×
5 7 12
264
= 1– 420
= 0.37
11. In a business venture, Mr. x can make a profit of ₹40,000 or incure a loss of ₹10,000. The
probability of making profit or incurring loss; from the past experience are known to be 0.65
and 0.15 respectively. What is his expected profit.
Ans. X (Profit) ₹40,000 - ₹10,000
P(Probability) 0.65 0.15

Thus, Expected profit of Mr. x is


E(x) = 𝑃1 𝑥1 + 𝑃2 𝑥2
= 0.65 × 40,000 + 10.15 × (–10,000)
= 26,000 – 1500
= ₹24,500.
12. A random variable 𝑥 has the following probability distribution:
X 0 1 2 3 4 5
P(x) 0 K 2K 3K 4K 75K2

Find the value of K


ii) P(x < 3)
iii) 𝑃(𝑥 ≥ 4)
Ans. i) ∑ 𝑃(𝑥) = 1
∴ 0 + K + 2K + 3K + 4K + 75K2 = 1
75K2 + 10K –1 = 0
75K2 + 15K – 5K – 1 = 0
www.escholars.in
63
15K (5K+1) –1 (5K+1) = 0
(5K+1) (15K–1) = 0
1 1
∴ 𝐾 = 15 [𝐾 ≠– 5]
ii) P(x<3) = P(x=0) + P(x=1) + P(x=2)
= 0 + K + 2K
= 3K
1
= 3 × 15 = 0.2
iii) 𝑃(𝑥 ≥ 4) = P(x=4) + P(x=5)
= 4K + 75K2
1 1
= 4 × 15 + 75 × 225
= 0.27 + 0.33
= 0.6
13. There are 3 boxes with the following composition:
Box I: 9 Red + 7 White + 6 Blue balls.
Box II: 7 Red + 8 White + 5 Blue balls.
Box III: 6 Red + 5 White + 4 Blue balls.
One of the box is selected at random and a ball is drawn from it. What is the probability that the
drawn ball is red.
1
P(B1) = P(BII) = P(BIII) = 3 [any three boxes may be drawn]
Ans. Let A be the event that drawn ball is red.
𝑅 9
𝑃 ( 1⁄𝐵 ) = 22
𝐼
𝑅2 7
𝑃 ( ⁄𝐵 ) =
𝐼𝐼 20
𝑅 6
𝑃 ( 3⁄𝐵 ) =
𝐼𝐼𝐼 15
Thus,
𝑃(𝐴) = 𝑃(𝑅1 ∩ 𝐵𝐼 ) + 𝑃(𝑅2 ∩ 𝐵𝐼𝐼 ) + 𝑃(𝑅3 ∩ 𝐵𝐼𝐼𝐼 )
𝑅 𝑅 𝑅
= 𝑃 ( 1⁄𝐵 ) × 𝑃(𝐵𝐼 ) + 𝑃 ( 2⁄𝐵 ) × 𝑃(𝐵𝐼𝐼 ) + 𝑃 ( 3⁄𝐵 ) × 𝑃(𝐵𝐼𝐼𝐼 )
𝐼 𝐼𝐼 𝐼𝐼𝐼
9 1 7 1 6 1
= 22 × 3 + 20 × 3 + 15 × 3
9 7 6
=66 + 60 + 45
= 0.136 + 0.117 + 0.133
= 0.386
14. Two broad divisions of probability are __________ and _______
Ans. Two broad divisions of probability are
i) Subjective Probability: – Allows the server to gain insight by referencing things they have
learned and their own experience.
ii) Objective Probability: – It is the probability an event will occur based on an analysis in which
each measure is based or recorded observation.
15. The theorem of a compound probability states that for only two events A and B is given by
____________
Ans. For mutually exclusive events
P(A or B) i.e. 𝑃(𝐴 ∪ 𝐵) = P(A) + P(B)
For mutually inclusive events
P(A or B) i.e. 𝑃(𝐴 ∪ 𝐵) = P(A) + P(B) – (𝐴 ∩ 𝐵)

www.escholars.in
64
Theoretical Distribution
✓ A Probability distribution also possesses all the characteristics of an observed distribution. We
define population mean (𝜇), Population median (𝜇̅ ), Population mode (𝜇0 ), Population standard
deviation (𝜎) etc. These characteristics are also known as population parameters. Probability
distribution or continuous probability distribution depending upon the random variable.
✓ Two important discrete probability distributions are a) Binomial distribution b) Poisson
distribution.
✓ Normal distribution is an important continuous probability distribution.
✓ Binomial distribution:
➢ Discrete Probability Distribution, Invented by Bernoulli.
➢ It is used when no. of trials is too large but finite.
➢ Every trial is Independent.
➢ Events should be Mutually exclusive and exhaustive.
➢ Every trial has two outcomes, the occurrence of one will be known as ‘success’ Indicated by
‘P’, and the occurrence of another is known as ‘failure’ represented by q.
nCr (𝑝)𝑟 (𝑞)𝑛−𝑟

𝒏 = 𝑁𝑢𝑚𝑏𝑒𝑟 𝑜𝑓 𝑡𝑟𝑖𝑎𝑙𝑠
𝒓 = 𝑁𝑢𝑚𝑏𝑒𝑟 𝑜𝑓 𝑠𝑢𝑐𝑐𝑒𝑠𝑠 𝑟𝑒𝑞𝑢𝑖𝑟𝑒𝑑
𝒑 = 𝑃𝑟𝑜𝑏𝑎𝑏𝑖𝑙𝑖𝑡𝑦 𝑜𝑓 𝑆𝑢𝑐𝑐𝑒𝑠𝑠 𝑖𝑛 𝑜𝑛𝑒 𝑡𝑟𝑖𝑎𝑙
𝒒 = 𝑃𝑟𝑜𝑏𝑎𝑏𝑖𝑙𝑖𝑡𝑦 𝑜𝑓 𝑓𝑎𝑖𝑙𝑢𝑟𝑒 𝑖𝑛 𝑜𝑛𝑒 𝑡𝑟𝑖𝑎𝑙
It is bi-Parametric.
✓ Properties of binomial distribution
➢ 𝑴𝒆𝒂𝒏 = 𝑛𝑝
➢ 𝒗𝒂𝒓𝒊𝒂𝒏𝒄𝒆 = 𝑛𝑝𝑞
➢ 𝝈𝟐 = 𝑛𝑝𝑞
➢ 𝝈 = √𝑛𝑝𝑞
➢ Variance is always less than mean
➢ Variance will be highest when 𝑝 = 𝑞 = 0.5, and it is skewed to the right.
➢ It can be bi-Modal or uni-Modal. If (n+1) P is an Integer, then it is bi-Modal where two modes
are (n+1) P and (n+1) P-1, and if (n+1) P is a non-integer, then it is Uni-Modal, where the
greatest integer in (n+1) P is the mode.
➢ Additive Property of Bimodal distribution
If X and Y are two independent variables such that
(𝑋~𝐵)(𝑛1 𝑃) 𝑎𝑛𝑑 𝑌~𝐵 (𝑛2 𝑃)
Then (𝑋 + 𝑌)~𝐵(𝑛1 + 𝑛2 , 𝑃)
✓ Poisson Distribution:
A random variable X is defined to follow the Poisson distribution. It is denoted by 𝑋~𝑃(𝑚) if the
probability mass function of 𝑥 is given by.
𝑒 −𝑚 ×𝑚𝑟
𝐹(𝑥) = 𝑃(𝑋 = 𝑟) = 𝑟! 𝑓𝑜𝑟 𝑟 = 0, 1, 2, …… ∞
➢ It is a discrete random variable invented by Simon Denis Poisson.
➢ It is Uni-Parametric distribution as it is characterized by only one Parameter m.
➢ It is used when no. of trials (n) are too large (tends to infinite) and the probability of success
is very small (tends to zero)
➢ Trials are Independent.
✓ Properties of Poisson distribution:
Mean = m 𝒎 = 𝑛𝑝
Variable =(𝜎)2 = 𝑚
𝝈 = √𝑚
➢ It can also be Uni-modal or Bi-Modal. If m is an integer, then it is Bi-Modal, Where m and m –
1 are modes. And it is Uni-Modal if m is non-Integer. Where greatest Integer in m is the mode
➢ Additive Property = If x and y are two Independent variables such that X ~ P(𝑚1 ) and Y ~ P
(𝑚2)then (𝑥 + 𝑦)~𝑃(𝑚1 + 𝑚2 )

www.escholars.in
65
➢ Applications or examples – No. of Printing mistakes per page on large book, no. of the accident
on the busy road per minute, no. of demand per minute for a health care.
➢ It is symmetrical When the mean value is high.
✓ Normal or Gaussian distribution:
➢ Continuous Probability distribution. Most important and universally accepted. It is based on
true parameters 𝜇 𝑎𝑛𝑑 𝜎 2. Denoted by 𝑥~𝑁 (𝜇, 𝜎 2 ).
➢ If the Probability density function of the random variable X is given by
−1 𝑥−𝜇 2
1 ( )
𝑭(𝒙) = −𝑒2 𝜎
𝜎√2𝜋
➢ 𝝈 = 𝑆. 𝐷., 𝝁 = 𝑀𝑒𝑎𝑛, 𝒙 = 𝑟 =No. of success required
➢ Where 𝝁 and 𝜎, are constants, and 𝜎 > 0
✓ Properties of the normal distribution:
➢ Under normal distribution, mean = median = mode. At mean, the Probability is highest
➢ The mean of the normal distribution is given by 𝝁.
➢ Mean deviation from mean = Mean deviation from median = Mean deviation from mode =
0.8 𝜎 = 𝝈√𝟐.
➢ Quartiles
First Quartile = 𝜇 − 0.675 𝜎
Third Quartile = 𝜇 − 0.675 𝜎
𝑄 −𝑄
Quartile deviation = 3 2 1 = 0.675𝜎
➢ Point of Inflexion = (𝜇 − 𝜎), (𝜇 + 𝜎)
➢ The normal distribution is symmetrical about 𝒙 = 𝝁. As such, its skewness is zero, i.e., the
normal curve is neither inclined to move towards the right (negatively skewed) nor towards
the left (positively skewed).
➢ We note that (𝜇 ± 𝜎) 𝑐𝑜𝑣𝑒𝑟 68.27% 𝑎𝑟𝑒𝑎, (𝜇 ± 2𝜎) 𝑐𝑜𝑣𝑒𝑟 95.45% 𝑎𝑟𝑒𝑎, 𝑎𝑛𝑑
(𝜇 ± 2𝜎) 𝑐𝑜𝑣𝑒𝑟 99.73% 𝑎𝑟𝑒𝑎,
➢ If x and y are independent normal variables with meant and standard deviations as 𝑢1
and𝑢2 and 𝜎1 and 𝜎2 respectively, then (𝒁 = 𝒙 + 𝒚) also follows a normal distribution with
mean (𝝁𝟏 + 𝝁𝟐 ) and S.D = √𝜎1 2 + 𝜎2 2 Respectively.

Q. Priority Questions
No.
1. What are the parameters of binomial distribution?
Ans. Binomial Distribution is a bi- parametric distribution, characterized by ‘n’ and ‘p’ [n represents
number of independent trials and p represents probability of success].
2. If standard deviation of a poisson distribution is 2, then its modes are_______
Ans. Given 𝜎 = S. D. = 2 ⇒ Variance = 𝜎 2 = 4
∵ In poisson distribution
Mean = Variance
∵ m = 4, which is an integer
∵ it is bi–modal
Modes are m and (m–1)
hence, 4 and 3,
3. If the inflexion points of a Normal Distribution are 6 and 14. Find its Standard Deviation.
Ans. ∵ The inflexion points of a Normal Distribution are given as (+) and (–)
here, we are given :
 +  = 14. (1)
and,  –  = 6 (2)
Solving (1) and (2) we get

www.escholars.in
66
 = 10 and  = 4
Hence S.D () = 4
4. For binomial distribution variance and mean will be ________
Ans. For Binomial distribution
npq < np
Variance < Mean (n represents number of trials, p represents probability of success and q
represents probability of failure)
5. An approximate relation between quartile deviation (QD) and standard deviation (S.D) of
normal distribution is:
Ans. We known that
In normal distribution
4 S.D = 5 M. D = 6 Q.D
So 4 S.D = 6 Q.D
2 S.D = 3 Q.D
or 3 Q.D = 2 S.D
6. In a certain Poisson frequency distribution, the probability corresponding to the successes is
half the probability corresponding to three successes. The mean of the distribution is
Ans. Given
1
P(x=2) = P(x=3)
2
2 P(x=2) = P(x=3)
𝑒 –𝑚 .𝑚2 𝑒 –𝑚 .𝑚3
2. =
2! 3!
2 𝑚
=
2 6
2
𝑚 =6×2=6
7. If six coins are tossed simultaneously. The probability of obtaining exactly two heads are:
Ans. Here Total trial (n) = 6
For coin p = 1⁄2, q = 1 –1⁄2 = 1⁄2
P(X=x) = nCx px.qn–x
1 2 1 6–2
P(X=2) = 6C2 (2) × (2)
6×5 1 2 1 4
= 2×1 × (2) × (2)
1 2+4
= 15×( )
2
1 6
= 15 × (2)
15
=( )
64
8. Area under 𝑈 ± 3𝜎
Ans. We know that 99.73 per cent of the values of a normal variable lies between (u–3) and (u+3)
Thus probability that a value of x lies. Outside the limit is as low as (100–99.73) = 0.27%
9. What is the SD and mean
√2 –2(𝑥–3)2 ,–∞<𝑥<∞.
x if 𝑓(𝑥) = 𝑒
√𝜋
Ans. The standard form of probability density function is
–(𝑥–𝜇)2
1
𝑓(𝑥) = .𝑒 2𝜎2 ---------------------- (1)
√2𝜋

www.escholars.in
67
2 2
Here, √ . 𝑒 2(𝑥–3)
𝜋
13 2
2 ( )
= √𝜋 . 𝑒 1/2

on comparing with Equation --------------- (1)


1
2𝜎 2 = 𝑈 = 3
2
1
𝜎2 =
4
1
𝜎=
2
1
So SD = , mean = 3
2
10. Which of the following is uni–parametric distribution?
Ans. Poisson distribution is uni-parametric distribution. (Poisson’s distribution is characterized by
single parameter m called mean)
11. If x is a binomial variate with P = 1/3, for the experiment of 90 trials, then the standard
deviations is equal to:
Ans. P if x ~ B(n,p)
Here n = 90, p = 1/3, q = 1 – p
1
=1–
3
2
=
3
S.D = √𝑛𝑝𝑞
1 3
= √90 × 3 × 3

= √20
S.D = 2√5
12. For a binomial distribution, there may be _____ mode
Ans. For a binomial distribution, there may be multimode. In a binomial distribution the probability
of success is constant and the number of trials is fixed. Therefore the distribution can have one
or more modes depending on the values of parameter.
13. Skewness of Normal Distribution is:
Ans. Skewness of Normal Distribution is zero because Normal Curve is Symmetrical.
14. The speeds of a number of bikes follow a normal distribution model with a mean of 83 km/hr
and a standard deviation of 9.4 km./hr. Find the probability that a bike picked at random is
travelling at more than 95 km/hr.? [P(Z>1.28) = 0.1003]
Ans. Mean (M) = 83, S.D (𝜎) = 9.4
𝑥–𝑀 95–83
P(x>95) = 𝑃 [ > ]
𝜎 9.4
= P(Z>1.28)
= 0.1003
15. In a normal distribution about 95 per cent of the observations lie between _____________ and
___________.
Ans. In a Normal distribution about 95% of the observations lies between  – 1.96  and  + 1.96.
16. T–test can be used only when the sample has been taken from
Ans. T–test can be used only when the sample has been taken from Normal Population. The t test has
assumption regarding the distribution of data within each group, rather than the entire
population. The key assumption is that the data within each group follows a normal distribution.

www.escholars.in
68
Correlation and Regression
✓ Correlation shows association or relation between two variables, whereas regression shows the
value of the variable based on other Bivariate data.
➢ Bivariate Data are the data collected for two variables irrespective of time.
➢ Can be Marginal and conditional distribution.
➢ Collected for 2 variables at the same time.
➢ For distribution p + q, a number of cells are pq.
➢ Some cells may be 0.
➢ For p × q, marginal distribution is 2, and conditional are p + q.
✓ Correlation: The change in one variable is Reciprocated by a corresponding change in the other
variable either directly or inversely; then the two variables are known to be associated or correlated.
Correlation analysis aim is establishing relation between two variables and measuring the extent of
relation between two variable.
✓ There are two types of correlation:
a) Positive correlation b) Negative correlation
The value of correlation is between –1 to 1. -1 means perfect negative, 0 to -1 means negatives, 0
means no correlation, while 0 to 1 means positive and +1 means perfect positive correlation.
✓ Scatter diagram: This is a simple diagrammatic method to establish a correlation between a pair of
variables, and it is used for linear and non-linear (Curvilinear) distribution.
➢ If the plotted points in a scatter diagram lie from upper left to lower right, then the correlation
is negative and vice versa correlation is positive.
➢ If all the potted points in a scatter diagram are evenly distributed, then the correlation is zero
and plotted points lie on the single line then the correlation is either positive or negative.
✓ Karl Pearson's Product moment correlation coefficient:
𝑐𝑜𝑣.(𝑥, 𝑦)
r = 𝑟𝑥𝑦 =
𝜎𝑥 × 𝜎𝑦
∑(𝑥−𝑥̅ )(𝑦−𝑦̅) ∑ 𝑥𝑦
Where Cov (𝑥, 𝑦) = 𝑁
or Cov. (𝑥, 𝑦)= 𝑛
- 𝑥̅ 𝑦̅
∑(𝑥−𝑥̅ )2 ∑ 𝑥2
𝜎𝑥 = √ 𝑁
or 𝜎𝑥 = √ 𝑁
− (𝑥̅ )2
∑(𝑦−𝑦̅)2 ∑ 𝑦2
𝜎𝑦 = √ or 𝜎𝑦 = √ − (𝑦̅)2
𝑁 𝑁
𝑛 ∑ 𝑥𝑦− ∑ 𝑥.∑ 𝑦
r= (Direct Method)
√𝑛 ∑ 𝑥 2 − (∑ 𝑥)2 √𝑛 ∑ 𝑦 2 − (∑ 𝑦)2
𝑛 ∑ 𝑑𝑥 𝑑𝑦− ∑ 𝑑𝑥×∑ 𝑑𝑦
r=
√𝑛 ∑ 𝑑𝑥 2 − (∑ 𝑑𝑥)2 √𝑛 ∑ 𝑑𝑦 2 − (∑ 𝑑𝑦)2
➢ The coefficient of correlation is a unit free measure.
➢ The coefficient of correlation is unaffected by change of origin or scale, but it changes its sign
with the change of sign of variables. If the sign of both variables is the same, r remains the
same. While if sign differs r sign also changes.
➢ The coefficient of correlation always is between – 1 and 1, including both limiting values.
➢ Spurious correlation means the correlation between two variables has no causal relation.
➢ Product moment correlation coefficient is considered for finding the nature of correlation and
the amount of correlation.
➢ Product moment correlation coefficient may be defined as the ratio of covariance between the
variable to the product of their standard deviations.
✓ Spearman’s rank correlation:
For finding the correlation between two attributes we consider it.
6 ∑ 𝑑2
𝒓𝑹 = 1 −
𝑛(𝑛2 − 1)
Where 𝑟𝑅 denotes rank correlation coefficient, and it lies between -1 and 1 inclusive of these two
values. 𝑑 = 𝑅𝑥 − 𝑅𝑦 . The 𝑅𝑥 𝑎𝑛𝑑 𝑅𝑦 are Rankings given to 𝑥 𝑎𝑛𝑑 𝑦 series. Ranks are given in
descending order. 1 is given to highest and so on.
In case the same Ranks are received by individuals, then

www.escholars.in
69
2 ∑ 𝑚3 −𝑚
6[∑ 𝑑 + ]
12
𝒓𝑹 = 1 − 𝑛(𝑛2 −1)
m represents the number of repetitions. (𝑚3 − 𝑚) will come under number of times numbers are
repeated.
✓ Coefficient of concurrent deviations:
It is the quickest method to find correlation between two variables.
(2𝐶−𝑀)
𝒓𝒄 = ±√±
𝑀
If (2𝑐 − 𝑚) > 0, then we take the positive sign both inside and outside the radical sign, and if (2𝑐 −
𝑚) < 0, we are to consider the negative sign both inside and outside the radical sign.
C = no. of positive signs
𝒎 = 𝑛– 1
n = no. of observation
✓ Regression:
In regression analysis, we are concerned with the estimation of one variable for a given value of
another variable or establishing a mathematical relationship between two variables and predicting
the value of the dependent variable for a given value of the independent variable. The method applied
for deriving the regression equations is knowns as least square method.
If 𝒚 = 𝑎 + 𝑏𝑥, a and b are regression parameters, regression equation 𝑦 𝑜𝑛 𝑥, 𝑏𝑦𝑥 methods based on
least square. Regression coefficient also represents the shape of regression equations.
𝐶𝑜𝑣.(𝑥,𝑦) 𝑠
𝒃𝒚𝒙 = 2 , 𝑏𝑦𝑥 = r. 𝑦
𝑠𝑥 𝑠𝑥
𝐶𝑜𝑣.(𝑥,𝑦) 𝑠𝑥
𝒃𝒙𝒚 = , 𝑏𝑥𝑦 = r.
𝑠𝑦 2 𝑠𝑦
𝒂 = 𝑦 − 𝑏𝑥 [where 𝑦 = 𝑚𝑒𝑎𝑛 𝑜𝑓 𝑦, 𝑥 = 𝑚𝑒𝑎𝑛 𝑜𝑓 𝑥] for solving 𝑎 𝑎𝑛𝑑 𝑏
∑ 𝒚 = 𝑛𝑎 + 𝑏 ∑ 𝑥
∑ 𝒙𝒚 = 𝑎 ∑ 𝑥 + 𝑏 ∑ 𝑥 2
Direct Method
𝑛 ∑ 𝑥𝑦− ∑ 𝑥.∑ 𝑦
𝒃𝒚𝒙= ∑ 2 (∑ 2
𝑛 𝑥 − 𝑥)
For solving equation 𝑏𝑥𝑦 replace 𝑥 𝑤𝑖𝑡ℎ 𝑦.
𝑛 ∑ 𝑥𝑦 − ∑ 𝑥. ∑ 𝑦
𝒃𝒙𝒚 =
𝑛 ∑ 𝑦 2 − (∑ 𝑦)2
The regression coefficients remain unchanged due to a shift of origin but change due to a shift of
scale.
This property states that if the original pair of variables is (𝑥, 𝑦) and if they are changed to the pair
(𝑢, 𝑣) under:
𝒖 = 𝑎 + 𝑏𝑥
𝒗 = 𝑐 + 𝑑𝑦
𝑏
𝒃𝒖𝒗 = 𝑑 × 𝑏𝑥𝑦
𝑑
𝒃𝒗𝒖 = 𝑏 × 𝑏𝑦𝑥
➢ The regression equations are interested in their means.
➢ The coefficient of correlation between two variables 𝑥 𝑎𝑛𝑑 𝑦 is the simple G.M of the two-
regression coefficients. The sign of the correlation coefficient would be the common sign of
the two-regression coefficients.
r = ± √𝑏𝑦𝑥 × 𝑏𝑥𝑦
➢ Correlation coefficient measuring a linear relationship between the two variables indicate the
amount of variation of one variable accounted for by the other variable. A better measure for
this purpose is provided by the square of the correlation coefficient known as the coefficient
of determination. This can be interpreted as the ratio between explained variance to total
variance.
𝐸𝑥𝑝𝑙𝑎𝑖𝑛𝑒𝑑 𝑣𝑎𝑟𝑖𝑎𝑛𝑐𝑒
𝒓𝟐 =
𝑡𝑜𝑡𝑎𝑙 𝑣𝑎𝑟𝑖𝑎𝑛𝑐𝑒
➢ Coefficient of non-determination = 1 – 𝑟 2
www.escholars.in
70
➢ Two regression lines coincide when 𝑟 = −1 or 1. And perpendicular if r = 0.
➢ Product of regression coefficient must be numerically less than 1.
✓ Probable error:
➢ It is a method of obtaining the correlation coefficient of population. It is defined as –
1−𝑟 2
➢ P.E = 0.6745 ×
√𝑁
➢ S.E = Standard error of correlation coefficient
1−𝑟 2
➢ S.E =
√𝑁
➢ The limit of correlation coefficient = r ± P.E.
= r > 6 P.E. = Presence of correlation is certain.
= r < P.E. = No evidence of correlation.

Q. Priority Questions
No.
1. For n number of towns, the coefficient of rank correlation between the people living below
the poverty line and increase of population is 0.50. If the sum of squares of the differences in
ranks awarded to these factors is 82.50, find the number of towns. If the number of towns is
increased by 12, find r.
Ans. Given
r = 0.50, ∑ D2 = 82.50
6 ∑ D2
r=1–
n(n2 –1)
6×82.50
0.50 = 1 – n(n2–1)
n(n2–1) = 990
Now, check with option.
a) n = 10
L.H.S
10(102–1) = 990
We know,
6 ∑ D2
r = 1 – n(n2 –1)
6×82.50
r = 1 – 12(122 –1)
r = 0.71
2. For the variables x and y, the regression equations are given as 7x–3y–18 = 0 and 4x – y – 11
=0
i) Find the arithmetic means of x and y
ii) Identify the regression equation of y on x
iii) Compute the correlation coefficient between x and y
iv) Given the variance of x is 9. Find the S.D. of y.
Ans. i) 7𝑥– 3𝑦– 18 = 0
4𝑥– 𝑦– 11 = 0
Solving these two equations, we get 𝑥 = 3 and 𝑦 = 1
ii) Let, assume that 7x–3y–18=0 represents the regression line y on x and 4x–y–11 = 0
represents the regression line x on y.
Now,
7𝑥 – 3y – 18 = 0 4𝑥 – y – 11 = 0
7 11 1
y = (–6) + 3 𝑥 𝑥= 4
+4𝑦
7 1
𝑏𝑦𝑥 = 𝑏𝑥𝑦 =
3 4
www.escholars.in
71
 r2 = 𝑏𝑦𝑥 × 𝑏𝑥𝑦
7 1 7
=3×4 = 12
<1
Since, 𝑟 ≤ 1 ⇒ 𝑟 2 ≤ 1, our assumptions are correct. Thus 7𝑥– 3𝑦– 18 = 0 truly
represents the regression line y on x
7
iii) Since r2 = 12

 r = √7⁄12 = 0.7638
𝑆𝑦
iv) 𝑏𝑦𝑥 = 𝑟 × 𝑆𝑥
7 𝑆𝑦
3
= 0.7638 𝑆𝑥
7×3
 Sy (𝜎𝑦 ) =
0.7638×3
= 9.1647.

3. If r = 0.7 and n = 64 find out the probable errors, standard error of the coefficient of
correlation and determine the limits for the population correlation coefficient.
Ans. r = 0.7; n = 64.
1–𝑟 2 1–𝑟 2
P.E. = 0.6745 × S.E =
√𝑛 √𝑛
1–(0.7)2 1–(0.7)2
= 0.6745 × =
√64 √64
= 0.6745 × 0.06375 = 0.06375
= 0.043
Limits for the population Correlation coefficient = r ± P. E.
= 0.7 ± 0.043
= 0.743, 0.657
4. If the relationship two variables x and u is 𝑢 + 3𝑥 = 10 and between two other variables y
and v is 2𝑦 + 5𝑣 = 25, and the regression coefficient of y on x is known as 0.80, what would
be the regression coefficient of v on u?
Ans. u + 3x = 10
𝑥–10⁄3
u= –1⁄3
and 2y + 5v = 25
𝑦–25⁄2
v=
–5⁄2
We know,
Coefficient of v
by x = × 𝑏𝑣𝑢
Coefficient of u
–5⁄
0.80 = –1 2 × 𝑏𝑣𝑢
⁄3
2 8
𝑏𝑣𝑢 = × 0.80 =
15 75
5. If r = 0.6, then the coefficient of determination and non–determination will be.
Ans. Coefficient of determination = r2 = (0.6)2 = 0.36.
Coefficient of non–determination = 1–r2
= 1–0.36 = 0.64
6. If 𝑥, 𝑦 denote the arithmetic means, 𝜎𝑥 and 𝜎𝑦 denotes the standard deviations 𝑏𝑥𝑦 and 𝑏𝑦𝑥
denote the regression coefficients of the variables ‘X’ and ‘Y’ respectively, then the point of
intersection of regression lines 𝑥 𝑜𝑛 𝑦 𝑎𝑛𝑑 𝑦 𝑜𝑛 𝑥 𝑖𝑠.

www.escholars.in
72
Ans. Since the two lines of regression pass through the point (𝑥, 𝑦), the mean values (𝑥, 𝑦) can be
obtained as the point of intersection of the two regression lines.
7. Determine the coefficient of co–relation between x and y series.
X Series Y Series
No. of items 15 15
Arithmetic Mean 25 18
Sum of square of Deviations from Mean. 136 138

Sum of product of Deviations of x and y series from mean = 122


Ans. Given,
N = 15, 𝑋 = 25, 𝑌 = 18, ∑ dx 2 = 136, ∑ dy 2 = 138
∑ d𝑥d𝑦 = 122.
Coefficient of Correlation
∑ d𝑥d𝑦 122
r= = = 0.89
√∑ dx2 ∑ dy2 √136×138
8. Compute the correlation coefficient between x and y from the following data.
n = 10, ∑ d𝑥𝑦 = 220, ∑ x 2 = 200, ∑ 𝑦 2 = 262, ∑ 𝑥 = 40 𝑎𝑛𝑑 ∑ 𝑦 = 50
Ans. r=
𝑛 ∑ 𝑥𝑦 – ∑ 𝑥 ∑ 𝑦
√𝑛 ∑ 𝑥 2 –(∑ 𝑥)2 ×√𝑛 ∑ 𝑦 2 –(∑ 𝑦)2
10×220–40×50
=
√10×200–(40)2 √10×262–(50)2
2200–2000
= = 0.91
20×10.9545
 Positive correlation between x and y variable is 0.91
Alternative method: –
∑𝑥 40 ∑𝑦 50
𝑥= = =4 𝑦= = =5
𝑛 10 𝑛 10
∑ 𝑥𝑦
Cov (𝑥, 𝑦) = 𝑛
– 𝑥𝑦
220
= –4×5=2
10
∑ 𝑥2 ∑ 𝑦2
𝜎𝑥 = √ – (𝑥)2 𝜎𝑦 = √ – (𝑦)2
𝑛 𝑛

200–160 262–250
=√ =2 =√ = 1.0954
10 10
Now,
𝑐𝑜𝑣 (𝑥,𝑦) 2
r= 𝜎𝑥 𝜎𝑦
= 2×1.0954 = 0.91
9. Given that the correlation coefficient between x and y is 0.8, write down the correlation
coefficient between u and v where
i) 2u + 3x + 4 = 0 and 4v + 16y + 11 = 0
ii) 2u – 3x + 4 = 0 and 4v + 16y + 11 = 0
iii) 2u – 3x + 4 = 0 and 4v – 16y + 11 = 0
iv) 2u + 3x + 4 = 0 and 4v – 16 y + 11 = 0
Ans. bd
rxy = |b||d| ruv.
i.e. rxy = ruv if b and d are of same sign and ruv = –rxy when b and d are of opposite signs, b and
d being the scales of x and y respectively. In (i) u = –2 + (–3/2) 𝑥 and v = (–11/4) + (–4) 𝑦
–3
since b = 2
and d = –4 are of same sign, the correlation coefficient between u and v would be
the same as that between x and y i.e. rxy = ruv = 0.8.

www.escholars.in
73
In (ii) u = –2 + 3/2 𝑥 and v = (–11/4) + (–4) 𝑦. Hence b = 3/2 and d = –4 are of opposite signs
and we have ruv = –rxy = –0.8
Proceeding in a similar manner, we have ruv = 0.8 and – 0.8 in (iii) and (iv)
10. While computing rank correlation coefficient between profits and investments for 10 years
of a firm, the difference in a rank for a year was taken as 7 instead by 5 by mistake and the
value of rank correlation coefficient was computed as 0.80. What would be the correct value
of rank correlation coefficient after rectifying the mistake.
Ans. Given r = 0.80 wrong D = 7 should be replaced by 5
6 ∑ 𝐷2 6 ∑ 𝐷2
r = 1 – 𝑁(𝑁2 –1) ⇒ 0.80 = 1– 𝑁(𝑁2 –1) ⇒ ∑ 𝐷 2 = 33
Corrected ∑ 𝐷 2 = 33 – 72 + 52 = 9
6×9
 rectified r = 1–10×(102 –1) = 0.95
11. Find coefficient of correlation, if n = 7, D2 = 44.50 m1 = 2, m2 = 2 and m3 = 3.
Ans. 𝑟 = 1 – 6 [∑ 𝐷 2 +
∑(𝑚3 –𝑚)
]
12
∑(𝑚3 –𝑚) (23 –2)+(23 +3)+(33 –3)
{ = = 3}
12 12
6(44.50+3)
= 1–
7(72 –1)
= 0.15
12. The following data relate the expenditure or advertisement in thousands of rupees and the
corresponding sales in lakhs of rupees:
Expenditure on Ad: 8 10 10 12 15
Sales: 18 20 22 25 28
Find an appropriate regression equation.
Ans. Since, Sales (y) depend on advertisement (x), the appropriate regression equation is of
𝑦 𝑎𝑛𝑑 𝑥 i.e. of sales on advertisement.
𝑛 ∑ 𝑥𝑦 – ∑ 𝑥× ∑ 𝑦
b=
𝑛 ∑ 𝑥 2 –(∑ 𝑥)2
5×1284–55×113 205
= = = 1.4643
5×633–(55)2 140
a = 𝑦– 𝑏𝑥
113 55
= 5
–1.4643× 5
= 22.60–16.1073
= 6.4927
Thus regression line of y on x is
𝑦 = 6.4927 + 1.4643𝑥
[𝑦 = 𝑎 + 𝑏𝑥]
13. If there is constant increase, constant decrease, then the corresponding graph will be
__________, what if r = 0.
Ans. If there is constant increase in a series, then the corresponding graph will be straight line from
lower left to upper right.
If there is constant decrease in a series then the corresponding graph will be straight line from
upper left to lower right.
If r (correlation coefficient) = 0, there is no correlation (non–sense) between two variables.
14. If concurrent coefficient is 1⁄ . If the sum of deviation is 6 for n pairs of data, find n.
√3

www.escholars.in
74
Ans. Given
1
Concurrent coefficient (𝜎𝑐 ) =
√3
Sum of deviation (c) = 6
2𝐶–𝑚
 Coefficient of concurrent deviation (c) = ± √ 𝑚

1 12–𝑚
= ±√ 𝑚
√3
1 12–𝑚
On squaring both sides 3 = 𝑚
(number of pairs of deviation) m = 9
n=m+1
= 9 + 1 = 10
15. Which of the following is spurious correlation?
a) Correlation between two variables having no casual relationship
b) Negative Correlation
c) Bad relation between two variables
d) Very low correlation between two variables.
Ans. Correlation between two variables having no casual relationship. There is existence of third
variable that affects both the variables.

www.escholars.in
75
Index Number
✓ It means ratio or average of ratios expressed as a percentage, having two or more time periods; one
of the time periods is base period.
✓ A series of numerical figures which show the relative position is called index number.
✓ Issues involved in index numbers:
➢ Selection of data;
➢ Base period;
➢ Selection of weights (weights play a very important part in the construction of index number);
➢ Use of averages (GM is particularly suitable for the construction of index number);
➢ Choice of variables;
➢ Selection of formula;
✓ There are three types of index numbers:
➢ Price index
➢ Quantity index
➢ Value index
✓ Price index numbers:
∑𝑃
➢ Simple Aggregative Price Index = ∑ 𝑃𝑛 × 100
0
𝑃1
∑( )
𝑃0
➢ Simple Average Relative = 𝑁 × 100
➢ Laspeyres Index: In this index, base year quantity is used as weights, or we can say base year
quantities:
∑𝑃 𝑄
➢ Laspeyres index = ∑ 𝑃𝑛𝑄0 × 100
0 0
➢ Paasche’s Index: In this index, current year quantities are used as weights, or we can say
current year quantities:
∑𝑃 𝑄
➢ Paasche’s Index = ∑ 𝑛 𝑛 × 100
𝑃0 𝑄𝑛
➢ The Marshal-Edgeworth index uses this method by taking the average of the base year and
the current year.
∑ 𝑃𝑛 (𝑄0 +𝑄𝑛)
Marshal-Edgeworth Index = ∑ 𝑃 (𝑄0 +𝑄𝑛)
× 100
0
𝐿+𝑃
➢ Dorbish and Bowley’s index number 𝑷𝟎𝟏 = (L = Laspeyres and P =Paasche’s index
2
number)
➢ Fisher’s ideal price index: This index is the geometric mean of Laspeyres and Paasche’s
∑ 𝑃𝑛 𝑄0 ∑ 𝑃𝑛 𝑄𝑛
Fisher’s Index = √ ∑ ×∑ × 100
𝑃0 𝑄0 𝑃0 𝑄𝑛
∑ 𝑃𝑛 𝑄0
➢ Weighted Average of Relative method = ∑ 𝑃0 𝑄0
× 100
Laspeyres Index number + Paasche′ s Index number
➢ Bowley’s Index number = 2
Link Relative of Current Year × Chain Index of Previous Year
➢ Chain Index = 100
𝑃
➢ Link Relative of Current Year = 𝑃1 × 100
0
𝑀𝑜𝑛𝑒𝑦 𝑤𝑎𝑔𝑒
➢ Real wage = × 100
𝑃𝑟𝑖𝑐𝑒 𝐼𝑛𝑑𝑒𝑥
✓ Quantity Index Numbers:
∑𝑄
➢ Simple Aggregate of Quantities = ∑ 𝑛 × 100
𝑄0
𝑄1
∑( )
𝑄0
➢ Simple Average of Quantity Relatives = × 100
𝑁
➢ Weighted Aggregate Quantity Indices:
∑𝑄 𝑃
▪ Laspeyres Index = ∑ 𝑄𝑛𝑃0 × 100
0 0
∑𝑄 𝑃
▪ Paasche’s Index = ∑ 𝑄𝑛𝑃𝑛 × 100
0 𝑛

www.escholars.in
76
∑𝑄 𝑃 ∑𝑄 𝑃
▪ Fisher’s Ideal Index = √ ∑ 𝑄𝑛𝑃0 × ∑ 𝑄𝑛𝑃𝑛 × 100
0 0 0 𝑛
✓ Value Index Numbers:
𝑉 ∑𝑃 𝑄
➢ Value Index = 𝑉𝑛 = ∑ 𝑃𝑛 𝑄𝑛
0 0 0
𝐶𝑢𝑟𝑟𝑒𝑛𝑡 𝑉𝑎𝑙𝑢𝑒
➢ Deflated Value (Real Value) =
𝑃𝑟𝑖𝑐𝑒 𝑖𝑛𝑑𝑒𝑥 𝑓𝑜𝑟 𝑐𝑢𝑟𝑟𝑒𝑛𝑡 𝑦𝑒𝑎𝑟
OR
𝐵𝑎𝑠𝑒 𝑃𝑟𝑖𝑐𝑒
Deflated Value = Current Value × 𝐶𝑢𝑟𝑟𝑒𝑛𝑡 𝑃𝑟𝑖𝑐𝑒
➢ Value = Price × Quantity
➢ Current Value = 𝑃𝑛 𝑄𝑛
𝑂𝑟𝑖𝑔𝑖𝑛𝑎𝑙 𝑃𝑟𝑖𝑐𝑒 𝑖𝑛𝑑𝑒𝑥
➢ Shifted Price Index = × 100
𝑃𝑟𝑖𝑐𝑒 𝑖𝑛𝑑𝑒𝑥 𝑜𝑓 𝑡ℎ𝑒 𝑦𝑒𝑎𝑟 𝑜𝑛 𝑤ℎ𝑖𝑐ℎ 𝑖𝑡 ℎ𝑎𝑠 𝑡𝑜 𝑠ℎ𝑖𝑓𝑡𝑒𝑑
✓ Test of Adequacy:
➢ Unit Test: This test Requires that the formula should be independent of the unit in which
price and quantities are quoted. Except for the simple Aggregative index, all other formulae
satisfy this test.
➢ Time Reversal Test: It is a test to determine whether the method will work both ways in
time forward and backwards. The test checks the index number should be such that two
ratios, the current on the base and base on the current. So, the two indexes should be
reciprocals of each other.
𝑃01 × 𝑃10 = 1
Laspeyres and Paasche’s do not satisfy this test, but fisher’s ideal formula satisfy this test.
This test is necessary to check the consistency of the index number.
➢ Factor Reversal Test: This holds when the product of price index and the quantity index
should be equal to the corresponding value index,
∑𝑃 𝑄
i.e., = ∑ 𝑃1 𝑄1 × 100
0 0
Symbolically
𝑃01 × 𝑄01 = 𝑉01
Fisher’s Index Satisfy Factor Reversal test. Because fisher’s index number satisfy the time-
reversal and factor reversal test, so, it is called an ideal index number.
➢ Circular Test: It is concerned with the measurement of price changes over a period of the
year when it is desirable to shift the base.
𝑃01 × 𝑃12 × 𝑃20 = 1
➢ It is a simple geometric mean of price relatives.
➢ Paasche’s, Laspeyres and Fisher’s index number do not satisfy the test.
➢ Simple aggregative method and the fixed weight aggregative method meet this test.
➢ It is an extension of the time reversal test.
➢ It is concerned with the measurement of price changes over a period of years, when it is
desirable to shift the base.
✓ Point to Remember:
➢ Purchasing power of money is the reciprocal of price index number.
➢ Index numbers for the base period are always taken as 100.
➢ Index numbers show the percentage changes rather than absolute amounts of change.
➢ Geometric mean makes the index number time-reversible.
➢ Circular test is an extension of the time-reversal test.
➢ Weighted Geometric mean of the relative formula satisfies the time-reversal test.
➢ The index number is a special type of average.
➢ The AM of the group of given general index.
➢ The choice of a suitable base period is the best temporary solution.
➢ There are 4 tests of adequacy.
➢ Index number is equal to average of price relative.
➢ The value at the base time period serves as the standard point of comparison.
➢ Base period is a point of reference in comparing various data describing individual behaviour.
www.escholars.in
77
Q. Priority Questions
No.
1. If ∑ 𝑃0𝑄0 = 114, ∑ 𝑃0 𝑄1 = 138 , ∑ 𝑃1𝑄0 = 95, ∑ 𝑃1 𝑄1 = 115, then Laspeyre’s, Paasche’s
Fisher’s, and Dorbish Bowley’s index number.
Ans. 1) Laspeyre’s Index Number = ∑
∑ 𝑃1 𝑄0
× 100 =
95
× 100
𝑃0 𝑄0 114
= 83.33
∑ 𝑃1 𝑄1 115
2) Paasche’s Index Number = ∑ 𝑃 𝑄 × 100 = 138
× 100
0 1
= 83.33
∑ 𝑃1 𝑄0 ∑ 𝑃1 𝑄1
3) Fisher’s Index Number = √∑ ×∑ × 100
𝑃0 𝑄0 𝑃0 𝑄1

95 115
=√ × × 100
114 138
= 83.33
∑ 𝑃1 𝑄0 ∑ 𝑃1 𝑄1
+
∑ 𝑃0 𝑄0 𝑃0 𝑄1
4) Dorbish Bowley’s Index Number = 2
× 100
95 115
+
= 114 138
× 100
2
= 83.33
2. Fisher’s Index number is based on __________
Ans. Fisher’s Index number is based on geometric mean of Laspeyre’s and Paasche’s index.
Note
Laspeyre’s Index number is based on base year quantity (𝑄0 )
Paasche’s index number is based on current year quantity (𝑄1 )
Dorbish Bowley’s Index number is based on arithmetic mean of Laspeyre’s and Paasche’s
index number.
3. Net monthly salary of an employee was ₹4,000. The consumer price index number in 2XX2 is
250 with 2XX1 as base year. If he has to be rightly compensated then additional dearness
allowance to be paid to the employee is _______
Ans. Let base year price index (2XX1) be 100.
Current year price index (2XX2) is 250.
Net monthly salary in base year (2XX1) is 4000.
∴ When the consumer price index was 250 in 2XX2, then monthly salary will be.
𝑂𝑙𝑑 𝑖𝑛𝑑𝑒𝑥 𝑁𝑢𝑚𝑏𝑒𝑟 𝑁𝑒𝑤 𝑖𝑛𝑑𝑒𝑥 𝑁𝑢𝑚𝑏𝑒𝑟
𝑂𝑙𝑑 𝑆𝑎𝑙𝑎𝑟𝑦
= 𝑁𝑒𝑤 𝑠𝑎𝑙𝑎𝑟𝑦
100 250
4000
= 𝑁𝑒𝑤 𝑆𝑎𝑙𝑎𝑟𝑦
250×4000
∴ New Salary = 100
= 10,000
∴ The Dearness allowance to be paid to the employee = ₹(10,000–4000) = ₹6000.
4. In year 2XX6 the wholesale price index number was 250 with 2XX1 as base year, then how much
prices have increased in 2XX6 in comparison to 2XX1
Ans. Price index of base year 2XX1 = 100.
Price index of current year 2XX6 = 250.
Price increase in 2XX6 = (250–100)
= 150
Increased % = 150%

www.escholars.in
78
5. The index that measures the change from month to month in the cost of a representative basket
of goods and services of the type which are bought by a typical household?
Ans. Retail price index measures the change from month to month in the cost of a representative
basket of goods and services of the type bought by a typical household.
6. The number of test of Adequacy is ______
Ans. The number of test of Adequacy is 4

Note
Unit Test → The formula should be unit free
Time Reversal Test → 𝑃01 × 𝑃10 = 1
Factor Reversal Test → 𝑃01 × 𝑄01 = 𝑉01
Circular Test → 𝑃01 × 𝑃12 × 𝑃20 = 1
7. Price relative is expressed in terms of
Ans. 𝑃
Price Relative (P) = 𝑃𝑛 × 100
0
𝑃𝑛 = Price in the given year.
𝑃0 = Price in the base year.
8. The simple index number for the current year using simple aggregative method for the following
data is _____________
∑ 𝑃0 = 600 ∑ 𝑃1 = 1200
Ans. Simple Index Number = ∑ 𝑃1 × 100
∑𝑃
0
12000
= × 100 = 200
600
9. Wholesale Price Index (WPI) is given by ___________
Ans. WPI is based on Laspeyre’s Price Index
∑ 𝑃1 ∑ 𝑄0
∑ 𝑃0 ∑ 𝑄0
× 100
𝑇𝑜𝑡𝑎𝑙 𝐸𝑥𝑝𝑒𝑛𝑑𝑖𝑡𝑢𝑟𝑒 𝑖𝑛 𝑐𝑢𝑟𝑟𝑒𝑛𝑡 𝑌𝑒𝑎𝑟
= × 100
𝑇𝑜𝑡𝑎𝑙 𝐸𝑥𝑝𝑒𝑛𝑑𝑖𝑡𝑢𝑟𝑒 𝑖𝑛 𝐵𝑎𝑠𝑒 𝑌𝑒𝑎𝑟
10. Purchasing power of a money is ______________
Ans. Purchasing power of money is the reciprocal of price index number:
1
i.e. Purchasing power = 𝑃𝑟𝑖𝑐𝑒 𝑖𝑛𝑑𝑒𝑥 𝑛𝑢𝑚𝑏𝑒𝑟
This means when prices are rising, the purchasing power of money decreases and vice–versa.
11. In the data group Bowley’s and Laspeyre’s index number is as follows.
Bowley’s index number = 150, Laspeyre’s index number = 180, then Paasche’s index number is
Ans. Dorbish & Bowley’s Index Number =
𝐿𝑎𝑠𝑝𝑒𝑦𝑟𝑒 ′ 𝑠+𝑃𝑎𝑎𝑐ℎ𝑒 ′ 𝑠
2
180+𝑃𝑎𝑎𝑠𝑐ℎ𝑒𝑠
150 = 2
∴ Paasche’s Index Number = 120.
12. Consumer price index is commonly knows as.
Ans. Consumer Price Index is also called as Cost of Living Index.
It is a statistical measure that tracks the average price changes of a basket of goods and services
commonly purchased by households over time. It is calculated by comparing the price of this
basket of goods and services at different points of time.
13. Fisher’s Ideal Index does not satisfy ____________ test.
Ans. Fisher’s Ideal Index number does not satisfy circular test.
Note
Fisher’s Index Number satisfies unit test, Time reversal test and Factor reversal test.
Laspeyre’s Index Number and Paasches Index number satisfies only unit test.
www.escholars.in
79
14. In price Index, when a new commodity is required to be added, which of the index is used?
Ans. Shifted price Index is used.
It is a method used in index numbers to change the base period from which the index is
calculated.
15. The weighted averaged of price relatives of commodities, when weights are equal to the value of
commodities in the current year yields ______________ index number.
Ans. The weighted Averaged of price relatives of commodities, when the weights are equal to the
value of commodities in the current year yield paasche’s index number.

www.escholars.in
80
Logical Reasoning
Number Series, Coding, Decoding
and Odd Man Out Series
Q. No. Priority Questions
1. In a certain code, RIPPLE is written as 613382 and LIFE is written as 8192. How is PILLER
written in that code?
Ans.
R I P P L E L I F E
         
6 1 3 3 8 2 8 1 9 2

P I L L E R
     
3 1 8 8 2 6

2. In a certain code ‘256’ means ‘you are good’, ‘637’ means ‘we are bad’ and ‘358’ means ‘good
and bad’. Which of the following represents ‘and’ in that code?
Ans. 1) 2 5 6 → You Are Good
2) 6 3 7 → We Are Bad
3) 3 5 8 → Good And Bad
In the first and second codes, i.e., 256 and 637, we can see that the common digit is 6. Also, if
we look at their corresponding phrases, i.e., “You Are Good”, and “We Are Bad”, we can see
that the word “Are” is common. Therefore, 6 is the code for “Are”.
Similarly, in the first and third codes, i.e., 256 and 358, we can see that the common digit is 5.
Also, if we look at their corresponding phrases, i.e., “You Are Good”, and “Good and Bad”, we
can see that the word “Good” is common. Therefore, 5 is the code for “Good”.
Similarly, in the second and third codes, i.e., 637 and 358, we can see that the common digit
is 3. Also, if we look at their corresponding phrases, i.e., “We Are Bad”, and “Good And Bad”,
we can see that word “Bad” is common. Therefore, 3 is the code for “Bad”.
Now, let’s look at the last code along with its corresponding phrase:
358 → Good And Bad
We know that 3 corresponds to “Bad”, and 5 corresponds to “Good”.
Therefore, 8 would corresponds to “And”.
3. Find out the next number in the following series 7, 11, 13, 17, 19, 23, 25, 29?
Ans.

4. Find the next number in the series:


7, 23, 47, 119, 167
Ans. 32 – 2 = 7; 52 – 2 = 23; 72 – 2 = 47; 112 – 2 = 119; 132 – 2 = 167.
Therefore, we can see that every number in the sequence is 2 less than the square of a prime
number. The next prime number after 13 is 17. Therefore, the next number in the series will
be 172 – 2 = 287.
www.escholars.in
81
5. Find the odd man out 5, 10, 17, 27, 37:
Ans. 27 is the only perfect cube.
6. Find the missing value in the series 0, 2, 3, 6, 10, 17, 28, ?, 75.
Ans. 0+2+1=3
2+3+1=6
3 + 6 + 1 = 10
6 + 10 + 1 = 17
10 + 17 + 1 = 28
17 + 28 + 1 = 46
7. Find the missing value in ,
3 8
,
18
,?,
78
8 19 41 173
Ans. Numerators are increasing in the order +5, + 10, + 20, + 40.
Therefore, the numerator will be 18 + 20 = 38
Denominators are increasing in the order +11, +22, +44, +88.
Therefore, the denominator will be 41 + 44 = 85
8. Find the missing term: P 3 C, R 5 F, T 8 I, V 12 L, ?
Ans. First letter → P + 2 = R; R + 2 = T; T + 2 = V; V + 2 = X
Number → 3 + 2 = 5; 5 + 3 = 8; 8 + 4 = 12; 12 + 5 = 17
Third letter → C + 3 = F; F + 3 = I; I + 3 = L; L + 3 = O
Therefore, X17O
9. Choose the missing term in the series.
1, 1, 8, 4, 27, __________, 64, 16
Ans. The series n3, n2
For n = 1, we have 13 = 1; 12 = 1
For n = 2, we have 23 = 8; 22 = 4
For n = 3, we have 33 = 27; 32 = 9
10. It FRAME is coded as 0618011305 then ARISE is coded as __________.
Ans. F is the 6th letter of the English Alphabet, so the code of F is 06, and so on.
Therefore, the code of ARISE will be 01 18 09 19 05
11. What comes at the last place in R, U, X, A, D …………….?
Ans. We have R + 3 = U
U+3=X
X+3=A
A+3=D
D+3=G
12. Find the odd one from the following:
Ans. Hero, Zebra, Giraffe, Horse are herbivorous animal Except ‘Tiger’.
13. If in a certain code “THANKS” is written as “SKNTHA” then how is “STUPID” written?
Ans. The code of THANKS is made as follow →
The last three letters are reserved, and then the first three letters are written as it is.
Similarly, the code of STUPID would be DIPSTU.
14. Find the next number in the given sequence?
11, 17, 39, 85, ?, 281, 447

www.escholars.in
82
Ans.

15. If ROSE ‘is coded as 6821, CHAIR is coded as 73456 and PEACH is coded as 961473, what will
be the code for SEARCH?
Ans. Here,
R O S E C H A I R P R E A C H
              
6 8 2 1 7 3 4 5 6 9 6 1 4 7 3
Then,
SEARCH

214673

www.escholars.in
83
Direction Sense Test
Q. No. Priority Questions
1. Aditya walked 20 m towards North. Then he turned right and walks 30 m. Then he turns right
and walk 35 m. Then he turns left and walk 15 m. Finally, he turns left and walk 15 m. In
which direction and how many meters is he from the starting point?
Ans.

He is in 45 m in East.
2. One evening before sunset Aditya and Vaibhav were talking, to each other face to face. If
Vaibhav’s shadow was exactly to the right of Vaibhav. Which direction was Aditya facing?
Ans.

Aditya was facing towards South.


3. A person moves towards North upto n meters, then walked towards East upto 10 km, then
take right turn towards South and walked upto 10 km, again he moved towards West and
reached at start point. What is the value of n.
Ans.

n = 10
4. A man walks 2 km towards North. Then he turns to East and walks 10 km. After this he turns
to North and walks 3 km. After this he turns to North and walks 3 km. Again he turns towards
East and walks 2 km. How far is the from the starting point?
Ans.

www.escholars.in
84
AE2 = AO2 + OE2
AE2 = 122 + 55
AE = 13
5. The length and breath of a room are 8 m and 6 m respectively. A cats runs along all the four
walks and finally along a diagonal order to catch a rat. How much total distance is covered by
the cat?
Ans.

= Perimeter + AC
= 28 + √82 + 62
= 28 + 10
= 38
6. Shivam started from his house towards West. After walking a distance of 15 km he turned to
the right and walked 10 km. He again turned to the right and walked 5 km. After this he is to
turn right at 135° and covered 10 km. In which direction should he go?
Ans. S–W

7. Reena walked from A to B in the East 10 Feet. The she turned to the right and walked 3 Feet.
Again she turned to the right and walked 14 Feet. How far is she from A?
Ans.

AD = 5FT
8. If A × B means A is to the South of B, A + B means A is to the North of B, A% B means A is to
East of B, A – B means A is to the West of B, then in P% Q + R – S, S is in which direction with
respect to Q.
Ans. S–E

www.escholars.in
85
9. Madhuri moved a distance of 75 meters towards north. She then turned to her left and walked
for about 25 m, turned left again and walked 80m. Finally, she turned to her right at an angle
of 45°. In which direction was she moving now?
Ans.

10. Mohan started from a point and walked towards west. He took left to reach Sohan’s house. In
which direction should he move to reach his house:
Ans.

So, Mohan should move in north east direction from Sohan’s house to reach his house.
11. A and B start moving towards each other from two places 200m apart. After walking 60m, B
turns left and goes 20m, then he turns right and goes 40 m. He then turns right again and
comes back to the road on which he had started walking. If A and B walk with the same speed,
what is the distance between them now?

Ans.

Here AK + KF + FC + CB = 200 m
60 = KF + 40 + 60 = 200
KF = 200 – 40 – 60
KF = 40

www.escholars.in
86
12. Five Friends A, B, C, D, E are staying in the same locality. B’s house is to the East of A’s house
and to the North of C’s house. C’s house is to the West of D’s house. D’s house is in which
direction with respect to A’s house?
Ans.

13. A sign board pointing direction towards north due to heavy wind. The points of sign word
show west instead of North. If a person moves to same direction of pointer. He moves 100
meter than turn left, and moves 100 meter than again turn left and move 100 meter than he
turn right & moves 100 meter. In which direction he is now?
Ans.

He is in south direction from starting point ‘Now’


14. Radha moves towards South–East a distance of 7 km, then she moves towards West and
travels a distance off 14 km. From here she moves towards North–West a distance of 7 km.
and finally she moves a distance of 4 km. towards East. How far is she now from the starting
point?
Ans.

Here,
BC = DA
BC = OE + EA
14km = 4 km + EA
EA = 14 km – 4 km.
EA = 10 km
www.escholars.in
87
15. It is 3’o clock in a watch. If the minute hand points towards the North–East then the hour
hand will point towards the:
Ans.

If the minute hand points towards North–East, then the hour hand will point towards the
South–East.

www.escholars.in
88
Seating Arrangements
Q. No. Priority Questions
1. a) Eleven students A, B, C, D, E, F, G, H, I, J and K are sitting in first line facing to the teacher.
b) D who is just to the left of F, is to the right of C at second place.
c) A is second to the right of E who is at one end.
d) J is the nearest neighbour of A and B and is to the left of G at third place.
e) H is next to D to the left and is at the third place to the right of I who is just in the middle?
Sol. I is in the middle.
Teacher

E K A J B I G C H D F
2. A, B, C, D and E are sitting on a bench, A is sitting next to B, C is sitting next to D, D is not sitting
with E who is on the left end of the bench, C is o the second position from the right. A is to the
right of B and E. A and C are sitting together, In which position A is sitting?
Sol. A is sitting in between B and C

E B A C D
3. Directions to solve
a) P, Q, R, S, T, U V and W are sitting round the circle and are facing the centre.
b) P is second to the right of T who is the neighbour of R and V.
c) S is not the neighbour of P
d) V is the neighbour of U
e) Q is not between S and W. W is not between U and S.
Hint
Q
W P

S R

U T
V

i) Which two of the following are not neighbour


Sol. RV
ii) Who is immediate right to the V?
Sol. T
iii) What is the position of S?
Sol. c) To the immediate right of W and immediate left of U
4. Six Friends are sitting in a circle and are facing the centre of the circle. Deepa is between
Prakash and Pankaj. Priya is between Mukesh and Lalit. Prakash and Mukesh are opposite to
each other.

Hint: Prakash
Lalit Deepa

Priya Pankaj

Mukesh

www.escholars.in
89
i) Who is sitting right to Prakash?
Sol. Lalit
ii) Who is just right to Pankaj?
Sol. Deepa
iii) Who are the neighbours of Mukesh?
Sol. Priya and Pankaj
iv) Who is sitting opposite to Priya?
Sol. Deepa
5. Six Friends P, Q, R, S, T and U are sitting around the hexagonal table each at one corner and
are facing the centre of the hexagonal. P is second to the left of U. Q is neighbour of R and S. T
is second to the left of S.
Hint: P
T R

U Q

S
i) Which one is sitting opposite to P?
Sol. S
ii) Who is the fourth person to the left of Q?
Sol. P
iii) Who are neighbours of P?
Sol. T&R
iv) Which one is sitting opposite to T?
Sol. Q
6. Five senior citizens are living in a multi-storeyed building. Mr. Manu lives in a that above Mr.
Ashokan, Mr. Lokesh in a flat below Mr. Gaurav, Mr. Ashokan lives in a flat below Mr. Gaurav
and Mr. Rakesh lives in a flat below Mr. Lokesh. Who lives in the topmost flat?
Sol. Mr. Manu
Mr. Gaurav
Mr. Lokesh
Mr. Rakesh
Mr. Ashokan
Mr. Manu lives in the top most flat.
7. Six children A, B, C, D, E and F are sitting in a row. B is between F and D. E is between A and
C. However, A does not sit next to F or D. C does not sit next to D. Then, F is sitting between
Sol.
A E C F B D
F is sitting between B and C
8. Six flats on a floor in two rows facing North and South are allotted to P, Q, R, S, T and U. Q gets
a North facing flat and it is not next to S. S and U get diagonally opposite flat. R next to U gets
a South facing flat and T gets a North facing flat. Whose flat is between Q and S?
Sol. N
U R P

W E

Q T S S
Flat ‘T’ is between Q & S.

www.escholars.in
90
9. Four girls are seated for a photograph. Shikha is to the left of Reena. Manju is to the right of
Reena. Reeta is between Reena and Manju. Who is the second left in photograph?
Sol.
Shikha Reena Reeta Manju
10. A, B, C and D are playing cards, A and B are partners. D faces towards North. If A faces West,
then who faces south?
Sol. a)
C
N

B A W E

D S
If A and B are Partner.
Here A faces towards west then B faces towards east. C and D are also partner.
‘D’ faces towards North then C faces South direction.
11. A, B, C, D, E, F and G are sitting in a row facing north:
1) F is to the immediate right of E.
2) E is 4th to the right of G.
3) C is the neighbor of B and D.
4) Person who is third to the left of D is at one of ends.
Who are to the right of D?
Sol. a)

G B C D E F A
So E, F, A are to the right of ‘D’
12. Four ladies A, B, C and D and four Gentlemen E, F, G and H are sitting in a circle around a table
facing each other.
i) No two ladies or gentleman are sitting side by side.
ii) C, who is sitting between G and E, is facing D.
iii) F is between D and A facing G.
iv) H is to the right of B.
Who is immediate neighbour of B?
Sol.
A
E F

C D

G H
B
13. In a line, P is sitting 13th from left. Q is sitting 24th from the right and 3rd left from P. How many
people are sitting in the line?
Sol. Q is sitting 3rd left of P, and Q is the 24th position from the right.
…. 10th 11th 12th 13th 14th 15th ……
Q P

Since Q is 24th and 3rd left of P, it means Q is on the 10th place from the left.
This means that counting the position of Q, there are 24 positions to the right.
Therefore, counting from 11th position, there are 23 positions to the right.
Therefore, total number of places = 10 + 23 = 33.
www.escholars.in
91
14. Five boys A, B, C, D, E are sitting in a row. A is to the right of B and E is to the left of B, but to
the right of C. A is to the left of D. Who is second from the left end?
Sol.
C E B A D
‘E is the second from the left end’
15. P, Q, R and S are playing a game of carrom. P, R and S, Q are partners, ‘S’ is to the right of ‘R’.
If ‘R’ is facing West, then ‘Q’ is facing which direction?
Sol.
N
S
P R
W E

Q
S
Q is facing North direction.

www.escholars.in
92
Blood Relations
Q. No. Priority Questions
1. Pointing to a lady in the photograph, Monika said, “Her son’s father is the son–in–law of my
mother” How is Monika related to the lady?
Ans.

Lady’s son’s father is lady’s husband. Thus Monika is a lady’s sister.


2. Pointing to a photograph Lata says, “He is the son of the only son of my grandfather”. How is
the man in the photograph related to Lata?
Ans.

The man in the photograph is the only son of Lata’s grandfather i.e., the man is the son of
Lata’s father. Hence, the man is the brother of Lata.
3. Pointing a photograph X said to his friend Y, “she is the only daughter of the father of my
mother”, How X is related to the person of photograph?
Ans.

Son
4. If A + B means A is the sister of B, A × B means A is the wife of B, A% B means A is the father
of B and A – B means A is the brother of B. Which of the following means T is the daughter of
P?
Ans. P × Q → P is the wife of Q.
Q % R → Q is the father of R.
R – T → R is the brother of T.
T + S → T is the sister of S.
Therefore, T is the daughter of P.
P×Q%R–T+S

www.escholars.in
93
5. A $ B means A is the father of B. A # B means A sis the sister of B, A * B means A is the daughter
of B and A @ B means A is the brother of B. Which of the following indicates that M is the wife
of Q?
Ans. $ = Father
# = Sister
* = Daughter
@ = Brother
Q$R@T*M
Q is father of R
R is brother of T
T is daughter of M
Hence, M is wife of Q.
6. P and Q are brothers, R and S are sisters. P’s son is R’s brother. How is Q related to R
Ans.

‘Q’ is the uncle of ‘R’


7. Ram and Mohan are brothers, Shankar is Mohan’s father. Chhaya is Shankar’s sister. Priya is
Shankar’s niece. Shubhra is Chhaya’s grand daughter. How is Ram related to Shubhra?
Ans. Let me to Ram. Mohan is my brother. Shankar is Mohan’s father, i.e., my brother’s father, and
hence my father, Chhaya is my father’s sister, hence, my aunt. Priya is Shankar’s niece, i.e.,
my father’s niece, and hence, my cousin. Shubhra is Chhaya’s grand–daughter, i.e., my aunt’s
grand–daughter, and hence, my niece. Therefore, I’m Shubhra’s uncle.
8. Pointing the old man Kailash said “his son is my son’s uncle”. How is Kailash is related to old
man.
Ans.

9. Pointing to a lady in a photograph, Ram said “Her son’s father is the son in law of my mother”.
How is Ram related to the lady?
Ans.

Ram is the brother of the lady.


10. Pointing to a lady, Sahil said, “She is the daughter of the woman who is the mother of the
husband of my mother”. Who is the lady to Sahil?

www.escholars.in
94
Ans.

11. R told to M as, “the girl, I met at the beach, was the youngest daughter of the brother–in–law
of my friend’s mother”. How is the girl related to R’s friend?
Ans. Let me be R’s friend. Now, my mother’s brother–in–law would be either my paternal uncle,
or my maternal aunt’s husband. Either way, the daughter would be my cousin.
12. P, Q, R, S, T, U are 6 members of a family in which there are two married couples. T, a teacher
is married to a doctor who is mother of R and U. Q the lawyer is married to P. P has one son
and one grandson. Of the two married ladies one is a housewife. There is also one student
and one male engineer in the family. Which of the following is true about the granddaughter
of the family?
Ans. The family tree is as follows:
Q is the male lawyer married to P who is a housewife. They have a son, T, who is a teacher. T
is married to S, who is a doctor. They have two children → R and U. Out of them, one is a
male engineer, and the other is a student. Therefore, the granddaughter of the family is a
student.
13. In a joint family, there are father, mother, 3 married sons and one unmarried daughter. Out
of the sons, two have 2 daughters each and one has a son only. How many female members
are there in the family?
Ans.

Total female members in the family = 6


14. Pointing to a man in the photograph. Khushi says, “This man’s son’s sister is my mother–in–
law.” How is the Khushi’s husband related to the man in the photograph?
Ans. “Khushi husband is the grand son of the man.”

15. If A $ B means A is father of B. A # B means A is daughter of B. A @ B means A is sister of B.


Then how is K related to M H @ K $ L # M
Ans. Means Sibling, = Pairs
– + –
H – K = M

L
K is the husband of M

www.escholars.in
95

You might also like